You are on page 1of 206

.

Mixtilinear circle in olympiad


geometry
Theories - Examples - Extra - Problems - Solutions

Olympiad geometry document for reference

Author: Dogu , Paramizo Dicrominique , Dsoong

First edition - May 2023

⋆⋆⋆
Olympiad geometry document for reference 1

Content

• 1. Introduction (Page 2)

• 2. Theories (Page 4)

• 3. Examples (Page 20)

• 4. Extra (Page 45)

• 5. Problems (Page 54)

• 6. Solutions (Page 67)

• 7. Tests (Page 188)

• 8. Lemmas & Definitions list (Page 193)

• 8. Sources & Signs & Final Messages (Page 200)


Olympiad geometry document for reference 2

Introduction

First of all, I , Dogu want to say thank you for everyone who helped me completed writing
this handout. Including Paramizo Dicromonique, Dsoong, the other authors, from Viet
Nam and a major contributor: k12byda5h, from Thailand.

I also appreciate the large helps from Skydning, from Serbia; Ravi Painuly, from India;
Tafi ak, from Bangladesh; MAF, from Hong Kong;

In this handout,I want to introduce beginners about Mixtilinear circle,also introduce geome-
ters to some very interesting,new geometry problems about Mixtilinear circle. Mixtilinear
circle is a very classic theory to talk about when it comes to Olympiad Geometry. It first
appeared in Japanese Sangaku (Temple geometry) and researched by Japanese. In recent
years,mixtilinear appears many time in national olympiads and even international like IMO.
Therefore, Mixtilinear is an important topic people should know about. In this handout we
will gives out theories, problems and solutions about the Mixtilinear circles, from the most
basic theorem like Sawayama to the hardest problems by some extreme geometers.
Olympiad geometry document for reference 3

In conclusion, thank you for choosing and trusting our handout. We will always appreciate
your supports,suggestions with comments and then improve our works.

Regards,

土偶
Olympiad geometry document for reference 4

Theories

Message: Now we will present definitions,some well known lemmas with this Mixtilinear
configuration. During the solving problems, we will use the Lemmas included here and not
re proving it.

Definition M.D.1: The internal/external mixtilinear circle is the circle tangent to two
sides of a triangle and internally/externally tangent to the circumcircle of that triangle.

Diagram M.Di.1

Definition M.D.2: The internal/external A−mixtilinear circle is denoted to be the Mix-


tilinear circle respect to angle ̸ A of triangle △ABC.The A−mixtilinear of triangle △ABC
is tangent to AB, AC and internally/externally tangent to (ABC).
Olympiad geometry document for reference 5

Diagram M.Di.2

Comments: Usually internal mixtilinear circle appear more usually in Olympiads than
external mixtilinear circle.

Message: So how can we construct the Mixtilinear circles and its parts the fastest way?
Here are our tips:

Base on Theorem M.T.1 .To construct the mixtilinear circle, construct the incircle center
I then from I perpendicular to IA intersects AB, AC at E, F . The lines from E, F perpen-
dicular to AB, AC cut at J so J is the A−mixtilinear circle center and construct (J; JE) or
(J; JF ) will give you the A−mixtilinear circle.

In addition, to construct the A−mixtilinear incircle the fastest way,base on Theorem M.T.3
we will take midpoint of arc BAC of (ABC) is P , then let P I intersects (ABC) again at
X.X is the point we want. The mixtilinear excircle construction is exactly the same,but
with the excenter.

There is also a very nice way of constructing the circle like this, construct two circles
(O1 ), (O2 ) with RO1 > RO2 and they are internally tangent. A lie on (O1 ), tangent lines
from A to (O2 ) intersect (O1 ) at B, C. Now we get △ABC with A−mixtilinear incircle as
desired. The mixtilinear excircle is constructed with the same idea.
Olympiad geometry document for reference 6

Theorem M.T.1: (Sawayama - Thébault Lemma) Let △ABC be a triangle with incircle
center I.The internal A−mixtilinear circle of △ABC is tangent to segments AB, AC at E, F .
Then E, I, F and I is the midpoint of EF .

Proof 1:

Diagram M.Di.3

Let N be the midpoint of arc AB not containing C of (ABC). Let I ′ be the intersection
of EF, CN . Since ̸ I ′F X = ̸ BEX = ̸ XCN = ̸ I ′ CX =⇒ I ′ XCF is cyclic =⇒
̸ I ′ F C = ̸ XF C = ̸ XEI ′ =⇒ △N EI ′ ∼ △N I ′ X =⇒ N I ′2 = N E · N X = N A2 = N I 2
=⇒ I ≡ I ′ . Since △AEF is isosceles so clearly I is the midpoint of EF .

Q.E.D.

Message: Now we will introduce the second proof.

Proof 2: Let BI intersects (ABC) again at M .By homothety so X, E, N , X, F, M . Apply


pascal for  
N, A, M
B, X, C
so E, I, F . Since AE = AF and AI bisects ̸ EAF so it is easily to see that I is the midpoint
of EF .

Q.E.D.

Comments: With the same idea of proving and the idea of changing the configuration
from internal to external, we also have this theorem: If we let the external A−mixtilinear
Olympiad geometry document for reference 7

circle of △ABC tangent to AB, AC at U, V respectively so the A−excenter of △ABC is the


midpoint of segment U V .

Diagram M.Di.4

We will keep the same notation with Theorem M.T.1

Theorem M.T.2: B, X, I, E lie on a same circle;C, F, I, X lie on a same circle

Proof: See proof of Theorem M.T.1 and do the same for vertice B.
Olympiad geometry document for reference 8

Diagram M.Di.5

Theorem M.T.3: If P is the midpoint of arc BAC of (ABC). Then X, I, P .

Proof:

Diagram M.Di.6

By Theorem M.T.2. We get BXIE, CXIF are cyclic. So that ̸ BXI = ̸ AEF =
̸ AF E = ̸ CXI =⇒ XI is the angle bisector of ̸ BXC hence X, I, P .

Q.E.D.

Comments: Similarly, readers can change the configuration from internal to external. From
the theorem we have an important lemma, IX is the I−symmedian line of △IBC since
P B, P C are tangent lines of (IBC).

Theorem M.T.4: XA, XI are isogonal in ̸ EXF (Or we can say ̸ EXA = ̸ F XI)

Proof:

Notice that XA is the X−symmedian line of △EXF and XI is the X−median line of
△EXF so it is well known that XA, XI are isogonal in ̸ EXF .

Q.E.D.

Comments: From here we result in a very important lemma. If M, N are midpoints of


Olympiad geometry document for reference 9

minor arcs AB, AC so quadrilateral XM AN is harmonic. Combine with Theorem M.T.2


we also get quadrilaterals BXIE, CXIF are harmonic. (Project through B, C to (ABC)).

Theorem M.T.5: The A−excircle is tangent to segment BC at D′ . Then AX, AD′ are
isogonal in ̸ BAC.

Theorem M.T.6: (EGMO 2013) The tangent line of the A−mixtilinear circle of △ABC
which is parallel to BC touch the A−mixtilinear circle at K (nearer to A). Then AX, AK
are isogonal in ̸ BAC.

Proof:

Diagram M.Di.7

For Theorem M.T.5

Let M be the midpoint of BC. Incircle (I) is tangent to BC at D. Let P be the midpoint
of arc BAC of (ABC) so by Theorem M.T.3 =⇒ X, I, P . Let P M intersects (ABC)
again at Q so clearly Q is the midpoint of arc BC not containing A of (ABC). It is well
known that A, D′ , D and IM//AD′ . We have QM · QP = QB 2 = QI 2 =⇒ ̸ XAQ =
̸ XP Q = ̸ IP Q = ̸ M IQ = ̸ D AI .Since AI is the bisector of ̸ BAC so clearly AX, AD′

are isogonal in ̸ BAC.


Olympiad geometry document for reference 10

Q.E.D.

For Theorem M.T.6

Let DD∗ be the diameter of (I) so it is well known that A, D′ , D∗ .By homothety we get
A, D∗ , K, using Theorem M.T.5 we get the result.

Q.E.D.

Comments: From the theorem and Steiner ratio lemma we have a very important note like
this: If AX cut BC at A1 so

A1 B D′ B AB 2 A1 B AB 2 D′ C AB 2 DB
· ′ = ⇐⇒ = · = ·
A1 C D C AC 2 A1 C AC 2 D′ B AC 2 DC

Also by cyclic quadrilateral ratio lemma we have

XB AC A1 B AC DB AB 2 AB DB
= · = · · 2
= ·
XC AB A1 C AB DC AC AC DC

From the problem, we also have a very important property that, XI passes through K and
A(XK, IP ) = −1, for the proof, by Theorem M.T.3 so X, I, P ,by Theorem M.T.6
so AI bisects ̸ XAK.By homothety so X, K, P so X, I, K, P and A(XK, IP ) = −1 by
Apollonius circle property.

Theorem M.T.7: XA, XD are isogonal in ̸ BXC.

Proof:
Olympiad geometry document for reference 11

Diagram M.Di.8

Let M be the midpoint of BC. Incircle (I) is tangent to BC at D. Let P be the midpoint of
arc BAC of (ABC) so by Theorem M.T.3 =⇒ X, I, P . Let P M intersects (ABC) again
at Q so clearly Q is the midpoint of arc BC not containing A of (ABC). By Theorem
M.T.5 and AI is the bisector of ̸ BAC so ̸ IAD′ = ̸ XAI = ̸ XAQ = ̸ XP Q = ̸ XID
=⇒ ̸ XIQ = ̸ XID + ̸ DIQ = ̸ QIM + ̸ DIQ = ̸ DIM =⇒ △IDM ∼ △IXQ (Since
̸ IXQ = ̸ IAP = 90◦ = ̸ IDM .) =⇒ ̸ IXD = ̸ IQM = ̸ AXI. By Theorem M.T.3
XI bisects ̸ BXC =⇒ XA, XD are isogonal in ̸ BXC.

Q.E.D.

Theorem M.T.8: AX passes through the exsimilicenter of (O), (I)

Proof: Trivia by Monge D’ Alembert theorem.

Theorem M.T.9: AX intersects EF at T ,M is the midpoint of BC. Then IM bisects


AT .

Proof:
Olympiad geometry document for reference 12

Diagram M.Di.9

Let IM cut AX at S.Let D′ be the tangency point of the A−excircle to BC.It is well known
that IM//AD′ so ̸ SIA = ̸ IAD′ = ̸ IAS (By Theorem M.T.5) also ̸ AIT = 90◦ so S is
the midpoint of AT .

Q.E.D.

Theorem M.T.10: Let Q be the midpoint of arc BC not containing A of (ABC). Y, Z be


the tangency point of B−mixtilinear,C−mixtilinear circles with (ABC). Then Y Z, EF, BC, QW
are collinear.

Proof:
Olympiad geometry document for reference 13

Diagram M.Di.10

Let EF cut BC at W . By homothety XE, XF passes through the midpoints of arcs AB, AC
so XE, XF bisects ̸ BXA, ̸ CXA.

By Menelaus:

F A EB W C WC F C EA XB XA XB
· · = 1 ⇐⇒ = · = · =
F C EA W B WB F A EB XA XC XC

So XW is the external bisector of ̸ BXC so W, X, Q.

By angle chasing we have ̸ W IB = ̸ W CI so W I 2 = W B · W C. Let U, V be the tangencies


of the B−, C−mixtilinear circles with BC.By homothety Q, U, Y , Q, V, Z. Since DU · DY =
DB 2 = DV · DZ so ZY U V is cyclic. By simple angle chasing we can prove ̸ W IV = ̸ IU B
so W U · W V = W I 2 . So that W U · W V = W I 2 = W B · W C =⇒ W lies on the radical
axis of (ZY U V ), (O) so Y Z passes through W .

Q.E.D.

Theorem M.T.11: EF cut BC at W .So W is the exsimilicenter of the B−, C−mixtilinear


circles.

Proof: Result of Theorem M.T.10 and Monge D’ Alembert theorem.


Olympiad geometry document for reference 14

Theorem M.T.12: (Iran TST 2012) EF cut AX at T ,XI cut BC at S. Then T S ⊥ EF


and T S bisects ̸ BT C.

Proof:

Diagram M.Di.11

Let M be the midpoint of BC.P be the midpoint of arc BAC,Q be the intersection of P M
with (ABC) or it is the midpoint of arc BC. EF cut BC at W . By Theorem M.T.10
=⇒ W, X, Q. By Theorem M.T.3 =⇒ X, S, I, P and we also have (W S, BC) = −1,
so if we can prove T S ⊥ EF so by Apollonius circle property T S bisects ̸ BT C. The
task is now to prove T S ⊥ EF or W T SX is cyclic. It is easy to see that SM QX is
cyclic .We have ̸ W SX = ̸ P SM = ̸ P QX = 90◦ − ̸ IP M . Since QI 2 = QM · QP so
90◦ − ̸ IP M = 90◦ − ̸ M IQ = 90◦ − ̸ D′ AI = 90◦ − ̸ XAI (Because of Theorem M.T.3)
= ̸ AT I = ̸ W T X.Hence W T SX is cyclic.

Q.E.D.

Message Now we will switch to properties which include two others mixtilinear circles (of
vertice B, C)

Theorem M.T.13: Let Y, Z be the tangencies of B−, C−mixtilinear circles with (ABC).
O is the circumcenter of △ABC. So AX, BY, CZ, OI are concurrent.
Olympiad geometry document for reference 15

Proof 1:

Diagram M.Di.12

By Theorem M.T.3, if D′ , E ′ , F ′ are tangencies of the excircles to the sides of △ABC so


AX, BY, CZ are concurrent at the point J which is the isogonal conjugate of the concurrency
of AD′ , BE ′ , CF ′ ,called the Nagel point Na . Since Na lie on the Feuerbach hyperbola which
is isogonal conjugate to OI so the isogonal conjugate J of Na lie on OI.

Q.E.D.

Proof 2:

By Theorem M.T.8, AX, BY, CZ passes through the exsimilicenter of (O), (I).

Q.E.D.

Theorem M.T.14: Let Ob , Oc be the centers of the B−, C−mixtilinear circle in △ABC.
So XOb , XOc are isogonal in ̸ BXC.

Proof:
Olympiad geometry document for reference 16

Diagram M.Di.13

Let Ob Oc cut BC at W by Theorem M.T.11 so XW is the external bisector of ̸ BXC and


XI is the internal bisector of ̸ BXC by Theorem M.T.3. So that (BC, (BC∩XI)W ) = −1
=⇒ (Ob Oc , (XI ∩ Ob Oc )W ) = −1 (Project center I). By Apollonius circle property so XI
bisects ̸ Oc XOb .

Q.E.D.

Theorem M.T.15: The hexagon formed by connecting the tangencies by order of the
A−, B−, C−mixtilinear with the sides of △ABC is the hexagon outscribed incircle (I).

Proof:
Olympiad geometry document for reference 17

Diagram M.Di.14

By Theorem M.T.1 or Sawayama - Thébault lemma we get I is the midpoint of HU, V L


where U, H are tangencies of B−mixtilinear circle with the sides and V, L are defined simi-
larly. Since U V is tangent to (I) so HL also tangent to (I). Similarly, we are done.

Q.E.D.

Theorem M.T.16: (AIX) is orthogonal to (ABC)

Theorem M.T.17: (AIX), (BIY ), (CIZ) are coaxial with X, Y, Z are tangencies of the
A−, B−, C−mixtilinear circle with (ABC).

Proof:
Olympiad geometry document for reference 18

Diagram M.Di.15

For Theorem M.T.16:

O is the circumcenter of △ABC and Q is the midpoint of arc BC. ̸ OAQ = ̸ OQA =
̸ AXI =⇒ OA is tangent to (AIX).

Q.E.D.

For Theorem M.T.17:

Denote ωa , ωb , ωc to be three mixtilinear circles By Theorem M.T.15 ,PO/(Oi ) = R2 (i =


a, b, c). Since these three circles passes through I so OI is there radical axis. Also by
Theorem M.T.15: we can prove the exsimilicenter of (O), (I) which is J satisfying PJ/(Oi ) =
R2 (i = a, b, c) so we are done.

Q.E.D.

Message: We will let readers prove this theorem on there own after reading those lemmas
in front (Hint: directed angle)

Theorem M.T.18: If C ′ , B ′ are midpoints of arc AB, AC so (AC ′ E) intersects (AB ′ F )


again at a point on K lie on EF and AC ′ KB ′ is a parallelogram.
Olympiad geometry document for reference 19

Diagram M.Di.16

Message: Now we will start up with some examples


Olympiad geometry document for reference 20

Examples

Message First examples are easy problems for warming up.

Example M.E.1: Let △ABC be a triangle with altitudes BE, CF intersects at ortho-
center H. Tangents from B, C to (ABC) intersects at T .T B, T C intersects EF at X, Y
respectively.

i) (Turkey 2012) Prove that: (XY T ) is tangent to (ABC)

ii) The tangency point of (XY T ), (ABC) is the second intersection of (AH) and (ABC).

Proof:

Diagram M.Di.17

i) Let I be the midpoint of BC. Since ̸ XBF = ̸ ACB = ̸ AF E = ̸ XF B =⇒ △XBF


is isosceles. Similarly △Y EC is isosceles. Since IB = IF = IE = IC so IX ⊥ BF and
Olympiad geometry document for reference 21

IY ⊥ EC so XI, Y I are bisectors of ̸ BXF , ̸ CY E so I is the incenter of △XY Z also I is


the midpoint of BC so by the converse of Theorem M.T.1 (XY T ) is tangent to (ABC).

Q.E.D.

ii) Denote the intersection to be W .Let AP be the diameter of (ABC). It is well known
that W, H, I, P and it is also well known that T W, T P are isogonal in ̸ BT C ,note that the
tangent from P to (ABC) is parallel to EF ,so by Theorem M.T.6 W is the tangency
point.

Q.E.D.

Example M.E.2: Let △ABC be a triangle with incircle (I) tangent to BC at D. P is


the midpoint of arc BAC of (ABC). (P ; P D) intersects (I) and BC at W, D and D′ . Prove
that: AW, AD′ are isogonal in ̸ BAC.

Solution:

Diagram M.Di.18

Let the A−mixtilinear circle to tangent to (ABC) at K. Since IW = ID, P W = P D and by


Theorem M.T.3 K, I, P so KP is the perpendicular bisector of W D so D reflects W over
KP . Let KD cut (ABC) at Q,by Theorem M.T.7,Theorem M.T.3 we get KI bisects
̸ AKQ so that A, W, K.

Q.E.D.
Olympiad geometry document for reference 22

Message: Next problem is an application of the important ratios from Mixtilinear circles.

Example M.E.3: (Dogu) Let △ABC be a triangle with tangents from B, C to (ABC)
cut at T . AT intersects BC at L and S lie on AC such that SL//AB. Incircle (I) is tangent
to BC at D,R lie on AB such that DR//AC. The A−mixtilinear circle is tangent to (ABC)
at K. Prove that: AK, BS, CR are concurrent.

Solution:

Diagram M.Di.19

RB LB AB 2 DB SA
It is well known that = = 2
,by Thales = . AK intersects BC at A1
RC LC AC DC SC
so by the comments mentioned in the comments of Theorem M.T.5,Theorem M.T.6
AB 2 DB
we get d A 1B
A1 C
= . Multiply all the ratio,then by converse Ceva theorem we get the
AC 2 DC
result.

Q.E.D.

Example M.E.4: Let (O1 ), (O2 ) be two circles which is externally tangent at X.A point
S lie on (O2 ),the tangent from S to (O1 ) are SU, SV (U, V lie on (O1 )).U X, V S intersects
(O2 ) again at U ′ , V ′ .Prove that: U ′ V ′ bisects U V .

Solution:
Olympiad geometry document for reference 23

Diagram M.Di.20

Diagram M.Di.21

If we change external to internal it immediately becomes a very well known problem. The
proof is still exactly the same idea.

Message: Now we will come to a streak of similar looking problems

Example M.E.5: Let △ABC be a scalene triangle ,AB < AC with incircle (I) tangent
Olympiad geometry document for reference 24

to the sides at D, E, F. The A−mixtilinear circle is tangent to (ABC) at X. AX intersects


(I) at J nearer to A. Prove that: DJ ⊥ EF .

Solution:

Diagram M.Di.22

Let P be the midpoint of arc BAC,T D cut (ABC) at Z. Redefine R as the point lie on (I)
such that DR ⊥ EF . We have
̸ ABC ̸ ABC
̸ IRD = ̸ IDR = ̸ IDF − ̸ F DR = − (90◦ − ̸ DF E) = − (90◦ − ̸ CED) =
2 2
̸ ABC ̸ ACB
= − .
2 2

Also
̸ BAC ̸ BAC
̸ IT D = ̸ P AZ = ̸ P AB − ̸ ZAB = 90◦ + − 180◦ + ̸ ABC = 90◦ − + ̸ ABC
2 2

̸ ABC ̸ ACB
= − = ̸ IRD
2 2

=⇒ RIDT is cyclic. Since IR = ID so T R is the reflection of T D over T I, by Theorem


M.T.7,Theorem M.T.3 we get T I bisects ̸ AT R so A, R, X so does J.
Olympiad geometry document for reference 25

Q.E.D.

Example M.E.6: Let △ABC be a scalene triangle,AB < AC with incircle (I) tangent to
the sides at D, E, F. The A−mixtilinear circle is tangent to (ABC) at T . AD intersects the
A−mixtilinear circle at S nearer to A. Prove that: S, I, D, T lie on a circle.

Solution:

Diagram M.Di.23

Let (DIT ) cut (I) at R, by Example M.E.5 we get R lie on AT . Consider Inversion center
I,radius AI, clearly (I) becomes the A−mixtilinear circle and R becomes T . Let S ′ be AS
cut DIRT so S ′ becomes D after inversion ,since D lie on (I) so S ′ lie on the A−mixtilinear
circle so do S.

Q.E.D.

Example M.E.7: (Diaconescu Tashi) Let ABC be a triangle. Let T be the point of
tangency of the circumcircle of triangle ABC and the A-mixtilinear incircle. The incircle
of triangle ABC has center I and touches sides BC, CA and AB at points D, E and F,
respectively. Let N be the midpoint of line segment DF. Prove that the circumcircle of
triangle BT N, line T I and the perpendicular from D to EF are concurrent.

Solution:
Olympiad geometry document for reference 26

Diagram M.Di.24

Perform an inversion center at I,radius ID. Clearly (BT N ) becomes itself, S becomes T , D
becomes D. R is the intersection of AX and (I) (which is nearer to A).To prove D, S, R we
need to prove R, I, D, T lie on a circle which is Example M.E.5.

Q.E.D.

Example M.E.8: Let △ABC be a scalene triangle ,AB < AC with incircle (I) tangent to
the sides at D, E, F. The A−mixtilinear circle is tangent to (ABC) at X and AX intersects
(I) at R nearer to A.Let the perpendicular from I to IA intersects BC at W .Prove that:
W R is tangent to (I).

Solution:

From Theorem M.T.10 we get ̸ W T I = 90◦ so W lie on (RIDT ) since IW//EF so


IW ⊥ RD (RD ⊥ EF because of Example M.E.5) so W R is tangent to (I).

Q.E.D.
Olympiad geometry document for reference 27

Diagram M.Di.25

Message: Now we come with a very famous problem by Yetti,also appear in Taiwan
TST,which has many solutions, but I will present the most simple way. And after that
will be a streak of problems similar to it.

Example M.E.9: (Taiwan TST 2014) Let △ABC be a triangle with point E lie on
(ABC).Tangent lines from E to incircle (I) of △ABC intersects BC at X, Y . Prove that:
(EXY ) passes through the tangency point of the A−mixtilinear circle with (ABC).

Solution:
Olympiad geometry document for reference 28

Diagram M.Di.26

Let (EXY ) cut (ABC) again at W , we will prove W I bisects ̸ BW C hence by Theorem
M.T.3 W is the tangency point of the A−mixtilinear circle with (ABC).Let EX, EY in-
tersects (ABC) at M, N and Z be the midpoint of arc M N not containing E in (M N E).
By Poncelet porism, M N is tangent to (I), so we can get Z, I, E since (I) is the incircle of
△EM N . Let W Y cut (ABC) at H ,by Reim we get M H//BC =⇒ W M, W Y are isogonal
in ̸ BW C.Let EI cut (EXY ) again at U . Also ̸ U XY = ̸ U EY = ̸ U EM = ̸ M W Z so
we will need to prove W I bisects ̸ ZW U . Since I is the center of the E−excircle of △EXY
+
so I lie on (U ; U X) so U X = U I. By spiral homothety △W M Z ∼ △W XU so We have

ZM ZW
=
UX ZU
Also

ZI ZM
=
UI ZI

Combine two ratios we get


WZ IZ
=
WU IU
So W I bisects ̸ ZW U .

Q.E.D.

Example M.E.10: Let △ABC be a triangle with incircle (I). d is the tangent of (I) and
it is parallel to BC. P is a point lie on BC. Tangent line from P to (I) other from BC
intersects d at E. AE intersects (ABC) again at F . Prove that: P F passes through the
tangency point of the A−mixtilinear circle and (ABC).

Solution:

Let d cut AB, AI, AC at R, S, V . M is the midpoint of arc BC of (ABC).Let P ′ be the


intersection of XF and BC where X is the tangency point of the A−mixtilinear circle with
(ABC). The line from I ⊥ IA cut BC at W .By Theorem M.T.10 we get W, X, M .
Olympiad geometry document for reference 29

Diagram M.Di.27

We have (P ′ W, BC) = X(P ′ W, BC) = (F M, BC) = A(F M, BC) = (ES, RV ) = I(ES, RV ) =


I(P W, BC) = (P W, BC) so P ≡ P ′ .

Q.E.D.

Example M.E.11: Let △ABC be a triangle with P lie on (ABC).The tangents from P
to incircle (I) intersects (ABC), d at Y, X where d is the tangent of (I) satisfying d//BC.
Prove that: AX, AY are isogonal in ̸ BAC.

Solution:
Olympiad geometry document for reference 30

Diagram M.Di.28

Let P X, P Y intersects BC at J, S. Let AX cut (ABC) again at R. Let W be the tangency


of the A−mixtilinear circle so by Example M.E.9 we get (P SJ) passes through W . By
Example M.E.10 we get R, J, W . By Reim So RY //JS ≡ BC so AX, AY are isogonal in
̸ BAC.

Q.E.D.

Example M.E.12: Let △ABC be a triangle with incircle (I),d is a tangent line to (I)
and d//BC. A point P lie on (ABC). Draw tangents from P to (I) and they intersects d
at X, Y . Prove that: (P XY ) is tangent to the A−mixtilinear circle of △ABC.

Solution:

Diagram M.Di.29

AX cut (ABC) at Z,P Y cut (ABC) at W . By Example M.E.11 AX, AW are isogonal
in ̸ BAC =⇒ ZW//BC//XY .By Reim and ZW//BC so AP Y X is cyclic. K, L are
intersections of (P XY ) with AB, AC. By Poncelet porism, we get KL is tangent to (I).
By the excenter version of Theorem M.T.1 mentioned in the comments, we get (AKL) is
tangent to the A−mixtilinear circle of △ABC. (Draw the tangents of the mixtilinear circle
to AB, AC are U, V so I midpoint U V )

Q.E.D.
Olympiad geometry document for reference 31

Comments: Readers can try proving a similar problem like this;

Using the notation of Example M.E.9 ,prove that (EXY ) is tangent to a fixed circle.

There is also a super cool Generalization of this problem by Linyangyuan, Problem Let
ABCD be a cyclic quadrilateral with AC intersects BD at P . A circle ψ tangent to segments
P B, P C and internal tangent to (ABCD). P lie on (ABCD). (I) is the incircle of △P BC
and has a tangent line d which is parallel to BC. The tangents from P to (I) intersects d
at X, Y . Prove that: (P XY ) is tangent to ψ.

, We will not present the solution here, but if anyone is curious about the solution, here is
the link
https://artofproblemsolving.com/community/c6h1183392p5738215

Example M.E.13: Let (O) be circumcircle of △ABC. Let ωa , Ωa be A-mixtilinear in-


cricle and A-excircle respectively. Let A1 , B1 , C1 be touching points of Ωa and BC, CA, AB
respectively. Let A2 , B2 , C2 be touching points of ωa and (O), AB, AC respectively. Let X
be midpoint of arc BAC and let Y = B1 C1 ∩ BC, Z = B2 C2 ∩ BC. Prove that: A, X, Y, Z
are cyclic.

Solution:

Diagram M.Di.30

Draw AH ⊥ BC ,diameter AA′ . Ia is the A−excenter It is well known that IA ⊥ IZ


=⇒ (AIHZ) is cyclic. It is well known that Ia A′ cut M D′ on (BAC), lets call T (Switch
Olympiad geometry document for reference 32

the configuration from excircle to incircle and we get the well-known lemma) Since EF cut
BC at Y ,drop Ia D′ ⊥ BC we get (BC, D′ Y ) = −1. But M T is the bisector of ̸ BT C
=⇒ X, T, Y . It is well known that △AIH ∼ △AA′ Ia because △AIB ∼ △ACJ =⇒
cyclic
180 − ̸ AZH = ̸ AIH = ̸ AA′ Ia = ̸ AXT = ̸ AXZ =⇒ AXY Z is cyclic.

Q.E.D

Example M.E.14: (Serbia MO 2023) Given is a scalene triangle ABC with incenter I
and circumcircle ω (AB < AC). The incircle is tangent to BC at D. The perpendicular at
I to AI meets AB, AC at E, F and the circle (AEF ) meets ω and AI at G, H. The tangent
at G to ω meets BC at J and AJ meets ω at K. Prove that (DJK) and (GIH) are tangent
to each other.

Solution:

Diagram M.Di.31

XD intersects (ABC) at T ,using Theorem M.T.7 so AT //BC.Let the tangency of the


A−excircle be D′ ,AD′ cut (ABC) at U , from Theorem M.T.5 ,we have ̸ JKX = ̸ ACX =
̸ ACB + ̸ XCB = ̸ ACB + ̸ U BC = ̸ D′ U B + ̸ U BC = ̸ AD′ B = ̸ T DC = ̸ JDX =⇒
JKDX is cyclic.

With the idea of the ratio part in the proof of Theorem M.T.10, from that we have JX
Olympiad geometry document for reference 33

is tangent to (ABC). Let AP be the diameter of (ABC) so G, H, P . Let R be the midpoint


of arc BAC. We have ̸ GHX − ̸ JDX = ̸ HXP + ̸ HP X − ̸ ACX

Let O be the circumcenter of the triangle.We have ̸ HXP = ̸ OXP = ̸ OP X = ̸ ACX so


̸ GHX = ̸ HP X = ̸ JXG. Hence the circles are tangent.

Q.E.D.

Example M.E.15: (Jean-Louis Ayme) Let △ABC be a triangle with (O), (I), (Oa ) be the
circumcircle, incircle, A−mixtilinear incircle. P is the point of contact of (O) and (Oa ).D
the point of contact of (I) and BC. Let M the midpoint of ID.Let T the foot of the
perpendicular to AD through I. Prove that: T, M, D and P are cyclic.

Solution:

Diagram M.Di.32

(I) is tangent to AC, AB at E, F .Let EF cut BC at S. W be the midpoint of SD. By


simple angle chasing W T M D is cyclic. It is also well known that I, T, S Let AD cut (ABC)
at Z. To prove W, P, Z, we need to prove
WB P B ZB
=
WC P C ZC

Let R lie on BC such that BD = RC. Also we have


ZB DB AC P B wellknown KC RC AB P B ZB RC AB DB AC DB 2
= ; = = =⇒ = = .
ZC DC AB P C KB RB AC P C ZC RB AC DC AB DC 2
Olympiad geometry document for reference 34

Let W Q tangent to (I), it is well known that QD is the bisector of ̸ BQC since (SD, BC) =
−1 and W is the midpoint of SD. We have
WB tangent QB 2 DB 2 P B ZB
= = =
WC QC 2 DC 2 P C ZC

From here W, P, Z. Let P D cut (ABC) at L, it is well known that AL//BC (Theorem
W T =W D
M.T.7).Since ̸ DP Z = ̸ DAL = ̸ T DW = ̸ W T D =⇒ T DP W is cyclic. We have
mentioned W T M D is cyclic so T M DP is cyclic.

Q.E.D

Example M.E.16: (Le Viet An) Let ABC be a scalene triangle inscribed circle (O) with
AB < AC. Let (O′ ) be the circle wich is tangent to the circle (O) and the sides CA, AB
at D and E, F , respectively. The line BC intersects the tangent line at A of (O), EF and
AO′ at T, S and L, respectively. The circle (O) intersects AS again at K. Prove that the
circumcenter of triangle AKL lies on the circumcircle of triangle ADT .

Solution:

Diagram M.Di.33

Let (I) tangent to BC at X. Call the circumcenter of AKL is G.


Claim: A, X, D, T lie on a circle.

Proof: DX cut (ABC) at J. It is well known that AJ//BC. ̸ ADX = ̸ ADJ = ̸ ACJ =
̸ ABC − ̸ ACB = ̸ AT X .Hence proven.
Olympiad geometry document for reference 35

Claim: A, K, X, L lie on a circle

Proof: Let M X cut (ABC) at K. It is well known that K lie on (AI) and lie on AS
=⇒ K, X, M Hence by Inversion over (M, M B) ,we get the result.

Claim: D, I, G

Proof: Let N lie on DA such that DN = DX. Since it is well known that DI is the bisector
̸ BAC
of ̸ ADX ≡ ̸ ADJ =⇒ N lie on (I). Also ̸ ALX = 2
+ ̸ ACB and since DN = DX
̸ N DX ̸ ACJ ̸ ABC−̸ ACB ̸
, ̸ DN X = 90 − 2
= 90 − 2
= 90 − 2
= BAC
2
+̸ ACB = ̸ ALX =⇒
N lie on (AKXL). We have GN = GX, IN = IX =⇒ G lie on the bisector of ̸ ADJ and
so do I ,then we got the claim.

Back to the problem, Note that GA = GX by circumcenter, DG ≡ DI is the bisector


of ̸ ADX ≡ ̸ ADJ =⇒ A, G, X, D lie on a same circle. With the first claim =⇒
A, G, X, D, T lie on a circle, the problem has been proved.

Q.E.D

Example M.E.17: Let △ABC be a triangle with ωa is the A−mixtilinear circle. AP is


the diameter of (ABC). AI intersects BP at X. Prove that: (P ; P X) is orthogonal to ωa .

Solution:

Diagram M.Di.34
Olympiad geometry document for reference 36

Let I be the incenter, J be the center of ωa and ωa is tangent to AB, AC at E.F . From
Theorem M.T.1 so we get I is the midpoint of EF and AJ ⊥ EF .Let AI cut CP at Y ,
trivially Y lie on (P, P X). Notice that (IEBX) is cyclic. We have ̸ IBC = ̸ ABI = ̸ JXF
,also ̸ BIC = 90◦ + ̸ CAI = ̸ F JX =⇒ △JF X ∼ △IBC,similarly we get △JF X ∼
△IBC ∼ △JEY =⇒

JF JY
= =⇒ PJ/(P ;P X) = JE 2 = Rω2 a
JX JE

Q.E.D.

Example M.E.18: (Dogu) Let ABC be a triangle with A−mixtilinear circle tangent to
(ABC) at X.I,J are incircle center and A−excircle center of triangle ABC. The external
bisector of angle ̸ BAC intersects the perpendicular line from J to BC at Z.Prove that
AX, ZI, BC are concurrent.

Solution:

Diagram M.Di.35

Let M, N be the midpoints of arcs BC, BAC. U, V are intersections of AM, AN with BC.ZI
Olympiad geometry document for reference 37

cut BC at L. KI cut AN at Q. Let K be the tangency point of the A−excircle with BC.
We need to show (V U, LK) = −1 . Project with center I we need (V A, QZ) = −1 which is
true since K(V A, QZ) = K(DD′ , IZ∞ ) = −1 (where the incircle tangent to BC at D and
ID intersects (I) again at D′ ).

Q.E.D.

Message Let’s we switch to a combo of nice configuration problems. This configuration is


new,discovered recently.

Example M.E.19: Let I, O be the incenter and circumcenter of △ABC respectively. Let
K be the intersection of IO with BC. A circle tangent to AB,AC and (ABC) at the same
time tangent to (ABC) at T . Reflection of A over I is A′ . Prove that: T, A′ , K.

Solution:

Diagram M.Di.36

Let D be the tangency of (I) and BC.Q is the midpoint of arc BC of (ABC). Let S be
the midpoint of arc BAC of (O). Let P be the midpoint of arc BAC of (ABC) so by
Theorem M.T.3 we have T, I, P . Since I(DK, T A′ ) = I(DO, P Q) = −1 .So we will have
to prove D(IK, T A′ ) = −1 Let U be the intersection of T D with AI. We will need to prove
Olympiad geometry document for reference 38

(IR, U A′ ) = −1 where AI cut BC at R or it is equivalent to proving

IA′ IU

=
RA RU

We can easily prove that (Bisector ratios)

A′ I AQ

= ′
AR AI

In the proof of Theorem M.T.5 we have proved that △IAP ∼ △IT Q ∼ △IDN where N
midpoint BC so we can easily see ̸ IDU = ̸ IP A.

So that
IU sin(IDU ) DI ∼ AP QA QA QA AQ
= · = · = = =
RU sin(GDU ) DR AI AP AI IA′ A′ I

Q.E.D.

Example M.E.20: (Inequality.) Let I, O be the incenter and circumcenter of △ABC


respectively. Let K be the intersection of IO with BC. A circle tangent to AB,AC and
(ABC) at the same time tangent to (ABC) at T . ℓ is the tangent line of (IT K) passing K.
Prove that: ℓ//AI.

Solution:
Olympiad geometry document for reference 39

Diagram M.Di.37

P, Q are midpoints of arcs BAC, BC of (ABC). Let T K cut AI at A′ so by Example


M.E.19 we get IA = IA′ .

IA′ AI PI sin(KIQ) KI
= = = =
IT IT QI sin(KIP ) KT

=⇒ △KIA′ ∼ △KT I so by angle chasing we are done.

Q.E.D.

Example M.E.21: (Jean-Louis Ayme) Let △ABC be a triangle with Ω, ωb , ωc be the


circumcircle,B−, C−mixtilinear circles. ωb is tangent to Ω at F ,ωc is tangent to AB, AC at
P, Q. (F QC) intersects BC again at P ∗ . Prove that: P, P ∗ are reflection over the midpoint
of segment BC.

Solution:

Diagram M.Di.38

Let ωb tangent to AB, AC at D, E. Redefine P ′ is a point lie on BC such that BP = P ′ C.By


the proof of Theorem M.T.15 DQ//BC and by homothety F D passes through midpoint
Olympiad geometry document for reference 40

of arc AB so F D bisects ̸ AF B (This property mentioned in Theorem M.T.3) So that


FA DA QA QA
= = = ′
FB DB QC PB

+
Since ̸ F AQ = ̸ F BP ′ =⇒ △F AQ ∼ △F BP ′ so by spiral homothety =⇒ F QP ′ C is
cyclic. =⇒ P ′ ≡ P ∗ . Hence proven.

Q.E.D.

Example M.E.22: Let △ABC be a triangle with the A−mixtilinear incircle tangent to
AB, AC, (ABC) at E, F, X. I is the incenter and M is the midpoint of segment BC. CE
cut BF at S. AX cut EF at W . Prove that: W S//IM .

Solution:

Diagram M.Di.39

By Theorem M.T.9 ,If we let IM cut AX at P so P is the midpoint of segment AX. Let
N be the midpoint of segment AS. By Theorem M.T.1 so I is the midpoint of segment
EF , by Gauss line in quadrilateral property we get N, I, M so that P, N, I, M but we have
P N//W S by midline so IM//W S.

Q.E.D.
Olympiad geometry document for reference 41

Example M.E.23: (buratinogigle) Let ABC be a triangle inscribed incircle (O). Let
A-mixtilinear incircle touches (O) at D. Similarly, we have E, F . Let radical axis of B, C-
mixtilinear incircles intersects EF at X. Similarly, we have Z, Y . Prove that DX, EZ, F Y
are concurrent.

Solution:

Diagram M.Di.40

Construct the tangential triangle of △DEF is △M N P . Since M E = M F so M X is the


radical axis of the B−, C−mixtilinear incircles =⇒ M X passes through the radical center
W of the three mixtilinear incircles so M X, P Y, N Z are concurrent also M D, N E, P F are
concurrent at the Gergonne point of △M N P so by Ceva Nest theorem DX, EZ, F Y are
concurrent.

Q.E.D.

Comments: Although this problem looked hard but the idea of solving is very nice and
simple.

Example M.E.24: (Viet Nam TST 2014) Let △ABC be a scalene triangle with a point
Olympiad geometry document for reference 42

D lie on minor arc BC. Suppose CD, BD intersects AB, AC at E, F . (K) is the circle
lie inside △EBD and tangent to EB, ED, (ABC). (L) is the circle lie inside △F CD and
tangent to F C, F D, (ABC).

i) M is the tangency point of (K) with BE and N is the tangency point of (L) and CF .
Prove that: (M N ) passes through a fixed point when D moves.

ii) The line from M parallel to CE intersects AC at P ,the line from N parallel to BF
intersects AB at Q. Prove that: (AM P ), (AN Q) are both tangent to a fixed circle
when D moves.

Solution:

Diagram M.Di.41

i) It is well known that, by the Sawayama lemma, M Z passes through Ia - the A−excircle
center where (K) tangent DE at Z. So that M Ia ⊥ EK. Similarly F L ⊥ Ia N . By simple
angle chasing we get EK ⊥ F L so Ia lie on (M N ) and Ia here is the fixed point.

Q.E.D.

ii) Let the A−mixtilinear excircle of △ABC to touch AB, ACat U, V . By the excircle
version of Theorem M.T.1 we mentioned so Ia is the midpoint of U V . Since M P//EC so
Olympiad geometry document for reference 43

̸ Ia M P = ̸ M ZE = ̸ Ia M E so M Ia bisects ̸ EM P , similarly we get P Ia bisects ̸ M P N


so Ia is the A−excenter of △AM P , △AN Q. Again by the excircle version of Theorem
M.T.1 we mentioned,or the converse, we get the A−mixtilinear excircle of △ABC is also
the A−mixtilinear excircle of △AM P so (AM P ) is tangent to the A−mixtilinear excircle
of △ABC which is fixed. Also similar for (AN Q) , we are done.

Q.E.D.

Message: To end the Example section, we will introduce a very beautiful problem by
livetolove212

Example M.E.25: (livetolove212) Let △ABC be a triangle with incircle (I). The tangent
line of (I) which is parallel to BC intersects AB, AC at E, F . Ω, ωb , ωc are the incircle of
△AEF ,B, C−mixtilinear circles of △ABC. Prove that: Ω, ωb , ωc has a common tangent
line.

Solution:

Diagram M.Di.42

Let ωb , ωc tangent to BC at U, V . By the proof of Theorem M.T.15 we get E, F are the


tangency points of ωb , ωc with AB, AC. Let the common tangent of ωb , ωc different from BC
meet ωb , ωc at X, Y ,clearly they are reflections of V, U through the line connecting centers
of ωb , ωc . Let XY cut AB, AC at P, Q then P Q = XQ + Y P − XY = QF + P E − U V (By
Olympiad geometry document for reference 44

symmetric) = QF + P E − EF (Reflections over I) =⇒ P Q + EF = QF + P E so from that


and Pitot theorem we get P QF E has incircle Ω so Ω, ωb , ωc has a common tangent line.

Q.E.D.

Message: Now we will move to some Extra knowledge about Mixtilinear circles section
before entering the Problems & Solution section.
Olympiad geometry document for reference 45

Extra

Message: First of all we will come to the Generalizations of the Mixtilinear circle by
livetolove212. .

Extra M.Ex.1: (livetolove212) Let △ABC be a scalene triangle with a circle Ω tangent
to AB, AC at E, F . Suppose Ω intersects (ABC) at distinct P, Q. Tangents from P, Q to Ω
intersects at T . K is the midpoint of EF . N is the midpoint of arc BAC of (ABC). Prove
that: N, K, T .

Solution:

Diagram M.Di.43

Let the tangents from N to Ω be U, V . Clearly A, N, V, J, U lie on a circle which is (N J).


By inversion centered at J radius RΩ we can easily see that U, K, T ′ , V where T ′ midpoint
U V .By radical axis theorem we get AN, U V, P Q are concurrent at a point called R. Since
̸ JZR = ̸ JT R = ̸ JAR = 90◦ so A, T ′ , J, Z, R lie on a circle. Perform an inversion centered
at J, radius RΩ so A becomes K,T ′ becomes N ,Z becomes T and also AT ′ JZ lie on a circle
so N, K, T .
Olympiad geometry document for reference 46

Q.E.D.

Comments: We can see that this is the general problem of Theorem M.T.3

Message: We will use the notation of Extra M.Ex.1

Extra M.Ex.2: (livetolove212) Let M be the midpoint of minor arc BC. AT intersects
(ABC) again at S. Prove that: EF, BC, SM are concurrent.

Solution:

Diagram M.Di.44

Let EF cut BC at W . Let AT intersects Ω at X, Y where X is closer to A. Tangent at


X, Y to Ω intersects at G. Since quadrilaterals XEY F, XP Y Q are harmonic so G lie on
EF, P Q. By radical axis theorem so A, G, R where (AEF ) intersects (ABC) at R. Since
(XY, EF ) = (XY, P Q) = −1 so −1 = (GH, EF ) where H is the intersection of AT, EF .
Also A(GH, EF ) = (RS, BC) so RSBC is a harmonic quadrilateral. We have:

SB RB △REB∼△RF C BE M enelaus,AE=AF WB
= = =
SC RC CF WC

So SW is the external bisector of ̸ BSC so SW passes through M .


Olympiad geometry document for reference 47

Q.E.D.

Comments: We can see that this is the general problem of Theorem M.T.10.

Message: Now we will switch to a different idea of Generalization by Mosquitall.

Extra M.Ex.3: (Mosquitail) Let △ABC be a scalene triangle with let M, N are midpoints
of minor arcs AB, AC. Let E, F on AB, AC such that EF ||M N . Let EM, F N meet (ABC)
second time at P, Q. Consider two intersection points E ′ , F ′ of (EF P Q) (we call this circle
ψ) with AB, AC different from E, F . Then EF ′ ∩ E ′ F is the incenter I of ABC.

Solution:

Diagram M.Di.45

By Reim so EP QF is cyclic. Let N P cut AC at F ′′ . Since N F ′′ · N P = N A2 = N F · N Q


so P QF F ′′ is cyclic,so that F ′ ≡ F ′′ . Similarly we get EE ′ F ′ F QP is cyclic. By Pascal for
   
M, A, N M, A, N
,
B, P, C B, Q, C

We get E, I, F ′ , E ′ , I, F .

Q.E.D.
Olympiad geometry document for reference 48

Message: We will use the same notation as Extra M.Ex.3.

Extra M.Ex.4: (Dogu) Let the tangent lines from P, Q to ψ intersects at T . Z is the
midpoint of arc BAC of (ABC). Prove that: T, I, Z.

Solution:

Diagram M.Di.46

let J be the center of ψ. We will still use the constructions as Extra M.Ex.3 .We have
̸ JEI = ̸ JEF ′ = 90◦ − ̸ EP F = 90◦ − ̸ M P N = ̸ M AN −90◦ = ̸ BAI =⇒ JI ·JA = Rψ2
so that if we draw tangents from Z to ψ to touch ψ at U, V so ̸ ZU J = ̸ ZV J = 90◦
=⇒ A, Z, V, J, U lie on a circle. By inversion centered at J,radius Rψ we can easily get
U, I, L, F with L midpoint U V . By radical axis theorem so AZ, U V, P Q are concurrent at a
point called W .Since 90◦ = ̸ W AJ = ̸ W LJ = ̸ W HJ so A, L, J, H lie on a circle. Perform
an iversion centered at J,radius Rψ so A becomes I,L becomes Z,H becomes T and also
(ALJ) passes through J so T, I, Z.

Q.E.D.

Comments: The idea of proving is exactly the same to Extra M.Ex.1 .This problem Dogu
found by himself and also inspired by the first generalization by livetolove212.
Olympiad geometry document for reference 49

Message: Here we also introduce the link to the Generalization of Telv Cohl - a very talented
geometer, and we will not present the general problem here. Readers may help themselves
finding out about this generalization:

https://artofproblemsolving.com/community/c284651h1277534 generalization of mixtilinear


incircle

Message: Now let’s move to some calculation problems.

Extra M.Ex.5: (Facebook) Let △ABC be a triangle satisfying AB + AC = 2BC,with


the A−, B−, C−mixtilinear incircles tangent to (ABC)at D, E, F. Prove that: EF = BC
and AE = AF and BE = CF = 2AF .

Solution: (Facebook)

Diagram M.Di.47

Denote a = BC, b = CA, c = AB Let the tangency of the mixtilinear circles be T, R; P, Z; Q, S


and we notice the structure of Theorem M.T.15 here.

3a
Let s = 2
. We have

BP EB AR DA c AF AE (s − a) 1
= = = = ; = = =
PC EC RC DC (s − c) F C EB a 2
Olympiad geometry document for reference 50

Let ̸ AER = α, ̸ CER = β.

(AE.sinα) AR sinα ̸ BAC ̸ BAC


= , = 2; ̸ AEP = ̸ ACB + ; ̸ CEP =
(CE.sinβ) RC sinβ 2 2
(sin(AEP )) (c(b + c))
= = 2.
(sin(CEP )) ac

Thus ̸ AEP = α, ̸ CEP = β and E, R, P are collinear. Similarly F, T, S are collinear. Let
E, R, P intersects F, T, S at X. We know X is the midpoint of arc BDC by homothety. Also
P R//AB, ST //AC, QZ//BC by Theorem M.T.15. So △AZQ ∼ △T BC ∼ △RP C ∼
△ABC. Thus Fd BC = 2̸ ACB+̸ BAC; BCE
d = 2̸ ABC+̸ BAC.Fd
BC+BCE
d = 2̸ BAC+
2̸ ABC + 2̸ ACB = 360◦ .

Thus BE = CF = 2AF = 2AE. Let BE ∩ CF = Y. Y B = Y F and Y E = Y C, △Y EF =


△Y CB. Therefore EF = BC.

Q.E.D.

Extra M.Ex.6: (buratinogigle) Let ABC be an equilateral triangle. B-mixtilinear incircle


touches circumcircle of ABC at M . C-mixtilinear incircle meets side AB at N (near B).
M N meets B-, C-mixtilinear again at S, T , respectively. Prove that:
ST

SN
.

Solution: (khanhnx)

Suppose that C - mixtilinear incircles touches (ABC) at P ; B -, C - mixtilinear incircles touch


AB, CA at V, U , respectively; O be center of (ABC). Then OU ⊥ CO, OV ⊥ OB, U V ∥ BC.
So
̸ OU V = ̸ CU V − ̸ CU O = 120◦ − 60◦ = 60◦
Similarly, we have ̸ OV U = 60◦ .

Hence △OU V is equilateral or

OU = OV = U V = AU = AV

From this, we have AU OV is rhombus, then U V is perpendicular bisector of AO or U V ≡


M P . Denote the radius of (ABC) is R; let Ic be center of C - mixtilinear incircle, Q be
midpoint of AB.
Olympiad geometry document for reference 51

Diagram M.Di.48

We have
OU CO R
Ic O = √ = =
3 3 3
So
R R R
QIc = OQ − Ic O = − =
2 3 6
But
2OU 2R
Ic N = Ic U = √ =
3 3

R 15
then N Q = . Hence
6
√ √ √ √
R 15 R 3 R R( 15 + 3)
N V = N Q + QA − AV = + −√ =
6 2 3 6
and √ √ √ √
R( 15 + 3) R R( 15 + 3 3)
N A = N V + AV = +√ =
6 3 6
But AM = OA = R then
√ √ √
2 2 2 R2 ( 15 + 3 3)2 2 R2 (13 + 3 5)
M N = N A + AM = +R =
36 6
Note that
̸ U AM = ̸ U M A = ̸ V AP = ̸ V P A = 30◦
Olympiad geometry document for reference 52

we have △AU M and △AV P are isosceles at U, V respectively, so

U M = AU = U V = AV = V P

Hence
TM T M · MN MU · MP 6
= 2
= 2
= √
MN MN MN 13 + 3 5

NT 7+3 5
From this, we have = √ . But
MN 13 + 3 5

NS NS · MN NV 2 3+ 5
= = = √
MN MN2 MN2 13 + 3 5

TS 4+2 5
then = √ . Therefore,
MN 13 + 3 5
√ √ √ √ √ √ √
TS T S MN 4 + 2 5 13 + 3 5 4+2 5 8+4 5 (3 + 5)(1 + 5) 1+ 5
= · = √ · √ = √ = √ = √ =
NS MN NS 13 + 3 5 3 + 5 3+ 5 2(3 + 5) 2(3 + 5) 2

Q.E.D.

Extra M.Ex.7: Let △ABC be a triangle right at A. ωa is the A−mixtilinear incircle.


Prove that: The radius of ωa is equal to AB + AC − BC.

Solution:

Diagram M.Di.49
Olympiad geometry document for reference 53

Let (I) be the incircle of △ABC and (I) tangent to AB at D. Let J be the center of ωa .
Let JE, JF ⊥ AB, AC,by Theorem M.T.1 so I is the midpoint of EF . It is easy to see
that JEAF is a square so I is also the midpoint of AJ. So that Rωa = JE = 2ID = 2AD.
AB+AC−BC
But it is well known that AD = 2
. So we are done.

Q.E.D.

Message: For the last problem of the section, we will let readers to prove by your own,the
problem is not hard, we believe you can solve it.

Extra M.Ex.8: Let △ABC be a triangle with the A−mixtilinear incircle ωa . Prove that
the radius of ωa is equal to

[ABC] 1
AB+BC+CA
· .
2
cos2 ( BAC
2
)
([ABC] is denoted to be the area of △ABC)

Similarly , readers can design a formula to calculate the radius of the A−mixtilinear excircle.

Message: After preparing almost every basic knowledge of the mixtilinear circle. Now you
and us will enter the world of Problems and Solution. From now any theorem will be used
and we won’t mention it anymore, instead we will just say ”well-known”.
Olympiad geometry document for reference 54

Problems

Problem 1: (IMO SL 1999) Let △ABC be a triangle inscribed in Ω. P lie on the minor
arc BC of Ω.Incenters of △P AB, △P AC are I1 , I2 . Prove that: (P I1 I2 ) passes through the
tangency of the A−mixtilinear incircle with Ω.

Problem 2: Let △ABC be a triangle inscribed in Ω. Suppose Ω is fixed with fixed chord
BC. A is moving on Ω. Prove that: The nine-point circle of △ABC is tangent to a fixed
circle.

Problem 3: (All-Russian MO 2013) Let ω be the incircle of the triangle ABC and with
centre I. Let Γ be the circumcircle of the triangle BIC. Circles ω and Γ intersect at the
point X and Y . Let Z be the intersection of the common tangents of the circles ω and Γ.
Show that the circumcircle of the triangle XY Z is tangent to the circumcircle of the triangle
ABC.

Problem 4: (Dsoong) Let △ABC be a triangle with the A−mixtilinear incircle tangent to
AB, AC at points P, Q. O is the circumcenter of △ABC and OP intersects IB at E,OQ
intersects IC at F . AK is the diameter of (ABC). JK intersects BC at L. Prove that:
IL, BF, CE are concurrent.

Problem 5: (Modified IMO SL 2016) Let △ABC be a triangle with the A−mixtilinear
circle tangent to (ABC) at X. EF intersects BC at K. R lie on BC such that AR, BF, CE
are concurrent. N is the midpoint of KR.M is the midpoint of segment BC.AM intersects
(ABC) again at Z,ZD intersects (ABC) at U where incircle (I) is tangent to BC at D.Prove
that: N U is tangent to (ABC).

Problem 6: Let △ABC be a triangle with altitudes AD, BE, CF concur at H. (O1 ), (O2 )
are the A−mixtilinear incircles of △AHB, △AHC. Suppose ω is a circle internally tangent
to (O1 ), (O2 ) and passes through A. Prove that: The center of ω lie on AD.

Problem 7: Let △ABC be a triangle with A− mixtilinear incircle tangent to AB at G,


the B−mixtilinear incircle tangent (ABC) at R, and incircle of △ABC tangent AB at F .
Prove that the intersection of CG, RF lies on (ABC)

Problem 8: Let △ABC be a triangle with AB + AC = 2BC. The A−mixtilinear circle


Olympiad geometry document for reference 55

ωa is tangent to (ABC) at X. AX intersects EF at W where ωa is tangent to AB, AC at


E, F . BW, CW intersects (ABC) again at U, V . Prove that: U V, BC, EF are concurrent.

Problem 9: Let △ABC be a scalene triangle with incircle (I) tangent to BC at D. Prove
that: The radical axis of the B−, C−mixtilinear incircles bisects ID and passes through
midpoint of minor arc BC of (ABC).

Problem 10: (USA TST 2016) Let △ABC be a triangle with incircle (I) tangent to BC
at D. The angle bisector of ̸ BAC intersects BC at E and intersects (ABC) at F . (DEF )
intersects the A−excircle at U, V and (ABC) at W . Prove that: AW passes through either
U or V .

Problem 11: (IMO SL 2014) Let △ABC be a triangle with incircle (I). Let the line passing
through I and perpendicular to CI intersects BC,arc BC (not containing A) of (ABC) at
U, V . Let the line passing through U and parallel to AI intersects AV at X,the line passing
through V and parallel to AI intersect AB at Y . M and N be the midpoints of AX and
BC, respectively. Prove that: If I, X, Y so I, W, Z.

Problem 12: (Le Viet An) ⋆ Let ABC be a triangle with its circumcircle (O), incircle (I)
and A−mixtilinear incircle (Oa ). M midpoint of BC, D the touch point of incircle and BC.
Let K be point on (O) such that ̸ AKI = 90◦ . The circumcircle of triangle ADK cuts (I)
again at G. Perpendiculars from O, Oa respectively to DG, GM intersect at X. Prove that
X lies on ID.

Problem 13: Given triangle △ABC with circumcenter O. M, N are midpoints of AB, AC.
Prove that: (BOC) is tangent to the A−mixtilinear incircle,excircle of △AM N .

Problem 14: (Tran Quan) Let △ABC be a triangle right at A. D is the reflection of A
over BC. Prove that: (AB) is tangent to the A−mixtilinear circle of △ACD.

Problem 15: (EGMO 2023) Let ABC be a triangle with circumcircle Ω. Let Sb and Sc
respectively denote the midpoints of the arcs AC and AB that do not contain the third
vertex. Let I be the incenter of ABC. Let ωb be the circle that is tangent to AB and
internally tangent to Ω at Sb , and let ωc be the circle that is tangent to AC and internally
tangent to Ω at Sc . Show that the radical axis of ωb and ωc , meet Ω at the tangency point
of the A−mixtilinear incircle with (ABC).

Problem 16: Let △ABC be a scalene triangle with incircle (I) tangent to AB, AC at
F, E.The A−mixtilinear incircle is tangent to (ABC) at J. JF, JE intersects (ABC) again
Olympiad geometry document for reference 56

at P, Q. A1 , N are midpoints of segment BC and minor arc BC.A′ is the midpoint of segment
A1 N . Prove that: P Q and AA′ are perpendicular.

Problem 17: Let ABCD be a quadrilateral outscribed the circle ω. Suppose AB intersects
CD at E,AD intersects BC at F . Prove that: Their exists a circle Ω that are tangent to
(EAD), (EBC), (F AB), (F CD).

Problem 18: Let △ABC be a triangle with A−mixtilinear circle ωa . The tangent line of
ωa which is parallel to BC and closer to A intersects AB, AC at X, Y ,CX intersects BY
at T . ωa is tangent to AB, AC at E, F and let CE cut BF at W . Prove that: W T passes
through the incenter I of △ABC.

Problem 19: Let △ABC be a scalene triangle,AB < AC with M is the midpoint of
segment BC,circumcenter O, the A−mixtilinear incircle is tangent to (ABC) at X. K is
the reflection of A over incenter I of △ABC. XK intersects the perpendicular bisector of
BC at N .P is the intersection of OI and AX.Prove that: IM//P N .

Problem 20: Let △ABC be a scalene triangle,AB < AC with X is the tangency point of
the A−mixtilinear incircle (has center J) with (ABC). Suppose the A−altitude of △ABC
intersects OI at Z where O, I are the circumcenter,incenter of △ABC. N is the midpoint
of ID where incircle (I) is tangent to BC at D.Prove that: J, N, Z.

Problem 21: (VZH) ⋆ ⋆ Given an acute triangle ABC with circumcenter O and incenter I.
Denote by TA the tangency point of the A-mixtilinear incircle and ⊙(ABC) and define TB
similarly. Let AO meet BC at D, BO meet AC at E. Let X be the intersection of lines AI
and TA D, Y be the intersection of lines BI and TB E. Prove that CI is the angle bisector of
̸ XCY .

Problem 22: (Khuong Nguyen) Let △ABC be a triangle with diameter AS. The A−mixtilinear
circle ωa is tangent to (ABC), AB, AC at X, E, F. XI intersects ωa again at K,AK inter-
sects ωa again at T . EF intersects BC at L and LK intersects ωa again at R.Prove that:
̸ IRT = ̸ JSA with J is the midpoint of AX.

Problem 23: (Nicaragua TST 2019) Let △ABC be a triangle with AC > AB, ω its
inscribed circle and I the center of ω. D and E be the tangency points of ω with CA, AB.BI
and CI meet DE at K, L. P, Q are located on the side BC so that ̸ LQC = 90◦ = ̸ KP B.
Prove that: (P BE), (QCD) intersects on (ABC).

Problem 24: Given △ABC be a triangle with incenter I,the A−mixtilinear incircle ωa is
Olympiad geometry document for reference 57

tangent to (ABC) at X.(I; IX) intersects ωa at Q and (ABC) at T . Prove that: ̸ AT Q =


90◦ .

Problem 25: (DGO 2021) ⋆ ⋆ Let △ABC be a triangle with incenter I and circumcircle
Ω with circumcenter O. The incircle touches CA, AB at E, F respectively. R is another
intersection point of external bisector of ̸ BAC with Ω, and T is Amixtillinear incircle touch
point to Ω. Let W, X, Z be points on Ω. RX intersect AI at Y . Assume that R is not
X. Suppose that E, F, X, Y and W, Z, E, F are concyclic, and AZ, EF, RX are concurrent.
Prove that:

i) AZ, RW, OI are concurrent.

ii) A−symmedian,tangent line to Ω at T and W Z are concurrent.

Problem 26: (buratinogigle) Let △ABC be a triangle with incircle (I) is tangent to AC, AB
at E, F . K, L lie on EF such that BK//AC and CL//AB. CK cut BL at X. Prove that:
IX bisects KL.

Problem 27: (Tran Viet Hung) Let △ABC be a triangle with incircle (I) tangent to
AB, AC at F, E and the A−mixtilinear incircle ωa tangent to AB, AC at P, Q. CF intersects
BE at Ge . ID intersects EF at R. A′ is the reflection of A over I. Prove that: CP, BQ, Ge R
are concurrent at a point X lie on A′ D.

Problem 28: (Mz T) Let △ABC be an acute triangle circumscribed outside the incircle
(I). (I) is tangent to BC, CA, AB at D, E, F .Let the second intersection of AO with (ABC)
be K. Let M be the midpoint of BC, KM intersects (ABC) at N different from K. AI
intersects (ABC) at P different from A, P D intersects EF at T . AT intersects BC at X.
Let Y be the point of symmetry of D through M . Prove that the four points A, N, X, Y lie
on a circle.

Problem 29: (Tran Viet Hung) Let △ABC be a triangle with A−mixtilinear incircle ωa
tangent to (ABC) at X. The line from X perpendicular to XA intersects ωa again at
P .AP intersects (ABC) at Q. O′ is the circumcenter of △XP Q. Prove that: AO′ is the
A−symmedian line of △ABC.

Problem 30: (i3435) In a △ABC, let the A-mixtilinear incircle meet AB at B1 and let
the incircle of △ABC be tangent to BC at D. Prove that: B1 D meets the line through C
perpendicular to the A-angle bisector on the A-median.
Olympiad geometry document for reference 58

Problem 31: (APMC 2016) Let △ABC be a triangle with Ia is the A−excenter. The
schwatt line corresponding to vertice A of △ABC intersects the external bisector of ̸ BAC
at P . P Ia intersects the A−mixtilinear incircle at T nearer to A. Prove that: (BT C) is
tangent to the A−mixtilinear incircle of △ABC.

Problem 32: Let ABC be a triangle with A−excenter J. The A−mixtilinear excircle is
tangent to (ABC) at X. A−excircle (J) is tangent to AB, AC at E, F . Prove that: The
line from M perpendicular to EF and lines XJ,EF are concurrent.

Problem 33: Let △ABC be a triangle with A−mixtilinear incircle ωa tangent to (ABC)
at X. The tangent from X to (ABC) intersects BC at R. D′ is the tangency point of the
A−excircle and BC. RP is tangent to ωa at P different from X. Prove that: P A, P D′ are
isogonal in ̸ BP C.

Problem 34: Let △ABC be a triangle with incircle (I). K is a point lie on (ABC). KI
intersects (ABC) again at L. X lie on KL such that AI bisects ̸ KAX. Prove that: (AXL)
is tangent to the A−mixtilinear incircle of △ABC.

Problem 35: Let △ABC be a triangle with incircle (I) tangent to AB, AC at E, F . EF
intersects BC at T . The A−mixtilinear circle is tangent to (ABC) at X. Prove that: T X
intersects the A−median of △ABC on (ABC).

Problem 36: ⋆ Let △ABC be a triangle with incenter I.O is the center of the circle (ABC).
The B, C−mixtilinear incircles are tangent to (ABC) at Z, T and to AB, AC at X, Y . XY
intersects T Z at S. Prove that: AS ⊥ OI.

Problem 37: Let △ABC be a triangle with A, B, C−excenters are Ia , Ib , Ic . The A, B, C−mixtilinear
incircles are tangent to (ABC) at X, Y, Z. Prove that: Ia X, Ib Y, Ic Z are concurrent.

Problem 38: (OTSS Mock JMO 2020) Let ABC be a triangle with circumcenter O, in-
center I, and circumcircle Γ. The A−mixtilinear incircle is tangent to (ABC) at X. The
perpendicular bisector of BC meets line AX at a point S. Let K be the point on the (AIX),
distinct from I, so KI ∥ BC. KS cut (AIX) again at T . Prove that the tangent at T to
the (T BC) passes through the circumcenter of (T AO).

Problem 39: (i3435) In a triangle ABC, let △DEF be the intouch triangle. Let L be
the midpoint of arc BAC of (ABC) and let EO be the intersection of LD and (ABC).
Prove that: The radical center of (ABC), (EF EO ), and the A-mixtilinear incircle lie on the
A−symmedian of △ABC.
Olympiad geometry document for reference 59

Problem 40: (Iran) Let △ABC be a scalene triangle,AB < AC with incircle (I) tangent
to BC at D.AI cut BC at E. A1 lie on (ABC) such that AA1 //BC. Let DA1 intersects
(DAE) at T . Prove that: IA = IT .

Problem 41: In △ABC, M lie on BC such that AM is tangent to (ABC). Suppose T


is the tangency point of the A-mixtilinear incircle with (ABC); let Ma be the midpoint of
major arc BAC and R as the midpoint of the segment connecting midpoints of major arcs
ABC and ACB. Show that lines M T and Ma R intersect on (ABC).

Problem 42: (HSGS) Let ABC be an acute triangle. (K) is A-mixtilinear incircle and it
touches (ABC),AC, AB at D, E, F . AL is the diameter of (ABC). LC cut KF at M, LB
cut KE at N . Prove that: AD ⊥ M N

Problem 43: (Cosmin Pohoatza) Let ABC and A1 B1 C1 be two triangles having the same
circumcircle O(O, R) and the same incircle I(I, r). Suppose the sidelines BC and B1 C1 are
parallel, prove that A1 is the tangency point of the A-mixtilinear incircle of triangle ABC
with O.

Problem 44: Let △ABC be a scalene triangle with the B−mixtilinear incircle tangents to
side AB at M and C−mixtilinear incircle tangents to side AC at N .Prove that: (AM N ) is
tangent to the A−mixtilinear incircle of △ABC.

Problem 45: (Romania TST 2018) Let ABC be a triangle, let I be its incenter, let Ω be
its circumcircle, and let ω be the A- mixtilinear incircle. Let D, E and T be the intersections
of ω and AB, AC and Ω, respectively, let the line IT cross ω again at P , and let lines P D
and P E cross the line BC at M and N respectively. Prove that: Points D, E, M, N lie on
a circle. Define the center of that circle.

Problem 46: (Peru) Let ABC be a triangle, if BD is the angle bisector of B(D ∈ AC),
the incircle of BDC touch BC at E. Let G be projection of E at BD, and M the midpoint
of BE. The B-excircle of △ADB touch BD at F . Suppose ABM G is cyclic.Prove that: G
is circumcenter of △AM F .

Problem 47: (Jean-Louis Ayme) Let △ABC be a triangle with the B, C−mixtilinear circles
ωb , ωc tangent to (ABC) at B ′ , C ′ . BB ′ , CC ′ intersects ωb , ωc again at L, K. Tangents from
K, L to ωc , ωb intersects at P . Prove that: AP is the A−symmedian line.

Problem 48: Let △ABC be a triangle with the A-mixtilinear circle tangent to AB, AC, (ABC)
at F, E, X. EF intersects (ABC) at Q, P where F lie between Q, E. ID intersects BC at
Olympiad geometry document for reference 60

T . QT intersects (ABC) at R. Prove that: AR ⊥ KP .

Problem 49: (Dogu) Let △ABC be a triangle with A−mixtilinear incircle is tangent to
AB, AC at E, F . Incircle (I) is tangent to AB, AC at X, Y . EF intersects (I) at R. AR
intersects (I) again at K. T is the midpoint of XY and L is the midpoint of AT . Prove
that: KL is tangent to (I).

Problem 50: (IMO 2019) The incircle ω of acute-angled scalene △ABC has centre I and
meets sides BC, CA, and AB at D, E, and F, respectively. The line through D perpendicular
to EF meets ω again at R. Line AR meets ω again at P . (P CE) intersects (P BF ) again
at Q . Prove that: Lines DI and P Q meet on the external bisector of ̸ BAC.

Problem 51: Let △ABC be a triangle with AB + AC = 3BC with circumcenter O. Prove
that: (AO) is tangent to the A−mixtilinear incircle of △ABC.

Problem 52: (Dsoong) Let △ABC be a triangle with A−mixtilinear incircle tangent to
(ABC) at T . I, O are incenter,circumcenter of △ABC. OI intersects AT at X. Tangent
lines from B, C to (ABC) intersects at K. IT intersects BC at L.Prove that: X, L, K.

Problem 53: (tkhalid) Let △ABC be a triangle with incircle ω. Let I be the center of ω
and let D, E, and F be the points of contact of ω with BC, CA, and AB respectively. Let
Ω be the circle passing through B and C tangent to ω at a point X. Furthermore let Ω
intersect AB and AC at P and Q respectively. Let DF intersect BI at Y and CI intersect
DE at Z. Finally let P Y intersect QZ at O. Prove that: B, Y, Z, C lie on a same circle with
center O.

Problem 54: (Dogu) Let △ABC be a triangle with the A−mixtilinear circle tangent to
(ABC) at X. M is the midpoint of major arc BAC of (ABC). J is the A−excenter. M X
intersects (BJC) again at S. Tangents from S, J to (BJC) intersects at T . Prove that:
M, K, T with K is the intersection of the A−symmedian line with (ABC).

Problem 55: (Viet Nam TST 2017) Let △ABC be a triangle with I is its incenter and the
circle (I) is tangent to BC, CA, AB at D, E, F respectively. Denote Ib , Ic as the excenters
of triangle ABC respect to vertices B, C. Let P, Q be the midpoints of segments Ib E, Ic F .
Suppose that (P AC) intersects AB at the second point R and (QAB) intersects AC at the
second point S.

i) Prove that: P R, QS, AI are concurrent.


Olympiad geometry document for reference 61

ii) Suppose that DE, DF intersect Ib Ic at K, J and EJ meets F K at M . The lines


P E, QF intersect (P AC), (QAB) at X, Y (X differs from P and Y differs from Q).
Prove that BY, CX and AM are concurrent.

Problem 56: Let △ABC be a triangle with A−mixtilinear excircle is Ω. F lie on segment
BC. G, H are distinct points lie on Ω. Suppose (AF G), (AF H) are tangent to Ω at G, H.
Prove that: (F GH) passes through the A−excenter Ia of △ABC.

Problem 57: (Pindp) Let △ABC be a triangle with incenter I, circumcircle (O), (K) is
the A mixtilinear incircle, Ia is the A excenter. X is the tangency point of incircle of △ABC
in the side BC. (K) touches (O) at D. DI cuts BC at P . Prove that: AX cuts Ia P at a
point on (K)

Problem 58: Let △ABC be a triangle with the A−mixtilinear circle ω tangent to (ABC)
at X ′ , E, F are the tangency points of ω with AB, AC.Suppose EF intersects BC at S. A
line d passes through S and intersects ω at distinct points X, Y . AX, AY intersects BC at
U, V . Prove that: (X ′ U V ) is tangent to (ABC).

Problem 59: Let △ABC be a scalene triangle with the A−mixtilinear incircle,excircle are
ω, Ω. The incircle (I),A−excircle (J) is tangent to BC at D, D′ . ω, Ω are tangent to (ABC)
at X, Y . Suppose AX cut (I) at P farther to A, AY cut (J) at Q closer to A.Prove that:
P, D, Q, D′ lie on a circle.

Problem 60: Let △ABC be a triangle with T is the tangency point of the A−mixtilinear
circle with (ABC).The line from T perpendicular to BC intersects (ABC) again at D. Prove
that: The nine-point circle of △DBC is tangent to the incircle (I) of △ABC.

Problem 61: (Dsoong) Let △ABC be a triangle with the A−mixtilinear incircle ω tangent
to AB, AC, (ABC) at X, Y, T . Let J be the center of ω and let JX, JY intersects ω again at
V, U . AU, AV intersects ω again at F, E.S is the reflection of A over T . Tangent lines from
A, I to (AIS) intersects at K. L is the center of (AEF ). Prove that: KL passes through
the circumcenter O of △ABC.

Problem 62: In △ABC,X is the tangency point of the A−mixtilinear circle with (ABC).
Prove that: The A−mixtilinear incircles of △ABX, △ACX are tangent.

Problem 63: ⋆ Let △ABC be a triangle with D lie on arc BC of (ABC) that does not
contain A. Prove that: The A−mixtilinear incircle ω of △ABC,the D−mixtilinear incircles
Olympiad geometry document for reference 62

of △DAB, △DAC have a same tangent line.

Problem 64: (Paramizo Dicrominique) Let △ABC be a triangle with incircle (I) tangent
to BC at D. D′ is the reflection of D over the midpoint of BC. O is the center of (ABC)
and AO intersects the line from D′ perpendicular to BC at P .The reflection of line AD over
AI intersects (ABC) at Q. Prove that: AD′ P Q is cyclic.

Problem 65: (Tran Viet Hung) In △ABC, Be is the Bevan point and O is the circumcenter.
Q is the midpoint of arc BC of (ABC) and not containing A. Point L lie on AO such that
Be L//OQ. The A−mixtilinear incircle of △ABC is tangent to (ABC) at T . AT intersects
BC at S. Prove that: IS//LQ.

Problem 66: (buratinogigle) Let △ABC be a triangle with A, B, C−mixtilinear incircles


are ωa , ωb , ωc and incircle (I). Let ψ be the triangle formed by the radical axis of ωa , ωb , ωc
with (I). Prove that: △ABC and ψ are perspective.

Problem 67: (Math-lover 123) Let △ABC be a triangle with orthocenter H,incircle (I)
tangent to BC at D. M is the midpoint of segment AH. M E intersects AI at F . Prove
that: (BHC) and (F ; F E) are tangent.

Problem 68: Let △ABC be a triangle with O is the circumcenter, I is the incenter. Let K
be the A−mixtilinear incircle center. Suppose the reflection I ′ of I over O lie on BC. Prove
that: K lie on BC.

Problem 69: (buratinogigle) ⋆ Let △ABC be a triangle inscribed in circle ω. Common


tangent (other than BC) of mixtilinear incircles at the vertices B and C meet ω at M and
N . Prove that bisector of ̸ M AN goes through centroid of excentral triangle.

Problem 70: (Tran Viet Hung) Let △ABC be a triangle with the A−mixtilinear circle ωa
tangent to (ABC) at X. (I) is the incircle. The lines from X perpendicular to XA, XI
intersect ωa again at K, L. Prove that: The intersection of AK, IL lie on ωa .

Problem 71: Let △ABC be a triangle with the A−mixtilinear incircle,excircle tangent to
(ABC) at X, Y . (I) is the incircle of △ABC and AI cut BC at W . Prove that: (W XY )
and BC are tangent.

Problem 72: Let △ABC be a triangle with A−excircle tangent to AC, AB at E, F . Sup-
pose EF intersects (ABC) at distinct points Y, X such that Y lie between X, F . The
A−bisector of △ABC cut BC at D.(F DY ), (EDX) intersects (ABC) again at U, V . Prove
Olympiad geometry document for reference 63

that: The circumcenter of △DU V lie on AD.

Problem 73: (buratinogigle) Let △ABC be a triangle inscribed in circle (O). (J) is A-
mixtilinear excircle that is tangent to (O) at D . Circle diameter AJ meets line BC at E,
F . Circle (DEF ) meets (J) again at P . Prove that JP ⊥ BC and (DEF ) passes through
A1 which is the intersection of the A−symmedian line with (ABC).

Problem 74: (buratinogigle) Let ABC be a triangle with mixtilinear incircles respectively
are (Oa ), (Ob ), (Oc ). (Oa ) cuts BC at A1 , A2 such that A1 is between B, A2 . (Ob ) cuts CA at
B1 , B2 such that B1 is between C, B2 . (Oc ) cuts AB at C1 , C2 such that C1 is between A, C2 .
Prove that A2 B1 , B2 C1 , C2 A1 intersect base a triangle that is perspective with triangle ABC.

Problem 75: Let △ABC be a triangle with altitudes AD, BE, CF are concurrent at H.
Tangent lines at B, C to (ABC) intersects at T . T B, T C intersects EF at X, Y . M is the
midpoint of BC.

i) Prove that: M H, EF, T D are concurrent at a point called Z.

ii) Prove that: The projection of Z on BC called P lie on T V with V is the second
intersection of (AH), (ABC).

Problem 76: Let △ABC be a triangle with A−mixtilinear incircle tangent to AB, AC, and
(ABC) at E, F, X.(I) is the incircle of △ABC and (A; AE) intersects AO at S closer to O
with O is the center of (ABC). (AIS) intersects (ABC) at R. (A; AE) intersects (ASX)
again at N .Prove that: N, A, R.

Problem 77: (buratinogigle) Let △ABC be a triangle inscribed in circle (O) and A-
mixtilinear excircle (J). Two common external tangent of (O) and (J) touches (O) at
M, N . Prove that M N is tangent to A-mixtilinear incircle of ABC.

Problem 78: Let △ABC be a triangle such that AB + AC = 3BC. The A−mixtilinear
excircle ω is tangent to AB, AC at U, V . J is the A−excenter. (AJV ) intersects ω at S and
(AJU ) intersects ω at T . Prove that: B, S, T, C lie on a circle.

Problem 79: Let △ABC be a triangle with A−mixtilinear circle ω tangent to (ABC) at
X. I is the incenter and IX intersects BC at S.Z lie on AS.Suppose segments ZB, ZC
intersects ω at P, Q. Prove that: BQ, CP intersects on AS.

Problem 80: (PCHP) Let △ABC be a triangle with the B, C−mixtilinear incircles ωb , ωc
Olympiad geometry document for reference 64

have centers O2 , O3 and tangent to (ABC) at Y, Z. Prove that: The intersection of ZO2 , Y O3
lie on the internal bisector of ̸ ABC.

Problem 81: (Luis Gonzales) Let △ABC be a triangle with the A−mixtilinear circle ω
tangent to AB, AC, (ABC) at E, F, X.CW is tangent to ω at W and line CW is different
from line CA. AW intersects CE at R. M is the midpoint of segment AC and XF intersects
EW at L. Prove that: L, R, M .

Problem 82: (Lin yangyuan) Let △ABC be a triangle with A, B, C−mixtilinear incircles
centered at O1 , O2 , O3 and tangent to (ABC) at D, E, F.

i) (O3 ) is tangent to BC at T. Prove that: ̸ T DO3 = ̸ EDO2 .

ii) DO3 cut (O3 ) at P further to D,define Q similarly. Prove that: P Q//O2 O3

iii) BO3 cut CO2 at A′ . Prove that: A′ lie on the internal bisector of ̸ BT C.

Problem 83: (Jean-Louis Ayme) Let △ABC be a triangle with the A−mixtilinear incircle
ω tangent to (ABC) at X. ω is tangent to AB, AC at E, F. I is the incenter of △ABC. Let
the perpendicular line from X to XA intersects ω again at L.LI intersects ω again at R.AX
intersects ω again at S. Prove that: SR, EF, BC are concurrent.

Problem 84: (Lin yangyuan) Let △ABC be a triangle with the A−mixtilinear circle ω
tangent to (ABC) at X. XI intersects BC at S,R lie on BC such that AR//IX. L different
from X lie on ω such that (BLC) is internally tangent to ω. Prove that: M, L, S with M
being the midpoint of AR.

Problem 85: (Dsoong) Let △ABC be a triangle with A−mixtilinear incircle with center J
tangent to (ABC) at X. M is the midpoint of segment BC and (I) is the incircle of △ABC.
(I) is tangent to BC at D. Prove that: The line from D parallel to IM ,the line from J
perpendicular to BC and XM are concurrent.

Problem 86: (Wizard Math & buratinogigle) Let △ABC be a scalene triangle with the
A−mixtilinear circle to be tangent to (ABC), AB, AC at X, P, Q. Incircle (I) is tangent to
AC, AB at E, F. EF, P Q cut BC at S, T. Let d be the perpendicular bisector of BC,let d
cut arc BAC of (ABC) at M . Z is a point lie on d.AZ intersects (ABC) at L. Prove that:
LX, ZT, M S are concurrent.

Problem 87: (Jean-Louis Ayme) Let △ABC be a triangle with the A−mixtilinear incircle
Olympiad geometry document for reference 65

ω tangent to (ABC), AB, AC at X, E, F. The line from X perpendicular to XA intersects


ω again at L. I is the incenter of △ABC,(IXL) intersects AX again at P , ω intersects AX
again at R. Prove that: RI//P K where AI cut BC at K.

Problem 88: (buratinogigle) ⋆ Let ABC be a triangle inscribed in circle (O) with A-excircle
(J). Circle passing through A, B touches (J) at M . Circle passing through A, C touches (J)
at N . BM cuts CN at P . Prove that: AP passes through tangent point of A-mixtilinear
incircle with (O).

Problem 89: Let △ABC be a triangle with distinct U, V lie on (ABC) such that AU = AV.
Let ω be the A−mixtilinear circle,ω is tangent to (ABC) at X, AX intersects ω again at
Y.Prove that: U V is parallel to the tangent from Y to ω.

Problem 90: (纯几何吧) Let △ABC be a triangle with incircle (I) tangent to AB, AC
at F, E and BC at D. ID intersects EF at L. N is the midpoint of arc BAC of (ABC).
AG is the diameter of (ABC).The A−mixtilinear circle ω is tangent to AB, AC at P, Q
and (AP Q) intersects (ABC) again at R. RG cut IN at T.Prove that: T, L are isogonal
conjugate in △ABC.

Problem 91: Let △ABC be a triangle with A, B, C−mixtilinear incircles be ωa , ωb , ωc .


Suppose ωa intersects BC at distinct points Ba , Ca . Tangents from Ba , Ca to ω intersects at
A′ . Define Ab , Ac , Cb , Ca and B ′ , C ′ similarly. Prove that: AA′ , BB ′ , CC ′ are concurrent at
point L′ which is the isogonal conjugate of L wrt. △ABC, where L is the symmedian point
of the excentral triangle of △ABC.

Problem 92: ⋆ ⋆ In △ABC with circumcenter O .Let X be radical center of three mix-
tilinear incircles and Y is the radical center of three mixtilinear excircles .Prove that: O is
the midpoint of XY .

Problem 93: (Stanley Rabinowitz) Let △ABC be a triangle with ̸ ABC = 2̸ ACB. M is
the midpoint of arc AC not containing B of (ABC). The A−mixtilinear incircle is tangent
to (ABC) at T. Prove that: T B + T A = T M.

Problem 94: Let △ABC be a scalene triangle with AB < AC and A−mixtilinear incircle
tangent to (ABC) at T. Let O1 , O2 be the circumcenters of △AT F , △AT E. Prove that:
Circumcenter O of △ABC is the midpoint of O1 O2 .

Problem 95: (Telv Cohl) ⋆ ⋆ Let I, O be the incenter, circumcenter of △ABC, respectively.
Let (wa ), (wb ), (wc ) be the A, B, C−mixtilinear incircles of △ABC and they are tangent to
Olympiad geometry document for reference 66

(ABC) at X, Y, Z. Let P be a point on OI and X1 = XP ∩ (wa ), Y1 = Y P ∩ (wb ), Z1 =


ZP ∩ (wc ). Prove that: AX1 , BY1 , CZ1 are concurrent

Problem 96: (andria) Let △ABC be a triangle with A, B, C−mixtilinear incircles respec-
tively are ωa , ωb , ωc tangent to (ABC) at X, Y, Z. I is the incenter of △ABC and IX cut
BC at D. Define E, F similarly. Let Ge be the Gergonne point and G be the centroid of
△ABC. Prove that: GGe , AD, BE, CF are concurrent.

Problem 97: Given △ABC with the A−mixtilinear incircle tangent to (ABC) at X. (I)
is the incircle of △ABC.The radical axis of (ABC), (I) intersects the polar of T with (I)
at W. (I) is tangent to BC, CA, AB at D, E, F. DD′ is the diameter of (I) and S lie on (I)
such that DS ⊥ EF. Prove that: W, S, D′ .

Problem 98: (Dogu) Let △ABC be a triangle with incircle (I) tangent to BC, CA, AB at
D, E, F . M is the midpoint of arc BC of (ABC) not containing A. M F intersects (ABC)
again at R,M D intersects (ABC) again at K. KB intersects EF at Z. RZ intersects (ABC)
again at W . W D intersects (ABC) again at Q. AP is the diameter of (ABC). Prove that:
Quadrilateral QBP C is harmonic.

Problem 99: (Dogu) Let △ABC be a triangle with A−excircle (J) tangent to AB, AC
at F, E and BC at D. (AEF ) intersects BC at distinct U, V . The A−symmedian line of
△ABC cut (ABC) again at K, (ADJ) intersects (ABC) again at W. Prove that: U KW V
lie on a circle.

Problem 100: ⋆ ⋆ ⋆ (Nguyen Minh Quang) Let ABC be a triangle is incribed (O), incenter
I. △Ia Ib Ic is cevian triangle of I wrt. △ABC. Let ωa , ωb , ωc are the A, B, C-mixtilinear
incricle of △ABC,respectively. The tangents from Ib , Ic (̸= AC, AB) to ωa intersect at Pa .
Similary, we define for Pb , Pc .

i) Prove that APa , BPb , CPc are concurrent.

ii) A∗ , B ∗ , C ∗ are touch points of ωa , ωb , ωc with (O),respectively.


Prove that A∗ Pa , B ∗ Pb , C ∗ Pc , are concurrent on OI.
Olympiad geometry document for reference 67

Solutions

Problem 1: (IMO SL 1999) Let △ABC be a triangle inscribed in Ω. P lie on the minor
arc BC of Ω.Incenters of △P AB, △P AC are I1 , I2 . Prove that: (P I1 I2 ) passes through the
tangency of the A−mixtilinear incircle with Ω.

Solution:

Diagram M.Di.50

Let M, N be the midpoint of minor arcs AC, AB.By angle chasing so P, I1 , I2 , I lie on a
+
circle. Since N, I1 , P , M, I2 , P so by spiral homothety △XI1 N ∼ △XI2 M so that
XN N I1 NA
= =
XM M I2 MA
. Now let X ′ be the tangency point of the A−mixtilinear incircle with (ABC). Let U, V
be the intersection of X ′ N, X ′ M with AB, AC so clearly it is well known that U, V are the
tangencies of the mixtilinear circle with AB, AC .Since N U ·N X ′ = N A2 , M V ·M X ′ = M A2
,also M N//U V so
N A2
X ′N NU N X′ X ′N 2 N A2 X ′N NA XN

= = M A2
⇐⇒ ′ 2
= 2
⇐⇒ ′
= =
XM MV M X′
XM MA XM MA XM
Olympiad geometry document for reference 68

so X ≡ X ′ (X, X ′ lies on the same arc BC).

Q.E.D.

Problem 2: Let △ABC be a triangle inscribed in Ω. Suppose Ω is fixed with fixed chord
BC. A is moving on Ω. Prove that: The nine-point circle of △ABC is tangent to a fixed
circle.

Solution: We will rewrite the problem like this: M is the midpoint of BC,N is the midpoint
of AH where H is the orthocenter of △ABC. U, V lie on the plane such that U V //BC and
U V passes through the center O of Ω.Prove that: (N M ) is tangent to (M ; M U ). Consider
an inversion with center I for Theorem M.T.1 we get the problem.

Problem 3: (All-Russian MO 2013) Let ω be the incircle of the triangle ABC and with
centre I. Let Γ be the circumcircle of the triangle BIC. Circles ω and Γ intersect at the
point X and Y . Let Z be the intersection of the common tangents of the circles ω and Γ.
Show that the circumcircle of the triangle XY Z is tangent to the circumcircle of the triangle
ABC.

Solution:

Diagram M.Di.51

Since Z is the exsimilicenter of (XY Z) and (BIC) so


ZI Rω XI
= =
ZM RΓ XM
Olympiad geometry document for reference 69

so XZ is the external bisector of ̸ IXM .Let Z ′ be the intersection of (XY Z) again with IA
so clearly XZ ′ bisects ̸ IXM by Apollonius circle property. By simple angle chasing we can
prove XM is the external bisector of ̸ O′ XI from there
R(XY Z) XO′ M O′
= =
Rω XI MI
so that M is the exsimilicenter of (XY Z) and ω. Let the tangent lines from M to ω intersects
(ABC) at S, T respectively so by Poncelet porism, ST is tangent to ω. U, V are the tangency
points of M S, M T with (XY Z). Since (M I, Z ′ Z) = −1 =⇒ U, I, V . Using the Theorem
M.T.1 so we are done.

Q.E.D.

Problem 4: (Dsoong) Let △ABC be a triangle with the A−mixtilinear incircle with center
J tangent to AB, AC at points P, Q. O is the circumcenter of △ABC and OP intersects
IB at E,OQ intersects IC at F . AK is the diameter of (ABC). JK intersects BC at L.
Prove that: IL, BF, CE are concurrent.

Solution:

Diagram M.Di.52

Let JK cut (O) at S. A−mixtilinear incircle of △ABC is tangent to (ABC) at T . We can


easily see △SP B ∼ △SQC since S lie on (AP JQ).So
SB BP TB
= =
SC CQ TC
Olympiad geometry document for reference 70

(Well known) so SBT C is harmonic. Let D be the intersection of T K and BC. To prove the
problem, we will need to prove D, E, F (Since (DL, BC) = K(DL, BC) = (T S, BC) = −1).
Let M, N, M ′ , N ′ be the midpoint of minor arcs AC, AB and large arcs BAC, ACB. We
apply Pascal for
A, T, M ′
 

M, C, K
=⇒ OQ, CM ′ , T K are concurrent at a point called X. (Since T, Q, M ). Similarly we will
get OP, BN ′ , T K are concurrent at a point called Y . By Pascal theorem for
C, N ′ , M
 

B, M ′ , N
=⇒ BN ′ , CM ′ , OI are concurrent.By Dersagues theorem for △OXY , △ICB with OX ∩
IC = E, OY ∩ IB = F, XY ∩ BC = D so E, F, D. We are done.

Q.E.D.

Problem 5: (Modified IMO SL 2016) Let △ABC be a triangle with the A−mixtilinear
circle tangent to (ABC) at X. EF intersects BC at K. R lie on BC such that AR, BF, CE
are concurrent. N is the midpoint of KR.M is the midpoint of segment BC.AM intersects
(ABC) again at Z,ZD intersects (ABC) at U where incircle (I) is tangent to BC at D.Prove
that: N U is tangent to (ABC).

Solution:

Diagram M.Di.53
Olympiad geometry document for reference 71

Since (KR, BC) = −1, so it is well known that T R bisects ̸ BT C. Let (AEF ) intersects
(ABC) at S. It is well known that SBT C is a harmonic quadrilateral. Since N is the
midpoint of KR so N T is tangent to (ABC) so N S is tangent to (ABC).

We will prove S, D, Z which will end the problem. So we will need


SB DB ZC DB M C AB DB AB
= = =
SC DC ZB DC M B AC DC AC
. Let L lie on (ABC) other than T such that T L//BC, let AL intersects BC at D′ so it is
well known that D, D′ are reflections over midpoint M of BC. Since it is well known that
quadrilateral SBT C is harmonic so
SB TB LC D′ C AB DB AB
= = = ′ =
SC TC LB D B AC DC AC
And we are done.

Q.E.D.

Problem 6: Let △ABC be a triangle with altitudes AD, BE, CF concur at H. (O1 ), (O2 )
are the A−mixtilinear incircles of △AHB, △AHC. Suppose ω is a circle internally tangent
to (O1 ), (O2 ) and passes through A. Prove that: The center of ω lie on AD.

Solution:
Olympiad geometry document for reference 72

Diagram M.Di.54


Perform an inversion with centered at A, radius AH · AD. (O1 ), (O2 ) becomes (Ib ), (Ic )
or the A−excircles of triangles △AHB, △AHC. ω becomes a line ψ tangent to (Ib ), (Ic )
and now we will need to prove ψ//BC. T is the intersection of Ic′ Ib with AD. ̸ AT Ic′ =
̸ CHA ̸ BAH
̸ AHIc′ + ̸ Ib AH = ̸ AHIc + ̸ Ib AH = 90◦ + 2
+ 2
= 135◦ . So that ̸ AT Ib =
symmetric
̸ AT Ic′ = 135◦ = 135◦ . Let T U, T V be tangent to (Ib ), (Ic ) so by angle chasing
̸ U T V = 2̸ Ic T Ib = 180◦ so T, U, V so U V //BC. Hence proven.

Q.E.D.

Problem 7: Let △ABC be a triangle with A− mixtilinear incircle tangent to AB at G,


the B−mixtilinear incircle tangent (ABC) at R, and incircle of △ABC tangent AB at F .
Prove that the intersection of CG, RF lies on (ABC)

Solution:

Diagram M.Di.55

Let IE perpendicular to AC and S lie on (ABC) such that BS//AC. IG cut AC at W .CI
cut EF at K, CG cut EF, (ABC) at T, X. We need to prove X, F, R
Lemma: T, B, S (which is also a lemma appeared in Sharygin).
Olympiad geometry document for reference 73

Proof: It is well known that ̸ BKC = 90 but CI is the bisector of ̸ ACB =⇒ K is


the midpoint of BL. Since ET //GW because they both perpendicular to AI =⇒ K is
the midpoint of T E since it is well known that I is the midpoint of GW . =⇒ BT //LE
≡ AC =⇒ T, B, S.

Back to the problem. It is well known that R, E, S. Let F ′ be the intersection of XR and
AB. Using Pascal theorem for  
R, C, B
A, S, X
=⇒ T, F ′ , E =⇒ F ′ ≡ F.

Q.E.D

Problem 8: Let △ABC be a triangle with AB + AC = 2BC. The A−mixtilinear circle


ωa is tangent to (ABC) at X. AX intersects EF at W where ωa is tangent to AB, AC at
E, F . BW, CW intersects (ABC) again at U, V . Prove that: U V, BC, EF are concurrent.

Solution:

Diagram M.Di.56

In the proof of Extra M.Ex.5 we known that B, V ′ are reflection over EF where V ′ is the
tangency of the C−mixtilinear incircle with (ABC). It is well known that the reflection of
B over EF lie on CW (See Theorem M.T.12) so that V ≡ V ′ .Similarly for U , so U is the
Olympiad geometry document for reference 74

tangency point of the B−mixtilinear circle with (ABC). It is well known that U V, EF, BC
are concurrent (See Theorem M.T.10).

Q.E.D.

Problem 9: Let △ABC be a scalene triangle with incircle (I) tangent to BC at D. Prove
that: The radical axis of the B−, C−mixtilinear incircles bisects ID and passes through
midpoint of minor arc BC of (ABC).

Solution:

Diagram M.Di.57

Let the C−mixtilinear incircle tangent to (ABC), BC, CA at Z, V, R. The B−mixtilinear


circle is tangent to (ABC), BC at Y, U . Incircle (I) is tangent to AC at E. Since the radical
axis of (I) and the C−mixtilinear is the midline of the isosceles trapezoid ERV D so their
radical axis passes through N which is the midpoint of ID.Similarly the radical axis of (I)
and the B−mixtilinear circle passes through N . By radical axis theorem, and since the two
others radical axis passes through N so the radical axis of B, C−mixtilinear circles passes
through N .
Now we are left to prove M lie on the radical axis of B, C−mixtilinear circles. By homothety
so ZW, U Y passes through midpoint M of minor arc BC. Therefore M V · M Z = M B 2 =
M U · M Y so M lie on the radical axis of B, C−mixtilinear circles.
Olympiad geometry document for reference 75

Q.E.D.

Problem 10: (USA TST 2016) Let △ABC be a triangle with incircle (I) tangent to BC
at D. The angle bisector of ̸ BAC intersects BC at E and intersects (ABC) at F . (DEF )
intersects the A−excircle at U, V and (ABC) at W . Prove that: AW passes through either
U or V .

Solution:

Diagram M.Di.58

We will focus on point U . Let W D cut (ABC) again at S, it is well known that AS//BC,
by Reim so DEF W is cyclic. Let JD′ ⊥ BC at D′ so it is well known that BD = D′ C.
Let ID cut (DEF ) again at R. Let U ′ be the reflection of D′ over JE,so that ̸ F U ′ E =
symmetric
̸ F D′ E = ̸ F DD′ = ̸ F DE =⇒ U ′ lie on (F DE), also since U ′ lie on (J; JD′ ) or
the A−excircle so U ≡ U ′ . From there we get lines AD′ , AU are reflection over AI. It is
well known that lines AW and AD′ are also reflections over AI so A, U, W .

Q.E.D.

Problem 11: (IMO SL 2014) Let △ABC be a triangle with incircle (I). Let the line
passing through I and perpendicular to CI intersects BC,arc BC (not containing A) of
Olympiad geometry document for reference 76

(ABC) at U, V . Let the line passing through U and parallel to AI intersects AV at X,the
line passing through V and parallel to AI intersect AB at Y . M and N be the midpoints
of AX and BC, respectively. Prove that: If I, X, Y so I, W, Z.

Solution:

Diagram M.Di.59

Let U X cut AB at T . Considering trapezoid AIV Y with diagonals AV, IY intersects at X


and T U//AI//Y V so it is well known, or we can just use Thales, that X is the midpoint
of U T . By angle chasing, ̸ BIV = ̸ BIU = ̸ BAI = ̸ BY V = ̸ BT U (By parallel). From
there we get T IU B, Y IV B are cyclic. Also I is the midpoint of arc T U of (BT U ) and we
proved that X is the midpoint of U T so ̸ IXU = 90◦ =⇒ ̸ AIX = ̸ U XI = 90◦ =⇒ Y
is the tangency point of the A−mixtilinear incircle with AB. We have proved that IY BV
is cyclic so it is well known that V is the tangency point of the A−mixtilinear circle with
(ABC). Now using Theorem M.T.9 and we are done.

Q.E.D.

Comments: This problem is not hard as a G7,but instead it is very easy for solver to fake
solve or be a blunder on it.

Problem 12: (Le Viet An) ⋆ Let ABC be a triangle with its circumcircle (O), incircle (I)
Olympiad geometry document for reference 77

and A−mixtilinear incircle (Oa ). M midpoint of BC, D the touch point of incircle and BC.
Let K be point on (O) such that ̸ AKI = 90◦ . The circumcircle of triangle ADK cuts (I)
again at G. Perpendiculars from O, Oa respectively to DG, GM intersect at X. Prove that
X lies on ID.

Solution:

Diagram M.Di.60

AI cut BC at P , S, N midpoints of arcs BC, BAC. Since K, D, S =⇒ AKGDP is cyclic.


=⇒ ̸ AGD = ̸ AKD = ̸ ABS L lie on (I) such that DL//AI. From angle chasing we
get ̸ LGD = ̸ AKD = ̸ AGD =⇒ A, L, G. Let (Oa ) tangent at (O) at T . It is well
known that A, L, T ; N, I, T and LIDT is cyclic (See the proof of Example M.E.5) =⇒
T G = T D (Angle chasing) =⇒ IN ⊥ GD.
Redefining X be the intersection of O//IN and ID. =⇒ OX ⊥ GD and IX = ON . It is
suffice that Oa X ⊥ GM . Since IT //OX, IX = OT =⇒ IOXT is cyclic and isosceles. J
lie on IX such that Oa J//IN//OX To show △GDM ∼ △Oa JX , since ̸ GDM = ̸ Oa JX
we will need to show
DG JOa Oa J
= =
DM JX Oa O
Olympiad geometry document for reference 78

.(By cyclic isosceles) H is the midpoint of DG. =⇒ N, I, H,̸ IHD = 90. IY ⊥ M N =⇒


△IY N ∼ △DHI, IY = DM .
DG 2DH 2ID
=⇒ = =
DM IY IN
By Menelaus theorem;
Oa O IT SN Oa O IN
= 1 =⇒ =
Oa T IN SO Oa T 2IT
. We need to prove that
2ID Oa J Oa J Oa T Oa J 2IT Oa J ID
= = = ⇐⇒ =
IN Oa O Oa T Oa O Oa T IN Oa T IT
But
Oa J JOa
=
Oa T JI
so it is true since △Oa JI ∼ △GIT (by angle chasing) Hence proven.

Q.E.D.

Problem 13: Given triangle △ABC with circumcenter O. M, N are midpoints of AB, AC.
Prove that: (BOC) is tangent to the A−mixtilinear incircle,excircle of △AM N .

Solution:

Diagram M.Di.61
Olympiad geometry document for reference 79

q
AB·AC
Perform an inversion with center A,radius 2
and a reflection over the line AI where
I is the incenter. Then the A−mixtilinear incircle,excircle become the excircle,incircle of
△ABC, (BOC) becomes the nine-point circle of △ABC. By Feuerbach theorem so the
nine-point circle are tangent with the incircle and the excircle of △ABC.

Q.E.D.

Comments: We can also change the statement to △ABC,with altitudes BE, CF cut at
H. Then replace (AM N ) by (AH).

Message: Now we will come to a simple problem by Tran Quan, who has many nice and
brilliant problems about tangent circles.

Problem 14: (Tran Quan) Let △ABC be a triangle right at A. D is the reflection of A
over BC. Prove that: (AB) is tangent to the A−mixtilinear circle of △ACD.

Solution:

Diagram M.Di.62

Let K be the center of the mixtilinear circle mentioned. Let the A−mixtilinear incircle of
△ADC tangent to AD at Y and (ABC) at X. Since B is the midpoint of arc AD of (ABC)
not containing C so by homothety B, Y, X.
Perform an inversion centered at with radius BA, since BY · BX = BA2 by midpoint of arc
property so the A−mixtilinear incircle of △ADC becomes itself. (AB) clearly becomes line
AC. Since AC and the A−mixtilinear circle of △ADC are tangent so we are done.
Olympiad geometry document for reference 80

Q.E.D.

Comments: Readers can try the general problem of this problem below.
Generalization: (buratinogigle) Let ABC be a triangle inscribed in circle (O). Let D and
F be the points lying on the segments BC and AB, respectively, such that F D ⊥ OB (i.e.
four points A, F , D, C are concyclic). Ray F D meets (O) at P . Let ω be the circle which is
tangent to the segments F P , F C and is tangent to (O). Prove that circumcircle of triangle
ABD is tangent to ω.

Problem 15: (EGMO 2023) Let ABC be a triangle with circumcircle Ω. Let Sb and Sc
respectively denote the midpoints of the arcs AC and AB that do not contain the third
vertex. Let I be the incenter of ABC. Let ωb be the circle that is tangent to AB and
internally tangent to Ω at Sb , and let ωc be the circle that is tangent to AC and internally
tangent to Ω at Sc . Show that the radical axis of ωb and ωc , meet Ω at the tangency point
of the A−mixtilinear incircle with (ABC).

Solution:

Diagram M.Di.63
Olympiad geometry document for reference 81

Let K be the tangency point of the A−mixtilinear incircle with (ABC). Let the tangent
lines from Sc , Sb to intersect at T . Clearly T lie on the radical axis δ of ωb , ωc . Let ωb , ωc to be
tangent to AB, AC at X, Y . By homothety so Sb , Sc , X, Y .It is well known that quadrilateral
ASc KSb is harmonic so K, A, T . It is also well known that Sb Sc ⊥ AI so XY ⊥ AI so △AXY
is isosceles hence AX 2 = AY 2 so we can easily see A lie on δ, we also proved that T lie on δ
so line AT ≡ δ ,we also proved that AK passes through T and we are finished.

Q.E.D.

Problem 16: Let △ABC be a scalene triangle with incircle (I) tangent to AB, AC at
F, E.The A−mixtilinear incircle is tangent to (ABC) at J. JF, JE intersects (ABC) again
at P, Q. A1 , N are midpoints of segment BC and minor arc BC.A′ is the midpoint of segment
A1 N . Prove that: P Q and AA′ are perpendicular.

Solution:

Let ID cut EF at R where (I) is tangent to BC at D so it is well known that A, R, Z. M


is the midpoint of IR so by homothety A, M, A′ . We will prove AM ⊥ P Q.
We will prove a beautiful Lemma below.

Lemma: Let ABC be a triangle with incircle (I) tangent to sides at D, E, F . ID intersects
EF at L. K reflects I over A. Prove that: I, L are isogonal conjugate pair wrt. △KBC
and IL, LK isogonal in ̸ BLC.

Solution:

Diagram M.Di.64
Olympiad geometry document for reference 82

CL, BL cut AC, AB at T, R. It is well known that RT //BC and RT tangent to (I). From
I ⊥ IC cut BC at S. It is well known that △BSI ∼ △BIA, since K reflects I over A,
and well known that K reflects I over A =⇒ △BIK ∼ △BST =⇒ ̸ KBL = ̸ IBS;
similarly ̸ KCL = ̸ ICS and we get (I, L). Now we will prove the isogonal part, ̸ CLx =
̸ ABC
̸ KCL + ̸ LKC = ̸ ICD + ̸ IKB = ̸ KIC − ̸ KBC = 90 + 2
− 2̸ IBC − ̸ IBL =
90 − ̸ LBD = ̸ BLD. Hence proven.
Back to the problem;

Diagram M.Di.65

Lemma: C, R, P ; B, R, Q
Solution: Let CP cut BQ at R′ . CP, BQ cut AB, AC at G, H. To prove R′ ≡ R we will
prove GH//BC since it is well known that AR bisects BC. By pascal for
 
P, A, Q
B, J, C

=⇒ F, R′ , E. Since
GA CA QA QA F A JB
= ; =
GB CB QB QB F B JA
GA CA F A JB
=⇒ =
GB CB F B JA
Olympiad geometry document for reference 83

Similarly
HA BA EA JC
=
HC BC EC JA
We need to prove
CA F A JB BA EA JC
=
CB F B JA BC EC JA
CA BA JB EC AC
⇐⇒ JB = JC ⇐⇒ α = =1
FB EC JC F B AB
AJ cut BC at S.
JB SB AC
=
JC SC AB
D′ lie on BC such that BD = D′ C. It is well known AJ, AD′ isogonal in ̸ BAC. By Steiner
ratio lemma;
SB D′ C AB 2 DB AB 2
= ′ =
SC D B AC 2 DC AC 2
.
JB AB DB
=⇒ =
JC AC DC
So that
JB F C AC DB EC
α= = =1
JC F B AB DC F B
. Done.

Back to the problem, Reflects I over A to get U . For AN ⊥ P Q ⇐⇒ RU ⊥ P Q <=>


̸ BRD = 90 − ̸ RBC = 90 − ̸ RQP = ̸ QRU = ̸ CRx . So to prove AN ⊥ P Q we need
RU, RD isogonal in ̸ BRC which is true by the first lemma. Hence proven.

Q.E.D.

Problem 17: Let ABCD be a quadrilateral outscribed the circle ω. Suppose AB intersects
CD at E,AD intersects BC at F . Prove that: Their exists a circle Ω that are tangent to
(EAD), (EBC), (F AB), (F CD).

Solution:
Olympiad geometry document for reference 84

Diagram M.Di.66

It is well known that (EAD), (EBC), (F AB), (F CD) are concurrent at the Miquel point
M of complete quadrialteral ABCD.EF . Let the tangent lines from M to ω be d, d′ . Let
I be the center of ω. Let d, d′ intersects (EBC), (EAD) at X, Y and P, Q. The line from
I perpendicular to IM intersects M X, M Y at W, K.By Poncelet porism so XY is tangent
to ω.Let Ω be the circle passes tangent to M X, M Y at W, K, clearly it is also tangent to
(M XY ) or (EBC) by property of mixtilinear circle. Again,by Poncelet porism we get P Q
is tangent to ω, notice that Ω is now the M −mixtilinear excircle of △M P Q so (M P Q) or
(EAD) and Ω are tangent. Similarly we let d, d′ intersects (F DC) and (F AB) and we will
get Ω is tangent to (F DC), (F AB).

Q.E.D.

Problem 18: Let △ABC be a triangle with A−mixtilinear circle ωa . The tangent line of
ωa which is parallel to BC and closer to A intersects AB, AC at X, Y ,CX intersects BY
at T . ωa is tangent to AB, AC at E, F and let CE cut BF at W . Prove that: W T passes
through the incenter I of △ABC.

Solution:
Olympiad geometry document for reference 85

Diagram M.Di.67


J is the center of ωa .Clearly ωa is the A−excircle of △AXY . Since ̸ JY C = 2
XY C =
◦ 1̸
90 − 2
ACB =⇒ JY ⊥ IC. It is easy to see that JY ⊥ KF =⇒ KF//V C. Let
CI intersects XF at V . EF intersects XY at Z and ωa is tangent to XY at K. We have
F (KI, V C) = (KZ, XY ) = −1 (Well known) =⇒ I is the midpoint of V C. Since it is well
known that I is also the midpoint of EF so V F//EC or EC//XF , similarly F C//EY so
by Pappus theorem ,if we let XF cut EY at G so W, T, G but T EW F is a parallelogram
=⇒ G, W, I, T .

Q.E.D.

Problem 19: Let △ABC be a scalene triangle,AB < AC with M is the midpoint of
segment BC,circumcenter O, the A−mixtilinear incircle is tangent to (ABC) at X. K is
the reflection of A over incenter I of △ABC. XK intersects the perpendicular bisector of
BC at N .P is the intersection of OI and AX.Prove that: IM//P N .

Solution:

Let U, V be the midpoints of arcs BAC, BC of (ABC) so it is well known that X, I, U .


L is the midpoint of AX and LI cut U V at W . It is easy to see that IL//XN . Since
△IAX ∼ △IU V so it is easy to see that △IP X ∼ △IW V
Olympiad geometry document for reference 86

Diagram M.Di.68

.We have ̸ XN V = ̸ IW N = ̸ IP X =⇒ ON XP is cyclic. X ′ lie on (ABC) other


than X such that XX ′ //BC. It is well known that AX ′ passes through D′ which is the
tangency point of the A−excircle with BC. It is also well known that IM//AD′ so ̸ OM I =
90◦ − ̸ IM B = 90◦ − ̸ AD′ B = 90◦ − ̸ AX ′ X = 90◦ − ̸ ACX = ̸ OXA = ̸ OXP = ̸ ON P
=⇒ IM//P N .

Q.E.D.

Comments: We can see this problem has the similar structure with Example M.E.19
and Example M.E.20.

Problem 20: Let △ABC be a scalene triangle,AB < AC with X is the tangency point of
the A−mixtilinear incircle (has center J) with (ABC). Suppose the A−altitude of △ABC
intersects OI at Z where O, I are the circumcenter,incenter of △ABC. N is the midpoint
of ID where incircle (I) is tangent to BC at D.Prove that: J, N, Z.

Solution:
Olympiad geometry document for reference 87

Diagram M.Di.69

Let U, S be the midpoints of arc BAC, BC of (ABC). It is well known that X, I, U . To


prove J, N, Z we will need
JI IN ID
= =
JA AZ 2AZ
. By Thales we have
IA
AZ = SO
IS
so we will need to show
JI ID ID IS
= IA
= ·
JA 2SO IS IA U S
⇐⇒ JI · IA · U S = ID · IS · JA ⇐⇒ JI · IA · U S = ID · BS · JA

. By Thales we can prove


AJ
JX = ID
AI
so now we will prove
JI SB
JI · U S = JX · BS ⇐⇒ =
JX SU
. We have
JI JI
=
JX JE
with JE ⊥ AB. By angle chasing, we can easily prove that △JIE ∼ △SBU so
JI SB
=
JE SU
Olympiad geometry document for reference 88

Hence proven.

Q.E.D.

Problem 21: (VZH) ⋆ ⋆ Given an acute triangle ABC with circumcenter O and incenter
I. Denote by TA the tangency point of the A-mixtilinear incircle and ⊙(ABC) and define
TB similarly. Let AO meet BC at D, BO meet AC at E. Let X be the intersection of lines
AI and TA D, Y be the intersection of lines BI and TB E. Prove that CI is the angle bisector
of ̸ XCY .

Solution: (Luis Gonzales)

Diagram M.Di.70

Let the A−mixtilinear incircle touch AB, AC at AC , AB . It’s well-known that I is mid-
point of AB AC and I, TA , B, AC are concyclic. It is well known that CI is tangent to
(IBTA ) and CI is tangent to (IATB ) =⇒ ATB , BTA , CI concur at the radical center J of
(ABC), (IBTA ), (IATB ).

It’s well-known that ATA , BTB , OI are concurrent at T (See Theorem M.T.13).Let S be
the intersection of OI with AB so

(IO, T S) = A(IO, T S) = A(XD, TA B) = B(XD, TA A)

and
(IO, T S) = B(IO, T S) = B(Y E, TB A) = A(Y E, TB B)
Olympiad geometry document for reference 89

=⇒ A(Y E, TB B) = B(XD, TA A)
=⇒ J, C and P are collinear where P is the intersection of AY and BX , also P lie on CI.
By Desargues involution theorem for the complete P XIY and the point C, it follows that
the involution CA 7→ CB, CP 7→ CI, CX 7→ CY is therefore reflection accross CI =⇒ CI
bisects ̸ XCY.

Q.E.D.

Comments: This problem is very beautiful,we are so interested in it when we first saw this
problem posted.Also, you guys have just enjoyed a very beautiful solution from one of AoPs
greatest geometer - Luis Gonzales.

Problem 22: (Khuong Nguyen) Let △ABC be a triangle with diameter AS. The A−mixtilinear
circle ωa is tangent to (ABC), AB, AC at X, E, F. XI intersects ωa again at K,AK inter-
sects ωa again at T . EF intersects BC at L and LK intersects ωa again at R.Prove that:
̸ IRT = ̸ JSA with J is the midpoint of AX.

Solution:

Diagram M.Di.71

It is well known that lines AK, AX are reflections over AI so IX = IT so XT //EF =⇒


̸ T RK = ̸ T XI = ̸ XIL = ̸ T IF =⇒ LRIT is cyclic.It is well known that IEBX is
Olympiad geometry document for reference 90

cyclic so ̸ LIT = 180◦ − ̸ F IT = 180◦ − ̸ EIX = ̸ ABX = 180◦ − ̸ AOJ = ̸ JOS. It is well
known that ̸ LXI = 90◦ ,also by the angle chasing done in front we have ̸ AOJ = ̸ LIX
=⇒ △LIX ∼ △AOJ.So that we have
IL IL OA OS
= = =
IT IX OJ OJ
=⇒ △JOS ∼ △LIT (Since we have proved ̸ JOS = ̸ LIT ) =⇒ ̸ JSA = ̸ IRT .

Q.E.D.

Problem 23: (Nicaragua TST 2019) Let △ABC be a triangle with AC > AB, ω its
inscribed circle and I the center of ω. D and E be the tangency points of ω with CA, AB.BI
and CI meet DE at K, L. P, Q are located on the side BC so that ̸ LQC = 90◦ = ̸ KP B.
Prove that: (P BE), (QCD) intersects on (ABC).

Solution:

Diagram M.Di.72

It is well known that IX is the symmedian line of △IBC and also it is well known that K, L
lie on (BC) so △ILK ∼ △IBC,there fore IX bisects KL at N . Let X be the tangency
of the A−mixtilinear incircle with (ABC). M is the midpoint of BC so ̸ IN L = ̸ IM B =
180◦ − ̸ ABX (Well known,See the proof of Theorem M.T.7) so that EN XB is cyclic.
It is well known that M K//AB (M K is the middle line) so ̸ M P N = ̸ M KN = ̸ AED
=⇒ BEN P is cyclic =⇒ (BEP ) passes through X,similarly for (QCD) so we are done.
Olympiad geometry document for reference 91

Q.E.D.

Problem 24: Given △ABC be a triangle with incenter I,the A−mixtilinear incircle ωa is
tangent to (ABC) at X.(I; IX) intersects ωa at Q and (ABC) at T . Prove that: ̸ AT Q =

90 .

Solution:

Diagram M.Di.73

Construct diameter AP of (ABC) so ̸ QT X = ̸ XIS = 90◦ − ̸ XSI = 90◦ − ̸ XP O =



2
XOP = ̸ XT P =⇒ T, Q, P .Hence ̸ AT Q = 90◦ .

Q.E.D.

Problem 25: (DGO 2021) ⋆ ⋆ Let △ABC be a triangle with incenter I and circumcircle
Ω with circumcenter O. The incircle touches CA, AB at E, F respectively. R is another
intersection point of external bisector of ̸ BAC with Ω, and T is Amixtillinear incircle touch
point to Ω. Let W, X, Z be points on Ω. RX intersect AI at Y . Assume that R is not
X. Suppose that E, F, X, Y and W, Z, E, F are concyclic, and AZ, EF, RX are concurrent.
Prove that:

i) AZ, RW, OI are concurrent.

ii) A−symmedian,tangent line to Ω at T and W Z are concurrent.


Olympiad geometry document for reference 92

Solution: (k12byda5h)

Diagram M.Di.74

Simplify point X.

Let AI intersect Ω again at N . Let ω1 be the circle (EF Y X). Since it passes through E
and F , their center lies on AI, and the tangent line at Y to ω1 is parallel to AR. By Reim
Theorem, N also lies on ω1 . Let (AEF ) intersect Ω again at L. By radical axis, AL, EF, XN
intersect at a point called S.

Simplify point Z.

By Pascal on hexagon ZALRXN , the lines RL, EF, and ZN are concurrent at a point called
U . Let D be the tangent point of the incircle to the line BC. By the well-known lemma
that L, N, D are collienar, it is also known that EF, BC, LR are concurrent at the harmonic
conjugate of D wrt BC. Thus, (BC, DU ) = −1. Since DN ⊥ U R (at L) and U D ⊥ RN ,
D is the orthocenter of △U RN . Therefore, R, D, Z are collinear.

Claim AZ, LI, EF are concurrent at a point called P .

By projecting (BC, U D) = −1 through R and A, we have (BC, LZ) = −1 and A(F E, LD) =
−1. Note that (AI, EF ) = −1, we get the desired concurrency.
Olympiad geometry document for reference 93

Simplify point W

By radical axis, Z, W, S are collinear. By pascal on hexagon W ZN ALR, AN, RW and U S


are concurrent at a point called V which is the midpoint of EF .

Let Q be the intersection of AZ and RW Let A′ be the antipode of A wrt O. Since AR ∥ N A′ ,

AQ AR A′ N A′ I
= = =
QP PV PV IP
. Thus,
AQ P I A′ O
· · = 1.
QP IA′ OA
By Menelaus’s theorem, Q, I, O are collinear as desired (problem i).

Let A−symmedian intersect Ω again at K and AN intersect BC at M . Since (BC, AK) =


(CB, N R) = −1, K, M, R are collinear. Let RZ intersect AI at Z ′ . For problem ii), we need
to show that (A, K), (W, Z), (T, T ) are reciprocal pairs of an involution on Ω. By projecting
it with R, we need to show that (A, M ), (V, Z ′ ), (I, I) are reciprocal pairs.

Claim: V, Z ′ , L, D are concyclic.

Let V ′ be the midpoint of BC. Spiral similarity at L sends △AEF ∪ V → △RCB ∪ V ′ .


Hence, ̸ AV L = ̸ RV ′ L = ̸ RDL (since it is obvious that V ′ , R, D, L are concyclic.)

By the claim, RV · RZ ′ = RL · RD = RI 2 = RA · RM which directly implies the involution.

Q.E.D.

Comments: This problem is very hard. During the test (DGO 2021), no one can solve fully
this problem.

Problem 26: (buratinogigle) Let △ABC be a triangle with incircle (I) is tangent to
AC, AB at E, F . K, L lie on EF such that BK//AC and CL//AB. CK cut BL at X.
Prove that: IX bisects KL.

Solution:
Olympiad geometry document for reference 94

Diagram M.Di.75

Let the A−mixtilinear circle touch AB, AC at P, Q so clearly I is the midpoint of P Q. We


claim that K, P, C. Let CI intersects EF at U ,BU cut AC at V , it is well known that
̸ BU C = 90◦ also CI bisects ̸ ACB so U is the midpoint of BV .Since BK//V E so U is
the midpoint of KE,but I is also the midpoint of P Q so it is easy to see K, C, P .Similarly
B, Q, L. But it is easy to see that P Q//KL so by homothety XI bisects KL.

Q.E.D.

Problem 27: (Tran Viet Hung) Let △ABC be a triangle with incircle (I) tangent to
AB, AC at F, E and the A−mixtilinear incircle ωa tangent to AB, AC at P, Q. CF intersects
BE at Ge . ID intersects EF at R. A′ is the reflection of A over I. Prove that: CP, BQ, Ge R
are concurrent at a point X lie on A′ D.

Solution:

Let the lines from B, C parallel to AC, AB respectively to cut EF at K, L. Using Problem
26 we get C, P, K, B, Q, L. Since A′ is the reflection of A over I so it is easy to see that
P A′ //AC//KW ,A′ Q//AB//LW where BK cut CL at W . It is easy to see that P Q//KL
also, by homothety for △A′ P Q and △W KL so CK, BL, A′ W are concurrent at X. We will
need to prove W, A′ , D, reflects W, A′ , D′ over midpoint M of BC to get A, A1 , D′ . Since
AA1 //IM and it is also well known that AD′ //IM so A, A1 , D′ and W, A′ , D, X.
Olympiad geometry document for reference 95

Diagram M.Di.76

Now we are left to prove X, Ge , R. With the idea of using Menelaus, we will need to prove

XC Ge F RK
· · =1
XK Ge C RF
Let AW cut CK at H.By Menelaus we have:

XC DC W B DC AC DC HC DC RE AC
= · = · = · = · ·
XK DB W K DB W K DB HK DB RK AE
(By Menelaus) Again,by Menelaus

Ge F EA BF
= ·
Ge C EC BA
Multiply all of the ratio we will need to prove:

DC RE AC EA BF RK
· · · · · =1
DB RK AE EC BA RF
Or
RE AB
=
RF AC
Olympiad geometry document for reference 96

Which is true because it is well known that A, M, R so


RE sin(RAE) AE sin(M AC) M C AB AB
= · = = · =
RF sin(RAF ) AF sin(M AB) M B AC AC
.

Q.E.D.

Problem 28: (Mz T) Let △ABC be an acute triangle circumscribed outside the incircle
(I). (I) is tangent to BC, CA, AB at D, E, F .Let the second intersection of AO with (ABC)
be K. Let M be the midpoint of BC, KM intersects (ABC) at N different from K. AI
intersects (ABC) at P different from A, P D intersects EF at T . AT intersects BC at X.
Let Y be the point of symmetry of D through M . Prove that the four points A, N, X, Y lie
on a circle.

Solution:

Diagram M.Di.77

Let P D intersects (ABC) at J so it is well known that J lie on (AEF ) and J, I, K, R


is the tangency point of the A−mixtilinear circle with (ABC) (Or we can define as W I
cut (ABC) where W midpoint arc BAC). Let AY intersects (ABC) again at S.We will
Olympiad geometry document for reference 97

prove N, X, R so since it is well known that RS//BC so by Reim we get A, N, X, Y lie


on a circle. Let JD intersects (AEF ) again at Z. AK intersects (AEF ) at V . Since
̸ AIZ = 180◦ − ̸ AJZ = 180◦ − ̸ AJP = ̸ AKP = ̸ AIV (Since ̸ AV I = 90◦ = ̸ AP K).
But ̸ AZI = 90◦ = ̸ AV I so AZ, AV are reflection over line AI. To prove the collinearity
,we need to show
N B RB XB
· =
N C RC XC
.We have
N B RB M B KC SC KC Y C AB KC DB AB
· = · · = · · = · ·
N C RC M C KB SB KB Y B AC KB DC AC
We also have
XB sin(BAX) AB sin(F AT ) AB T F AE AB JF ZF AB BF V E AB
= · = · = · · = · · = · ·
XC sin(CAX) AC sin(EAT ) AC T E AF AC JE ZE AC CE V F AC

BD sin(V F E) AB BD sin(KAC) AB BD sin(KBC) AB BD KC AB


= · · = · · = · · = · ·
CD sin(V EF ) AC CD sin(KAB) AC CD sin(KCB) AC CD KB AC
N B RB
= ·
N C RC

So we are done.

Q.E.D.

Problem 29: (Tran Viet Hung) Let △ABC be a triangle with A−mixtilinear incircle ωa
tangent to (ABC) at X. The line from X perpendicular to XA intersects ωa again at
P .AP intersects (ABC) at Q. O′ is the circumcenter of △XP Q. Prove that: AO′ is the
A−symmedian line of △ABC.

Solution:

Perform an inversion I centered at A, radius AB · AC and reflects over AI. Clearly XP
passes through A′ which is the antipode of A with (ABC). After I A′ becomes H where
AH ⊥ BC at H, Q will become the intersection of AP ′ and BC. The line XP will become
(AHD′ ) where the A−excircle Ω tangent to BC at D′ . P ′ will become the intersection of
(AHD′ ) and Ω. From there we can easily see Q becomes D. So that (XP Q) becomes (DD′ ).
Olympiad geometry document for reference 98

Diagram M.Di.78

After the inversion here we get AO′ reflects AM over AI.

Q.E.D.

Problem 30: (i3435) In a △ABC, let the A-mixtilinear incircle meet AB at B1 and let
the incircle of △ABC be tangent to BC at D. Prove that: B1 D meets the line through C
perpendicular to the A-angle bisector on the A-median.

Solution:
Olympiad geometry document for reference 99

Diagram M.Di.79

M is the midpoint of BC.Let the A−mixtilinear circle to tangent to AC at C1 . Let B1 D cut


CS at R where the line from C ⊥ AI intersects AB at S. With the idea of using Menelaus,
we need
RC M B AS
· · =1
RS M C AB
We have
MB
=1
MC
By Menelaus
RC DC B1 B DC B1 B
= · = ·
RS DB B1 S DB C1 C
It is well known that △BB1 I ∼ △BIC ∼ △IC1 C so
B1 B IB 2
=
C1 C IC 2
. Let AI cut BC at W , by angle chasing,we easily see ID, IW are isogonal in ̸ BIC so by
Steiner ratio equation we get
IB 2 DB W B DB AB
2
= · = ·
IC DC W C DC AC
So that
RC AB
=
RS AC
Multiply all the ratio we are done.

Q.E.D.

Problem 31: (APMC 2016) Let △ABC be a triangle with Ia is the A−excenter. The
schwatt line corresponding to vertice A of △ABC intersects the external bisector of ̸ BAC
at P . P Ia intersects the A−mixtilinear incircle at T nearer to A. Prove that: (BT C) is
tangent to the A−mixtilinear incircle of △ABC.

Solution: (livetolove212)

Let M, N be the midpoints of BC, AH where AH ⊥ BC at H. I is the incenter. S is the


midpoint of arc BC not containing A of (ABC). AI cut BC at Q. Let X, Y, W be the
tangency points of the A−mixtilinear incircle with AB, AC, (ABC). E is the midpoint of
arc BAC of (ABC) so it is well known that W, I, E. Let W I cut BC at K. It is well known
that W S, XY, BC concur at a point called G.
Olympiad geometry document for reference 100

Diagram M.Di.80

Let L be the midpoint of GQ. Let LT ′ tangent to the A−mixtilinear incircle at T ′ ,since it
is well known that W L is tangent to (ABC).We have (BT ′ C) is tangent to LT ′ at T ′ , so we
will need to prove T ≡ T ′ . We have −1 = (GK, BC) = (GJ, XY ) (Let XY cut AK at J) so
it is well known that AJ ⊥ GO′ with O′ is the center of the mixtilinear circle.Therefore GO′
intersects AK at R so it lies on (GK) (Note that it is well known that (GK) passes through
W also). Since O′ T ′ , O′ W are tangent lines of GK so −1 = (T ′ W, GR) = K(T ′ W, GR) =
−1 = (IIa , QA) =⇒ T ′ , K, Ia .
Reflects E over BC to be E ′ so P, H, E ′ .
We have −1 = (A(AE ∩ BC), P E) = K(A(AE ∩ BC), P E) = (RG, T ′ W ) =⇒ P, T ′ , K, Ia
so T ≡ T ′ .

Q.E.D.

Problem 32: Let ABC be a triangle with A−excenter J. The A−mixtilinear excircle is
tangent to (ABC) at X. A−excircle (J) is tangent to AB, AC at E, F . Prove that: The
line from M perpendicular to EF and lines XJ,EF are concurrent.
Olympiad geometry document for reference 101

Solution:

Diagram M.Di.81

This is the result we proved in the proof of Problem 23,but just change the configuration
from incircle to excircle. The idea is exactly the same. (This is called the idea of intraversion)

Q.E.D.

Problem 33: Let △ABC be a triangle with A−mixtilinear incircle ωa tangent to (ABC)
at X. The tangent from X to (ABC) intersects BC at R. D′ is the tangency point of the
A−excircle and BC. RP is tangent to ωa at P different from X. Prove that: P A, P D′ are
isogonal in ̸ BP C.

Solution:

I is the incenter of △ABC.Let XI intersects ωa again at K so it is well known that A, K, D′ .


Let EF cut BC at W so it is well known that XW is the external bisector of ̸ BXC so W
lie on XU where XI cut BC at U . It is well known that R is the midpoint of W U , since
RP = RX so P lie on (W U ).
Let KK ′ be the diameter of ωa .Now by Pascal, notice that it is well known the tangent
lines from K, K ′ to ωa is parallel to BC so they intersect at infinity, tangents at P, X to
ωa intersects at R so that U ′ R//BC (U ′ is the intersection of P K ′ , XK) or U ′ lie on BC
=⇒ U ≡ U ′ .
Olympiad geometry document for reference 102

Diagram M.Di.82

Since ̸ K ′ P K = 90◦ = ̸ W P U so W, P, K. It is easy to see that A, U lie on the polar


of W to ωa (By La Hire, W lie on the polar of A to ωa so A lie on the polar of W to
ωa , and R is the midpoint of U W ).Let P K ′ cut AD′ at T .So that −1 = U (W (U A ∩
XK ′ ), XK ′ ) = U (W A, P K) = U (D′ A, T K) = (D′ A, T K) = P (D′ A, T K) = (D′ S, U W )
(S is the intersection of AP and BC).It is easy to see that P U bisects ̸ BP C, ̸ SP D′ so
P A, P D′ are isogonal in ̸ BP C.

Q.E.D.

Problem 34: Let △ABC be a triangle with incircle (I). K is a point lie on (ABC). KI
intersects (ABC) again at L. X lie on KL such that AI bisects ̸ KAX. Prove that: (AXL)
is tangent to the A−mixtilinear incircle of △ABC.

Solution:
Olympiad geometry document for reference 103

Diagram M.Di.83


We perform an inversion I centered at A and radius AB · AC then reflects over AI. K
becomes S which is the intersection of AX with BC. I will become the A−excenter J. IK
will become the circle (ASJ). L becomes Y which is the intersection of (ASJ) and BC.
X becomes U which is the intersection of AK and (ASJ).(ALX) will become Y U . The
A−mixtilinear circle becomes the A−excircle (J). Since AJ bisects ̸ SAU so Y J bisects
̸ SY U . Since (J) is tangent to Y S so (J) is tangent to Y U =⇒ (ALX) is tangent to the
A−mixtilinear incircle.

Q.E.D.

Problem 35: Let △ABC be a triangle with incircle (I) tangent to AB, AC at E, F . EF
intersects BC at T . The A−mixtilinear circle is tangent to (ABC) at X. Prove that: T X
intersects the A−median of △ABC on (ABC).

Solution:
Olympiad geometry document for reference 104

Diagram M.Di.84

Let the incircle (I) tangent to BC at D. Let AM cut (ABC) again at R. Let XD intersects
(ABC) again at V so it is well known that AV //BC,by Reim so XDM R is cyclic. Let
XR cut BC at T ′ so T D · T M = T X · T R = T B · T C so my conversed Maclaurin we get
(T D, BC) = −1 =⇒ T ≡ T ′ .

Q.E.D.

Problem 36: ⋆ Let △ABC be a triangle with incenter I.O is the center of the circle
(ABC). The B, C−mixtilinear incircles are tangent to (ABC) at Z, T and to AB, AC at
X, Y . XY intersects T Z at S. Prove that: AS ⊥ OI.

Solution:

Let the incircle (I) to tangent to BC at D and (I) has diameter DD′ . It is well known that
XY is tangent to (I) at D′ . Let (I) tangent to AC, AB at E, F . Construct orthocenter H of
△DEF then it is well known that O, I, H. Let Mb be the midpoint of arc ABC of (ABC) and
Nb be the midpoint of arc AC of (ABC) not containing B. We perform an inversion over the
circle (I). Then XY becomes (ID′ ),Z becomes U which is the midpoint of HE (Since (ABC)
becomes the nine-point circle of △DEF ). Also we have I(HE, Mb B) = I(OE, Mb Nb ) = −1
so Mb , I, U . We have T becomes the midpoint of HF which is V . S is the intersection
of XY, ZT so it becomes S ′ which is the intersection of (ID′ ) and (IU V ). So if we need
to prove AS ⊥ OI so we will need to show the center of (IA′ S ′ ) lie on OI with A′ is the
midpoint of EF .
Olympiad geometry document for reference 105

Diagram M.Di.85

Now we will change the config of the whole problem to orthocenter, or wrt. △DEF . We
have the problem as shown:
Let △ABC be a triangle with orthocenter H,U, V are the midpoints of HB, HC. AX is the
diameter of (ABC). I is the midpoint of BC. Let (OU V ) intersects (OX) at R. Then the
center of (OIR) lie on OH.

We let OH cut BC at K then (OIK) has the center to lie on OH so our task is to prove
OIRK cyclic ⇐⇒ ̸ ORK = 90◦ − ̸ ORX ⇐⇒ K, R, X.

Let J be the nine-point circle center. Reflects over J then U, V becomes N, M which are
midpoints of AC, AB. Diameter OQ of (OU V ) becomes diameter HP of (HM N ). J is the
midpoint of P Q and Q, R, X so to prove K, R, X we will need K, Q, X. Let AA′ ⊥ M N
at A′ ,P P ′ ⊥ M N at P ′ . O′ be the center of (HM N ), since O is the midpoint of HP so
O′ A′ = O′ P ′ since O′ M = O′ N so M A′ = P ′ N . let OP ′′ ⊥ M N at P ′′ , since J is the
midpoint of OH and JM = JN we can easily see M A′ = P ′′ N =⇒ P ′′ ≡ P ′ so that
OP ⊥ M N .

Let the orthocenter of △P N O be Z so it is easy to see that △AN H, △P ZO are homothetic,


since Z lie on M N so AP, M N, OH are concurrent at a point called S. We have Kh = 2OS,
let L be the midpoint of AH so HX//OL and HX = 2OL. So that △KHX, △SOL
Olympiad geometry document for reference 106

are homothetic. Since SL bisects OP by homothety and also OP//HQ since P HQO is a
parallelogram and also OP = HQ =⇒ Q lie on KX.

Diagram M.Di.86

Q.E.D.

Comments: There is some very short solutions for this problem (See the link below)
https://artofproblemsolving.com/community/q1h477226p3709462
Despite that,we still think this problem is very hard to solve in idea.

Problem 37: Let △ABC be a triangle with A, B, C−excenters are Ia , Ib , Ic respectively.


The A, B, C−mixtilinear incircles are tangent to (ABC) at X, Y, Z. Prove that: Ia X, Ib Y, Ic Z
are concurrent.

Solution:

It is well known that AX, BY, CZ are concurrent at a point lie on OI where O is the center
of (ABC) and I is the incenter.
We will prove a more general problem:

Problem: (Generalization of Steinbart’s theorem) Let △ABC be a triangle with a point


P inside, △Pa Pb Pc is its pedal triangle. K is the circumcenter of △Pa Pb Pc .X lie on line
Olympiad geometry document for reference 107

KP ,△Xa Xb Xc is the circumcevian triangle of X wrt. △Pa Pb Pc . Prove that: AXa , BXb , CXc
are concurrent.

Proof: (Collected)

Diagram M.Di.87

Let Q be the isogonal conjugate of P , △Qa Qb Qc is the pedal triangle of Q wrt. △ABC.
Let Sa be the intersection of Qb Qc with Pb Pc . It is well known that ASa is the radical axis
of (AP ), (AQ) and ASa ⊥ P Q. By Brocard theorem so ASa ⊥ KY so that Y lie on P Q (Y
is the intersection of Pb Qc and Pc Qb ).By Pascal theorem for
 
Pb , Qb , Xc
Pc , Qc .Xb
We get Qc Xc cut Qb Xb on P Q.We get Qa Xa , Qb Xb , Qc Xc are concurrent on P Q.
We have
Y sin(Xa AB) Y X a Q c · X a Pc
=
cyc
sin(Xa AC) cyc
Xa Pb · Xa Qb
Since we proved Qa Xa , Qb Xb , Qc Xc are concurrent and we have Pa Xa , Pb Xb , Pc Xc are con-
current so by Ceva sine theorem and the cyclic property of Pa , Pb , Pc , Qa , Qb , Qc we have
Y X a Pc
=1
cyc
X a P b
Olympiad geometry document for reference 108

and
Y Xa Qc
=1
cyc
Xa Q b

So that
Y sin(Xa AB)
= 1.
cyc
sin(Xa AC)

By Ceva sine theorem so we get AXa , BXb , CXc are concurrent.

Q.E.D.

Problem 38: (OTSS Mock JMO 2020) Let ABC be a triangle with circumcenter O, in-
center I, and circumcircle Γ. The A−mixtilinear incircle is tangent to (ABC) at X. The
perpendicular bisector of BC meets line AX at a point S. Let K be the point on the (AIX),
distinct from I, so KI ∥ BC. KS cut (AIX) again at T . Prove that the tangent at T to
the (T BC) passes through the circumcenter of (T AO).

Solution: (k12byda5h)

Diagram M.Di.88

Let M, N are the intersection of internal and external bisector of ̸ BAC and Γ. It is
well-known that N, I, X. We have (angles are measured) ̸ AKI = ̸ AXI = ̸ AM N =
Olympiad geometry document for reference 109

90◦ − ̸ AIK. ̸ KAI = 90◦ , K, A, N and K, X, M .


Therefore, I is the orthocenter of △N M K. Since O is the midpoint of M N so it is well
known that O, I, T .
Now, We will set △T M O to be the main triangle. Let O′ , O1 be the center of (T AO), (T BC).
We need to show that ̸ O1 KO′ = 90◦ ⇐⇒ O1 T is tangent to (T M O) with M N . Let O1′ be
the intersection of tangent at T to (T M O). It suffice to show that (O1′ ; O1′ T ), (M ; M I), (O; OM )
are coaxial. But (O1′ ; OT ) is the T -Apollonius Circle of △T M O. Let (M ; M I) ∩ (O; OM ) =
C ′ . We need to show that
C ′M TM

=
CO TO
2
.Since, △T M O ∼ △M IO (By OM = OI · OT ),so
CM ′ MI TM

= =
CO OM TO
.

Q.E.D.

Problem 39: (i3435) In a triangle ABC, let △DEF be the intouch triangle. Let L be
the midpoint of arc BAC of (ABC) and let EO be the intersection of LD and (ABC).
Prove that: The radical center of (ABC), (EF EO ), and the A-mixtilinear incircle lie on the
A−symmedian of △ABC.

Solution: (k12byda5h)
Olympiad geometry document for reference 110

Diagram M.Di.89

Let S be the intersection of (AEF ) and (ABC) and E0 , Q be the intersections of (EF E0 ) and
(ABC). By radical axis for (ABC), (AEF ), (EF E0 ), we get AS, EF, E0 Q are concurrent at
R. Let M be the midpoint of arc BC of (ABC). It is well-known that M, D, S. Let Z be
the orthocenter of △LDM which lie on M E0 , BC, LS. By Pascal for QE0 M ASL, we get
R, Z, N (Where M A cut QL at N ).
S
Since (BC, ZD) = (BC, LM ) = −1, so Z lies on EF . Therefore, N is the concurrency point
of AM, LQ, EF . Let P be the intersection of DL and AM and M ′ be the midpoint of BC
and K ′ is AM cut BC and A−symmedian intersect (ABC) at K. We have (BC, KA) =
−1 = (BC, LM ) so L, K ′ , K are collinear. Spiral similarity at S sends △LBC ∪ {M ′ } →
△AF E ∪{N } so ̸ SDL = ̸ SM ′ L = ̸ SN A (Angles are measured). Therefore, M K ′ ·M A =
M I 2 = M D · M S = M P · M N . Hence, {A, K ′ }, {I, I}, {N, P } are recipocal pairs. Project
through L, we get AK, T T, QE0 are concurrent (T is the mixtilinear touch point and well-
known that L, I, T are collinear). Which completes the proof.

Q.E.D.

Problem 40: (Iran) Let △ABC be a scalene triangle,AB < AC with incircle (I) tangent
to BC at D.AI cut BC at E. A1 lie on (ABC) such that AA1 //BC. Let DA1 intersects
(DAE) at T . Prove that: IA = IT .

Solution:
Olympiad geometry document for reference 111

Diagram M.Di.90

Let X be the tangency of the A−mixtilinear circle and (ABC). It is well known that X, D, A1
and XI bisects ̸ AXA1 ≡ ̸ AXT . We have ̸ AT D = ̸ AED = ̸ AEB = ̸ ACB + 12 ̸ BAC.
Also ̸ AXT = ̸ AXA1 = ̸ ACA1 = ̸ BCA1 − ̸ BCA = ̸ ABC − ̸ ACB.By simple angle
chasing we can get 90◦ − 1̸
2
AXT = ̸ AT X =⇒ △XAT is isosceles at X. Since XI is the
bisector of ̸ AXT so XI is the perpendicular bisector of AT so IA = IT .

Q.E.D.

Problem 41: In △ABC, M lie on BC such that AM is tangent to (ABC). Suppose T


is the tangency point of the A-mixtilinear incircle with (ABC); let Ma be the midpoint of
major arc BAC and R as the midpoint of the segment connecting midpoints of major arcs
ABC and ACB. Show that lines M T and Ma R intersect on (ABC).

Solution:

Diagram M.Di.91

Let △Ia Ib Ic be the triangle formed by the external bisectors of △ABC. Let I be the
incenter so it is well known that T, I, Ma . Let T D cut (ABC) at W so it is well known
Olympiad geometry document for reference 112

that AW//BC. So that ̸ M DT = ̸ AW T = ̸ M AT so M ADT is cyclic. It is easy to see


that P, Q, Ma are midpoints of segments of △Ia Ib Ic so clearly Ma , R, Ia . To prove M, T, S
where S is the intersection of SMa and (ABC) so we need ̸ AT M = 180◦ − ̸ AT S. We have
̸ AT M = ̸ ADB, also 180◦ − ̸ AT S = ̸ ACS = ̸ AMa Ia . So we need ̸ Ia Ma Ic = ̸ ADB.
Change the configuration to △Ia Ib Ic ,we get the orthocenter configuration problem and we
will prove the below problem:
Problem: Let △ABC be a triangle with altitudes AD, BE, CF concur at H. HP is
perpendicular to EF at P . M is the midpoint of BC. Prove that: ̸ AM C = ̸ DP E.

Proof:

Diagram M.Di.92

Let R, L be the midpoints of segments BF, CE. Reflects F, B over P, D to get H ′ , B ′ . Since
̸ HF F ′ = ̸ HBB ′ so △HF F ′ ∼ △HBB ′ and they are isosceles, from here we can easily
prove △HF B ′ ∼
= △HF ′ B so 2RP = BF ′ = B ′ F = 2RD =⇒ RP = RD ,similarly
LP = LD. So that P D ⊥ RL. Since BN//BE ⊥ AC and LN//CF ⊥ AC so N is the
orthocenter of △ARL so that AN ⊥ RL so DP//AN . From here we get ̸ AM C = ̸ AN F
(since △ABC ∼ △AEF ) = ̸ DP E.

Q.E.D.

Comments: Readers can prove the result DP//AN using Thales also. There is also a very
interesting Generalization by buratinogigle like this:
Generalization: Let ABCD be cyclic quadrilateral inscribed in circle (O) with AC cuts
Olympiad geometry document for reference 113

BD at G, AB cuts CD at P . Circle (K) is tangent to segment GC, GD at E, F and (O) at


Q. M, N are midpoints of major arcs BC, AD. J is midpoint of EF . QJ cuts (O) again at
L. P Q cuts (O) again at R. Prove that LR bisects segment M N .

We will not present the solution here but if any readers are curious, here is a solution posted
on AoPs by sir Luis Gonzales:
https://artofproblemsolving.com/community/c6h1215947p6052570

Problem 42: (HSGS) Let ABC be an acute triangle. (K) is A-mixtilinear incircle and it
touches (ABC),AC, AB at D, E, F . AL is the diameter of (ABC). LC cut KF at M, LB
cut KE at N . Prove that: AD ⊥ M N .

Solution:

Diagram M.Di.93

Let KL cut M N at Z so since KM LN is a parallelogram so Z is the midpoint of M N . Let


(AK) or (AEF ) intersects (ABC) again at S so clearly S, K, L. Let AD cut EF at P so

PF sin(P AF ) AE sin(BAD) sin(BCD) DB


= · = = =
PE sin(P AE) AF sin(CAD) sin(CBD) DC

It is well known that


DB BF
=
DC CE
So that
DB BF SF
= =
DC CE SE
Olympiad geometry document for reference 114

(By similarity of △SF B,△SEC).

PF SF
=⇒ =
PE SE
so SP is the bisector of ̸ F SE and since K is the midpoint of arc EF not containing A of
(AK) so S, P, K.
Let AP cut (AEF ) again at Q so it is easy to see that QA bisects ̸ F QE so

QF PF SF
= =
QE PE SE

so that quadrilateral SF QE is harmonic ,so that K(QZ, N M ) = (QS, EF ) = −1 but Z is


the midpoint of M N as proven so QK//M N . Since ̸ AQK = 90◦ so M N ⊥ AD.

Q.E.D.

Problem 43: (Cosmin Pohoatza) Let ABC and A1 B1 C1 be two triangles having the same
circumcircle O(O, R) and the same incircle I(I, r). Suppose the sidelines BC and B1 C1 are
parallel, prove that A1 is the tangency point of the A-mixtilinear incircle of triangle ABC
with O.

Solution:

Diagram M.Di.94

Let A1 B1 , A1 C1 intersects BC at B ′ , C ′ . Since B ′ C ′ //BC so by homothety (A1 B ′ C ′ ) is


tangent to (ABC), but by Example M.E.9 we get (A1 B ′ C ′ ) passes through the tangency
point of the A−mixtilinear incircle with (ABC),call it T . So that A1 ≡ T .
Olympiad geometry document for reference 115

Q.E.D.

Problem 44: Let △ABC be a scalene triangle with the B−mixtilinear incircle tangents
to side AB at M and C−mixtilinear incircle tangents to side AC at N .Prove that: (AM N )
is tangent to the A−mixtilinear incircle of △ABC.

Solution:

Diagram M.Di.95

It is well known that M N//BC and M N is tangent to incircle (I) of △ABC so the problem
becomes the special case of Example M.E.12.

Q.E.D.

Problem 45: (Romania TST 2018) Let ABC be a triangle, let I be its incenter, let Ω be
its circumcircle, and let ω be the A- mixtilinear incircle. Let D, E and T be the intersections
of ω and AB, AC and Ω, respectively, let the line IT cross ω again at P , and let lines P D
and P E cross the line BC at M and N respectively. Prove that: Points D, E, M, N lie on
a circle. Define the center of that circle.

Solution:

It is well known that P is the tangency point of the tangent line of ω which is parallel to
BC. Let P P ′ be the diameter of ω and let P P ′ cut BC at W so P P ′ ⊥ BC at W . Also
notice that ̸ P DP ′ = 90◦ = ̸ P EP ′ so we have P D · P M = P W · P P ′ = P E · P N . So that
M, D, E, N lie on a circle.
Olympiad geometry document for reference 116

Diagram M.Di.96

Now we will define the center of the circle.Let J be the A−excenter.Since ̸ BM D =


̸ P P ′D = ̸ P DA = ̸ M DB so BM = BD,similarly CN = CE. Since JB, JC are ex-
ternal bisectors so clearly JB, JC are perpendicular bisectors of DM, EN so clearly J is the
center of the mentioned circle.

Diagram M.Di.97

Q.E.D.

Problem 46: (Peru) Let ABC be a triangle, if BD is the angle bisector of B(D ∈ AC),
the incircle of BDC touch BC at E. Let G be projection of E at BD, and M the midpoint
of BE. The B-excircle of △ADB touch BD at F . Suppose ABM G is cyclic.Prove that: G
is circumcenter of △AM F .
Olympiad geometry document for reference 117

Solution: (Collected)

Diagram M.Di.98

Let Ib be the B−excircle center of △BM G,since △BGE is right at G and M midpoint BE
we can easily prove Ib lie on the perpendicular bisector of GE,it is also easy to prove that
Ib lie on (BGE) because Ib lie on the angle bisector of ̸ GBE, so that ̸ BIb E = 90◦ then it
is well known that the incircle of △BCD is the B−mixtilinear excircle of △BGM . Let the
incircle of △BDC tangent to CD at V . It is well known that E, V, F . By the Sawayama
lemma, we have the B−excenter of △ABM lie on EV hence F is also the excenter of △ABM
then GA = GF = GM .

Q.E.D.

Problem 47: (Jean-Louis Ayme) Let △ABC be a triangle with the B, C−mixtilinear
circles ωb , ωc tangent to (ABC) at B ′ , C ′ . BB ′ , CC ′ intersects ωb , ωc again at L, K. Tangents
from K, L to ωc , ωb intersects at P . Prove that: AP is the A−symmedian line.

Solution:

We suppose AB < AC, other cases are similar.


Let ωb , ωc tangent to AB, AC at Z, W .Let ωc to touch BC at V . Let P L, P K cut AB, AC
at X, Y . It is well known that C ′ IW A is cyclic where I is the incenter.
Olympiad geometry document for reference 118

Diagram M.Di.99

Since ̸ KW V = ̸ KC ′ V = ̸ IC ′ W (Since quadrilateral KW C ′ V is harmonic) = ̸ IAW


=⇒ (AW I) is tangent to KW .It is easy to see that △Y W K is isosceles. Since ̸ W Y K =
180◦ − 2̸ KW Y = 180◦ − 2(̸ CW V − ̸ KW V ) = 180◦ − 2(90◦ − 1̸
2
ACB − ̸ BAI) =
̸ ACB + ̸ BAC = 180◦ − ̸ ABC = ̸ AC ′ K
=⇒ AC ′ KY is cyclic.
So that CW 2 = CK · CC ′ = CY · CA so A, Y are reflection over (C; CW ),similarly X, A are
reflection over (B; BZ) so it is easy to see that W Z//BC//XY (Proof by Thales and note
that it is well known W Z//BC). We have ̸ XY K = ̸ AY K − ̸ AY X = 180◦ − ̸ AC ′ C −
̸ ACB = 180◦ − ̸ ABC − ̸ ACB = ̸ BAC so P Y is tangent to (AXY ) ,similarly =⇒
P is the intersection of two tangent lines from X, Y to (AXY ),note that XY //BC so by
homothety clearly AP passes through the intersection of the tangent lines from B, C to
(ABC) which AP is the A−symmedian line.

Q.E.D.

Problem 48: Let △ABC be a triangle with the A-mixtilinear circle tangent to AB, AC, (ABC)
at F, E, X. EF intersects (ABC) at Q, P where F lie between Q, E. ID intersects BC at
T . QT intersects (ABC) at R. Prove that: AR ⊥ KP .

Solution:
Olympiad geometry document for reference 119

Diagram M.Di.100

It is well known that XI bisects ̸ BXC and


BE XB
=
CF XC
so that
TB BE
=
TC CF
Combine AE = AF , multiply the ratios and by Ceva conversed theorem we get AT, BE, CF
are concurrent.Let AR intersects (AEF ) again at V ′ .So that (P V, F E) = A(P V, F E) =
(P R, CB) = Q(P R, CB) = ((P Q ∩ BC)T, CB) = ((EF ∩ BC)T, CB) = −1 (Since we
proved that AT, BE, CF are concurrent). Also since V ′ A bisects ̸ F V ′ E so K, V ′ , P and
KP ⊥ AR.

Q.E.D.

Comments: There is an interesting Generalization as below:


Generalization: (Khoa) Let △ABC be a triangle with isogonal cojugate pair P, Q.QD ⊥
BC at D. P E, P F ⊥ AC, AB at E, F . EF intersects (ABC) at M, N and F lie between
M, E. N D intersects (ABC) again at K. Prove that: AK ⊥ P M .

Problem 49: (Dogu) Let △ABC be a triangle with A−mixtilinear incircle is tangent to
AB, AC at E, F . Incircle (I) is tangent to AB, AC at X, Y . EF intersects (I) at R. AR
intersects (I) again at K. T is the midpoint of XY and L is the midpoint of AT . Prove
that: KL is tangent to (I).
Olympiad geometry document for reference 120

Solution:

Let the tangent from K, R to (I) intersects at S so −1 = (AZ, KR) = S(AZ, KR) =
(AT, (SK ∩ AI)R) =⇒ SK cut AI at L which is the midpoint of AT .

Q.E.D.

Problem 50: (IMO 2019) The incircle ω of acute-angled scalene △ABC has centre I and
meets sides BC, CA, and AB at D, E, and F, respectively. The line through D perpendicular
to EF meets ω again at R. Line AR meets ω again at P . (P CE) intersects (P BF ) again
at Q . Prove that: Lines DI and P Q meet on the external bisector of ̸ BAC.

Solution: (IMO SL)

Diagram M.Di.101

Suppose AB < AC. The other cases are the same.


Let X ne the tangency of the A−mixtilinear incircle with (ABC). By Example M.E.5 so
A, R, X. M ′ is the midpoint of arc BAC of (ABC) so it is well known that X, I, M ′ .U, Y are
Olympiad geometry document for reference 121

the intersections of (BP F ) with AX, EF . Since ̸ BU P = ̸ BF P = ̸ F RP so F R//BU . By


inversion it is well known that RF A1 X is cyclic where the mixtilinear circle tangent to AB
at A1 . It is also well known that A1 BXI is cyclic so ̸ BIX = ̸ BA1 X = ̸ F RX = ̸ BU X
=⇒ BU IX is cyclic.
Also ̸ Y U B + ̸ BU I = ̸ Y F B + ̸ BA1 I = ̸ F A1 I + ̸ BA1 I = 180◦ =⇒ I, U, Y .
Let EF intersects BC at S. By simple angle chasing have ̸ ICB = ̸ REF so that ̸ IY F =
̸ U P F = ̸ RP F = ̸ REF = ̸ ICB so SY IC is cyclic
Let (P CE) intersects EF at Z and intersects AX at V , with the same idea we can prove
that C, V, I, X lie on a circle and B, I, Z, W lie on a circle.
Let BY cut CZ at Q′ . Then ̸ Y Q′ B = ̸ CY Z + ̸ BZY = 180◦ − ̸ SIC + ̸ SIB =
180◦ − ̸ BIC = ̸ AF E = ̸ Y F B =⇒ Q′ lie on (P BF ),similarly so Q lie on (P CE) we get
Q ≡ Q′ . (The case when Q′ is P is easy).
Let N be the midpoint of EF ,DN cut (DEF ) again at S. It is well known that SI ⊥ AD,also
since IC ⊥ DE so 180◦ − ̸ SIC = ̸ EDA = ̸ SDF (By symmedian line property).
Let DD′ be the diameter of incircle (I),consider the configuration of △DEF so it is well
known that D′ , N, P , since D′ , R reflects over AI so we will go prove P, Q, S.
We have ̸ SP F = ̸ SDF = ̸ EDA = 180◦ − ̸ SIC = 180◦ − ̸ SY C = ̸ QY F = ̸ QP F
=⇒ P, Q, S.

Q.E.D.

Comments: Of course there is many easier solution for this problem but because this is a
Mixtilinear book so I want to introduce the usage of Mixtilinear circle.

Problem 51: Let △ABC be a triangle with AB + AC = 3BC with circumcenter O. Prove
that: (AO) is tangent to the A−mixtilinear incircle of △ABC.

Solution:

Let M, N be the midpoints of AB, AC. Perform an inversion centered at A with radius

AM · AN . Then the mixtilinear circle becomes the incircle (I) and (AO) becomes M N .
We need to show M N is tangent to (I) or quadrilateral BM N C is bycentric but it is true
by Pytot theorem since BM + CN = M N + BC (Use AB + AC = 3BC).
Olympiad geometry document for reference 122

Diagram M.Di.102

Q.E.D.

Problem 52: (Dsoong) Let △ABC be a triangle with A−mixtilinear incircle tangent to
(ABC) at T . I, O are incenter,circumcenter of △ABC. OI intersects AT at X. Tangent
lines from B, C to (ABC) intersects at K. IT intersects BC at L.Prove that: X, L, K.

Solution:

Diagram M.Di.103
Olympiad geometry document for reference 123

Let M, N be the midpoints of arcs BC (not containing A),BAC of (ABC) and S is the
midpoint of BC. It is well known that T, I, N and N, O, M, K.
With the idea of using Menelaus theorem, to prove the collinearity we will need to show

XI KO LN
α= · · =1
XO KN LI
Let ID ⊥ BC at D. We have
LN NS
=
LI ID
It is well known that X is the exsimilicenter of (ABC) and incircle (I). We have

XI ID
=
XO NO
Multiply all of them,we will need to show

ID KO N S
α= · · =1
N O KN ID
Since
OS ON NS KN
OS · OK = ON 2 ⇐⇒ = ⇐⇒ =
ON OK NO KO
Substitute the ratio we get α = 1.

Q.E.D.

Problem 53: (tkhalid) Let △ABC be a triangle with incircle ω. Let I be the center of ω
and let D, E, and F be the points of contact of ω with BC, CA, and AB respectively. Let
Ω be the circle passing through B and C tangent to ω at a point X. Furthermore let Ω
intersect AB and AC at P and Q respectively. Let DF intersect BI at Y and CI intersect
DE at Z. Finally let P Y intersect QZ at O. Prove that: B, Y, Z, C lie on a same circle with
center O.

Solution:

Since IY · IB = ID2 = IZ · IC so BY ZC is cyclic.


By mixtilinear circle property for △P BC, △QCB so it is easy to see that Y, Z are incenters
of △P BC, △QCB and P, Y, O′ , Q, Z, O′ with O′ is the midpoint of arc BC not containing
P, Q of (BP QC) =⇒ O ≡ O′ . Since Y, Z are incenters of △P BC, △QCB, so we have
OY = OB = OC = OZ which concludes O is the center of (BY CZ).
Olympiad geometry document for reference 124

Diagram M.Di.104

Q.E.D.

Problem 54: (Dogu) Let △ABC be a triangle with the A−mixtilinear circle tangent to
(ABC) at X. M is the midpoint of major arc BAC of (ABC). J is the A−excenter. M X
intersects (BJC) again at S. Tangents from S, J to (BJC) intersects at T . Prove that:
M, K, T with K is the intersection of the A−symmedian line with (ABC).

Solution:

Diagram M.Di.105

Let I be the incenter of △ABC so it is well known that X, I, M . Let N be the midpoint
Olympiad geometry document for reference 125

of arc BC not containing A of (ABC). Since quadrilateral ABKC is harmonic so it is well


known that AN, KN, BC are concurrent at a point called E.
It is well known that M B, M C are tangent to (IBC) so that M I · M S = M B 2 = M A · M P
=⇒ EAIS is cyclic, it is easy to see that ̸ M AI = 90◦ so ̸ ISE = 90◦ . Since IJ is the
diameter of (BIC) =⇒ J, S, E.
Perform an inversion over (BIC). K becomes E,T becomes R which is the midpoint of
SJ.M becomes Z which is the midpoint of BC. Since it easy to show that A, Z, N, R, E lie
on a circle which is (N E) so by the inversion we get M, K, T .

Q.E.D.

Problem 55: (Viet Nam TST 2017) Let △ABC be a triangle with I is its incenter and the
circle (I) is tangent to BC, CA, AB at D, E, F respectively. Denote Ib , Ic as the excenters
of triangle ABC respect to vertices B, C. Let P, Q be the midpoints of segments Ib E, Ic F .
Suppose that (P AC) intersects AB at the second point R and (QAB) intersects AC at the
second point S.

i) Prove that: P R, QS, AI are concurrent.

ii) Suppose that DE, DF intersect Ib Ic at K, J and EJ meets F K at M . The lines


P E, QF intersect (P AC), (QAB) at X, Y (X differs from P and Y differs from Q).
Prove that BY, CX and AM are concurrent.

Solution: (Viet Nam IMO Booklet 2017)

i) Since EF and Ib Ic are both perpendicular to AI then Ib Ic F E is the trapezoid. So P Q is


the midline of both trapezoid Ib Ic F E and triangle AEF . Thus P, Q belong to the radical
axis of the degenerate circle (A, 0) and (I). Similarly, Q belongs to the radical axis of (B, 0)
and (I). Hence, QA2 = QF · QY = QB 2 , which implies that (QAB) is tangent to (I) at Y
. Similarly, (P AC) is also tangent to (I) at X. Thus, (I) is the S−Mixtilinear of triangle
ASB then the incenter of triangle ABS is the midpoint N of the segment EF which implies
that SQ is the angle bisector of ̸ ASB and then SQ passes through N . Similarly, RP also
passes through N. Therefore, P R, QS, AI are concurrent at the point N.
Olympiad geometry document for reference 126

Diagram M.Di.106

ii) In the circle (I), the line Ib Ic is the antipole of N then JE, KF, DN are concurrent at the
point M that lies on circle (I). By Steinbart’s theorem, we can see that to prove three lines
AM, BY, CZ are concurrent, we have to prove that F X, EY, DM are concurrent. We have
̸ EY F = ̸ AEF = ̸ Ib AC then quadrilateral Ic AEY is cyclic. Similarly, the quadrilateral
AIb XF is also cyclic. These mean the point X, Y define above are the same as definition in
problem. Consider the transformation S which is the union between the inversion IAAB·AC
and the reflection respect to the line RAI . We have S : (O) ←→ A BC,(I) ←→ (T ) with
(T ) is the A−mixtilinear excircle of triangle ABC. This circle is tangent to AC, AB at E ′ , F ′
then E ←→ E ′ , F ←→ F ′ .
It is well known that excenter Ia is the midpoint of segment E ′ F ′ . By Pappus’s theorem ,
we have Ib E ′ meets Ic F ′ at the point Z lie on BC.
Denote G as the tangent point of (T ) with (O). We already know that Ia G passes through
the point L, the midpoint of the arc BAC of circle (O) which is also the midpoint of Ib Ic .
But Ib Ic //E ′ F ′ then since by Thales’s theorem, we get L, Z, G, Ia .
Olympiad geometry document for reference 127

We have S : E ′ Ib ←→ (Ic AE), F ′ Ic ←→ (Ib AF ), D ←→ G, I ←→ Ia then GIa ←→ (AID).


Since E ′ Ib , F ′ IC, Ia G are concurrent then circles (AIc E), (AIb F ), (AID) are coaxial. We have
N M · N D = N E · N F = N A · N I then AM ID is cyclic. Consider the radical axis of three
circles (I), (Ic AE), (Ib AF ), we have EY cuts F X at U which is the radical center of these
circles. Continue to consider the radical axis of three circles (I), (Ic AE), (AID), we have
M D cuts EY at U ′ which is the radical center of these circles. But (AIc E), (AIb F ), (AID)
are coaxial, which implies that U ≡ U ′ . Therefore, three lines M D, EY, F X are concurrent
at U .

Q.E.D.

Problem 56: Let △ABC be a triangle with A−mixtilinear excircle is Ω. F lie on segment
BC. G, H are distinct points lie on Ω. Suppose (AF G), (AF H) are tangent to Ω at G, H.
Prove that: (F GH) passes through the A−excenter Ia of △ABC.

Solution:

Diagram M.Di.107


Perform an inversion centered at A with radius AB · AC and a reflection over AI ,RAI . F

will become a point F which moves on (ABC). Ω becomes the incircle (I). (AF G), (AF H)
become the tangent lines from F ′ to (I). G, H will become X, Y which is the tangency points
of the tangent lines mentioned with (I).Ia will become I
To prove F, G, H, Ia lie on a circle, after the inversion and reflection, we will need to prove
F ′ , X, Y, I lie on a circle. Let F ′ X, F ′ Y intersects (ABC) again at U, V . By Poncelet porism,
U V is tangent to (I) =⇒ (I) is the incircle of △F ′ U V so clearly (F ′ XY ) passes through I.
Olympiad geometry document for reference 128

Q.E.D.

Problem 57: (Pindp) Let △ABC be a triangle with incenter I, circumcircle (O), (K) is
the A mixtilinear incircle, Ia is the A excenter. X is the tangency point of incircle of △ABC
in the side BC. (K) touches (O) at D. DI cuts BC at P . Prove that: AX cuts Ia P at a
point on (K)

Solution:

Diagram M.Di.108

Let N lie on AD so N P//AI. Q be the midpoint of BC. O be the intersection of AD, IQ.
M is the midpoint of arc BAC. XJ is the diameter of (I). It is well known that N lie on
EF and O is the midpoint of AN . From K perpendicular to BC cut (K) at H, G (H closer
to A)
By homothety at D =⇒ H lie on D, I, M . By homothety at A we also get A, J, H, A, X, G.
Let N be the midpoint of arc BC not containing A. Since ̸ M DG = ̸ HDG = 90 =⇒
D, G, N .
With the idea of using Menelaus conversed theorem, to prove P, G, Ia ,we need to prove that
P D GN Ia I
=1
P I GD Ia N
Olympiad geometry document for reference 129

By Menelaus we got
IH KG N D
· · =1
ID KH N G
We know that
KG = −KH

We receive
ID ND ID − HI DN − GN DG
=− =⇒ = =
IH NG IH NG NG
Substitute in we need to show
PD IH Ia I
· · = −1
P I ID − HI Ia N
It is well known that
Ia I
Ia I = 2Ia N =⇒ = 2; (1)
Ia N
As mentioned above, O is the midpoint of AN and we get OI//A, J, H.

IH AO ON ON
= = = ; (2)
ID − HI OD − AO OD − ON ND

PD ND ND
= = ; (3)
PI NA 2N O
. From (1), (2), (3) multiply them in we get

PD IH Ia I ND ON
· · =( )·( )2 = −1
P I ID − HI Ia N 2N O ND
. Hence proven.

Q.E.D.

Problem 58: Let △ABC be a triangle with the A−mixtilinear circle ω tangent to (ABC)
at X ′ , E, F are the tangency points of ω with AB, AC.Suppose EF intersects BC at S. A
line d passes through S and intersects ω at distinct points X, Y . AX, AY intersects BC at
U, V . Prove that: (X ′ U V ) is tangent to (ABC).

Solution:
Olympiad geometry document for reference 130

Diagram M.Di.109

Let ω intersects BC at P, Q. It is easy to see that Y −→ X, F −→ E, Q −→ P is an


involution. Project this involution onto BC by A so we get V −→ U, F −→ E, Q −→ P is
an involution ,since (X ′ BC), (X ′ P Q) are tangent so (X ′ U V ), (X ′ BC), (X ′ P Q) are tangent.

Q.E.D.

Problem 59: Let △ABC be a scalene triangle with the A−mixtilinear incircle,excircle are
ω, Ω. The incircle (I),A−excircle (J) is tangent to BC at D, D′ . ω, Ω are tangent to (ABC)
at X, Y . Suppose AX cut (I) at P farther to A, AY cut (J) at Q closer to A.Prove that:
P, D, Q, D′ lie on a circle.

Solution:
Olympiad geometry document for reference 131

Diagram M.Di.110

It is well known that XI, JY intersects at point N which is the midpoint of arc BAC of
(ABC). Let S lie on (ABC) such that AS//BC. It is well known that X, D, S and Y, D′ , S.
It is also well known that XI bisects ̸ AXS and Y J bisects ̸ AY S. Let P ′ be the reflection
of D over IX so clearly P ′ lie on AX and P ′ lie on (I) =⇒ P ≡ P ′ or P is the reflection
of D over XI. Similarly D′ , Q are reflections of each other over Y J so that N P = N D
and N Q = N D′ , since it is well known that N D = N D′ =⇒ N P = N D = N D′ = N Q
=⇒ P, D, Q, D′ lie on a circle with center N.

Q.E.D.

Problem 60: Let △ABC be a triangle with T is the tangency point of the A−mixtilinear
circle with (ABC).The line from T perpendicular to BC intersects (ABC) again at D. Prove
that: The nine-point circle of △DBC is tangent to the incircle (I) of △ABC.

Solution:

Diagram M.Di.111
Olympiad geometry document for reference 132

Tangent lines from T to (I) intersects (ABC) at U, V . It is well known that U V //BC and
U V is tangent to (I). Let M, M ′ be the midpoints of BC, U V . Let DT intersecrs BC, U V
at X, X ′ .
Let α be the nine-point circle of △DBC with center N1 .Let ω be the nine-point circle of
△T U V with center N2 . Let Ω be the nine-point circle of △DU V with center N3 .It is well
known that Rα = Rω = RΩ = 12 R(ABC) or these circles are congruent
Let ψ be the line from I parallel to BC. Perform a reflection over ψ called Rψ . Let
D be the tangency point of (I) with BC, DD′ is the diameter of (I) so D′ lie on U V
so that Rψ : D ←→ D′ . From that, and since XX ′ ⊥ U V, BC; M ′ M ⊥ U V, BC so
Rψ : M ←→ M ′ ; X ←→ X ′ .Let Rψ : α ←→ Γ.So there are two possible case of Γ are ω or
Ω.We will prove that Ω is not Γ.
Reflects T over U V to get T ′ . The circumcenter of △T ′ U V is O′ . Clearly D is the orthocenter
of △T ′ U V so O′ , N3 , D and N3 is the midpoint of segment O′ D. It is easy to see that
△T ′ U V , △T U V are reflections of each other over U V so that O, O′ are reflections of each
other over U V =⇒ M is the midpoint of OO′ =⇒ N3 M ′ //OD. Let H be the orthocenter
of △DBC and J is the midpoint of segment DH, it is well known that N1 is the midpoint
of segments OH,JM and DOM J is a parallelogram =⇒ JM//OD =⇒ JM//N3 M ′ =⇒
N1 M//N3 M ′ but it is well known that N1 , N2 , N3 (Notice that nine-point center is the
midpoint of segment connecting orthocenter,circumcenter) and N1 , N2 , N3 ⊥ U V, BC and
also M, O, M ′ ⊥ U V, BC so it is easy to see that N3 M ′ M N1 is a parallelogram.
Suppose Γ ≡ Ω so Rψ : α ←→ Γ so that R : N1 ←→ N3 but since R : M ←→ M ′ so
M M ′ N3 N1 is a trapezoid, but we proved M M ′ N3 N1 is a parallelogram,from here we can
easily show contradiction.
So that Γ ≡ ω. By Feuerbach theorem α is tangent to (I) but since R : α ←→ ω, (I) ←→ (I)
so ω is tangent to (I).

Q.E.D.

Comments: We can see that this problem is not simple, many people must have fake solved
or blunder on it because they forgot about the second nine-point circle. It is important that
we need to be strict on every proof.

Problem 61: (Dsoong) Let △ABC be a triangle with the A−mixtilinear incircle ω tangent
to AB, AC, (ABC) at X, Y, T . Let J be the center of ω and let JX, JY intersects ω again at
V, U . AU, AV intersects ω again at F, E.S is the reflection of A over T . Tangent lines from
A, I to (AIS) intersects at K. L is the center of (AEF ). Prove that: KL passes through
the circumcenter O of △ABC.
Olympiad geometry document for reference 133

Solution:

Perform an inversion I centered at A with radius AB · AC then a reflection over AI,
′ ′
RAI . Then X, Y becomes E , F which are the tangency points of the A−excircle (Ia ) with
AC, AB. U becomes H which is the intersection of (AF ′ ) with (Ia ). F becomes F ” which
is the second intersection of AH and (Ia ),clearly F ′ F ” is the diameter of (Ia ). Similarly we
construct point R and E” and we get E ′ E” is the diameter of (Ia ). Let (AEF ) intersects
(ABC) again at G so the problem is equivalent to proving (AIG), (AIS) are orthogonal. We
can see that,after I and RAI we get G becomes Z which is the intersection of E”F ” and
BC. (AIG) becomes the line Ia Z. T becomes D′ which is the tangency point of (Ia ) with
BC, S becomes Z which is the midpoint of AD′ . It is well known that Ia , M, Z (Readers
can prove by let D be the touch point of incircle and BC then use Thales). We have (AIS)
becomes the line Ia , M, Z. So our task is to prove Ia M ⊥ Ia Z.

Diagram M.Di.112

We will prove this problem instead in order to complete the problem.


Problem: Let △ABC be a triangle with A−excircle (Ia ) tangent to AC, AB at E ′ , F ′ .
E ′ E”, F ′ F ” are diameters of (Ia ). E”F ” intersects BC at Z. M is the midpoint of segment
BC. Prove that: ̸ M Ia Z = 90◦ .
Olympiad geometry document for reference 134

Solution:

Diagram M.Di.113

Let incircle (I) tangent to BC at D. (Ia ) is tangent to BC at D′ and construct diameter


D′ D” of (Ia ). Reflects A over Ia to get A′ . Clearly D′ A′ //D”A, also by homothety A, D, D”
so AD//A′ D′ . It is a well known lemma for △D′ E”F ” that Ia Z ⊥ D′ A′ , by the results we
proved above so Ia Z ⊥ AD. But it is well known that Ia M//AD so Ia Z ⊥ Ia M .

Q.E.D.

Problem 62: In △ABC,X is the tangency point of the A−mixtilinear circle with (ABC).
Prove that: The A−mixtilinear incircles of △ABX, △ACX are tangent.

Solution:

Perform an inversion centered at A, with radius AB · AC and reflects over AI.
The A−mixtilinear circle will become the A−excircle (Ia ) .X will becomes D′ where (Ia )
is tangent to BC at D′ . The A−mixtilinear incircles of △ABX, △ACX will become the
A−excircles of △AD′ C, △AD′ B. Let I1 , I2 be the A−excenter of △ABD′ , △ACD′ . To
prove the problem, we will need to show (I1 ), (I2 ) are tangent. Let I1 R ⊥ AD′ at R.
Olympiad geometry document for reference 135

Diagram M.Di.114

We have D′ R = 12 (BA+BD′ −AD′ ) = 21 (BA+ 12 (CA+CB −AB)−AD′ ) = 12 ( 12 (AB +BC +


CA) − AD′ ). Similarly, let CR′ ⊥ BC at R′ so D′ R′ = 21 ( 12 (AB + BC + CA) − AD′ ) = D′ R
=⇒ R ≡ R′ .

Q.E.D.

Problem 63: ⋆ Let △ABC be a triangle with D lie on arc BC of (ABC) that does not
contain A. Prove that: The A−mixtilinear incircle ω of △ABC,the D−mixtilinear incircles
of △DAB, △DAC have a same tangent line.

Solution: (Luis Gonzales)

Let ψ be the common tangent of the D−mixtilinear incircles ω1 , ω2 of △DAB, △DAC. ψ


intersects arcs AB, AC of (ABC) that not containing the other vertice of △ABC. Let
ψ tangent to ω1 , ω2 at X, Y . Let I be the incenter of △DEF and I1 , I2 are incenters of
△DAB and △DAC. Let ω1 , ω2 tangent to AD at X ′ , Y ′ . By the well known lemma, or the
Sawayama lemma we get X, X ′ , I, Y, I, Y ′ ,from here it is easy to show that ̸ XIY = 90◦ . Let
O1 , O2 be the centers of ω1 , ω2 . Let O1 O2 intersects (XY ) again at R. We have O1 R · O1 I =
O1 X 2 = Rω1 = O1 I1 · O1 D =⇒ I1 , R, I, D lie on a circle, similarly I2 , R, I, D lie on a circle,
so D, I1 , I, I2 lie on a circle. Using Problem 1 we get (DI1 I2 ) passes through T which is
the tangency point of the A−mixtilinear incircle with (ABC).
Olympiad geometry document for reference 136

Diagram M.Di.115

Now we will prove a lemma;


Lemma: Let ABCD be a quadrilateral inscribed in ω with W is a point lie on ω. Suppose
diagonals AC cut BD at E.Suppose there exists a circle Ω internally tangent to ω at R
and tangent to segments EA, ED. Incenters of △W AC, △W BD are I1 , I2 . Prove that:
I1 , I2 , W, R lie on a circle.

Solution:

Diagram M.Di.116

Let Ω to touch EA, ED at P, Q. Let RP, RQ intersects ω again at X, Y . By homothety


Olympiad geometry document for reference 137

so X, Y are midpoints of arcs AC, BD of ω =⇒ W, I1 , X, W, I2 , Y . By homothety so


P Q//XY . We have
√ s r
XI1 XA XP · XR XP XR P Q//BC XR 2 XR
= = √ = · = ( ) =
Y I2 YD YQ·YR YQ YR YR YR

+
=⇒ △RI1 X ∼ △RI2 Y and by spiral homothety so W lie on (RI1 I2 ). And we are done.

Back to the problem. Notice that T, D, I2 , I lie on a circle,use the lemma for quadrilateral
AF CE so there exists a circle α tangent to (ABC) at T and tangent to EF, AC, we can
easily prove the uniqueness of this circle so Ω ≡ the A−mixtilinear incircle of △ABC.

Q.E.D.

Problem 64: (Paramizo Dicrominique) Let △ABC be a triangle with incircle (I) tangent
to BC at D. D′ is the reflection of D over the midpoint of BC. O is the center of (ABC)
and AO intersects the line from D′ perpendicular to BC at P .The reflection of line AD over
AI intersects (ABC) at Q. Prove that: AD′ P Q is cyclic.

Solution:

Diagram M.Di.117

Let AD cut (ABC) again at S, let the A−mixtilinear incircle to touch (ABC) at X. AD′
cut (ABC) at R so it is well known that RX//BC. Let QX cut BC at Z. We have

ZB XB QB AB DB SC AB DC DC AB AB 2
= · = · · = · · =
ZC XC QC AC DC SB AC DB DB AC AC 2
Olympiad geometry document for reference 138

(See comments of Theorem M.T.5 and Theorem M.T.6)


From the ratio =⇒ AZ is tangent to (ABC). So that ̸ ZAP = 90◦ = ̸ ZD′ P =⇒ ZAD′ P
is cyclic.
Since ̸ AQZ = ̸ AQX = ̸ ARX = ̸ AD′ B = ̸ AD′ Z so that ZAD′ Q is cyclic, so that
Z, A, D′ , P, Q lie on a circle.

Q.E.D.

Problem 65: (Tran Viet Hung) In △ABC, Be is the Bevan point and O is the circum-
center. Q is the midpoint of arc BC of (ABC) and not containing A. Point L lie on AO
such that Be L//OQ. The A−mixtilinear incircle of △ABC is tangent to (ABC) at T . AT
intersects BC at S. Prove that: IS//LQ.

Solution:

Diagram M.Di.118

Let (I) be the incircle of △ABC so O is the midpoint of Be I. Let J be the A−excenter and
JD′ ⊥ BC at D′ . Clearly Be D′ ⊥ BC and Be , D′ , L, J. Let M be the midpoint of BC and
ID ⊥ BC at D. Let IA intersects BC at W ,it is easy to see that A, I, W, Q, J.
Olympiad geometry document for reference 139


Perform an inversion I centered at A,radius AB · AC and reflects over AI. Q becomes
W ,J becomes I,D′ becomes T so L becomes R which is the intersection of (AIT ) and the
A−altitude (Since line AO is isogonal to the A−altitude). So we get the line QL becomes
the circle (ARW ). After the inversion and reflection S becomes T ′ which is the second
intersection of AD′ and (ABC). Let SI intersects (AIT ) again at Z. It is easy to see Z lie
on (BIC).
Claim: Z, R, W
Proof:

Again perform I and reflects so we get Z becomes U which is the intersection of (AJT ′ )
and JD′ .R becomes L,W becomes Q, so to prove the claim our task is to prove AU LQ
is cyclic.We have D′ U · D′ L = D′ A · D′ T = D′ B · D′ C =⇒ U lie on (JBC) or U lie on
(IBC),from here we can easily prove that U, I are symmetric over the perpendicular bisector
of BC.
We have ̸ QU J = ̸ QU D′ = ̸ QID = ̸ OQA = ̸ OAQ = ̸ LAQ =⇒ AU LQ is cyclic.
So we are done.
Back to the problem, after the inversion, to prove IS//DQ′ we will need to show (ARW ),(AU J)
are tangent or ̸ (T ′ A, T ′ J) = ̸ (RA, RW ).
We have ̸ (RA, RW ) = ̸ (RA, RZ) = ̸ (IA, IZ) = ̸ (IA, IS) = ̸ (T ′ A, T ′ J).

Q.E.D.

Comments: Combining with Problem 64 we can prove T, R, D.

Problem 66: (buratinogigle) Let △ABC be a triangle with A, B, C−mixtilinear incircles


are ωa , ωb , ωc and incircle (I). Let ψ be the triangle formed by the radical axis of ωa , ωb , ωc
with (I). Prove that: △ABC and ψ are perspective.

Solution:

Let the incircle (I) tangent to BC, CA, AB at D, E, F . Let the midpoints of segments
ID, IE, IF be U, V, T .
Let ϕX be the radical axis of the circle (I) with the X−mixtilinear circle (X is A, B, C)
From the proof of Problem 9, we can see that U lie on ϕB , ϕC ,V lie on ϕC , ϕA ,T lie on
ϕA , ϕB . So that ψ ≡ △U V T . Now our task is to prove △U V T and △ABC are perspective
which is equivalent to proving AU, BV, CT are concurrent.
Olympiad geometry document for reference 140

Diagram M.Di.119

Instead we will prove a more general problem below


Problem: (Kariya’s Theorem) Let △ABC be a triangle with incircle (I). (I) is tangent to
BC, CA, AB at D, E, F. X, Y, Z lie on ID, IE, IF such that

ID IE IF
= = =k
IX IY IZ
Prove that: AX, BY, CZ are concurrent at a point S and S lie on the Feuerbach hyperbola
of △ABC (Or we can say the isogonal conjugate point of S lie on the line OI with O is the
circumcenter of △ABC.)

Solution:
Olympiad geometry document for reference 141

Diagram M.Di.120

Instead of proving AX, BY, CZ are concurrent, we will prove the isogonal lines of them
respects to angle ̸ A, ̸ B, ̸ C are concurrent. Let the isogonal line of line AX intersects OI
and the line from O parallel to AI at T, L. From simple angle chasing we can prove that
△IAX ∼ △OLA. We have

TI AI IX ID R(I)
= = = =
TO OL OA OA · k R(ABC) · k

Similarly, if we define T ′ , T ” similarly,with the ratio above we easily get T ≡ T ′ ≡ T ”. Hence


the isogonal lines of AX, BY, CZ respects to angle ̸ A, ̸ B, ̸ C are concurrent on OI.

Q.E.D.

Problem 67: (Math-lover 123) Let △ABC be a triangle with orthocenter H,incircle (I)
tangent to BC at D. M is the midpoint of segment AH. M E intersects AI at F . Prove
that: (BHC) and (F ; F E) are tangent.

Solution:

Diagram M.Di.121

Let AX, BY, CZ be the altitudes of △ABC so clearly E lie on (AY Z). Let the incircle

of △AY Z to be tangent to Y Z at D′ . Since △AY Z ∼ △ABC so AD′ , AE are isogonal
in ̸ BAC so clearly E is the tangency point of the A−mixtilinear excircle of △AY Z with
Olympiad geometry document for reference 142

(AY Z). Since M is the center of (AY Z) so (F ; F E) is the A−mixtilinear excircle of △AY Z.

We perform an inversion centered at A,radius AH · AD. (F ; F E) becomes the incircle (I)
of △ABC. (BHC) becomes the nine-point circle of △ABC,by Feuerbach theorem, the nine-
point circle and the incircle are tangent,so by the inversion (BHC), (F ; F E) are tangent.

Q.E.D.

Problem 68: Let △ABC be a triangle with O is the circumcenter, I is the incenter. Let
K be the A−mixtilinear incircle center. Suppose the reflection I ′ of I over O lie on BC.
Prove that: K lie on BC.

Solution:

Diagram M.Di.122

It is easy to see that I ′ is the tangency point of the A−excircle with BC.Let AI cut BC at
K ′ .Let AZ ⊥ BC at Z. Let J be the A−excenter. We have −1 = (AK ′ , IJ) = (ZK ′ , DI ′ ).
Also I(N M, DO) = −1 so Z, I, N =⇒ Z, I, N, S with S is the tangency point of the
A−mixtilinear circle with (ABC). Notice that M I ′ //S, Z, I, N so SO, M I ′ intersects at L
lie on (ABC) (Since M N is the diameter of (ABC) so N LM S is a rectangle) or S, L, O.
Also M K ′ · M A = M B 2 = M I ′ · M L so AK ′ I ′ L is cyclic. We have ̸ M LS = ̸ M AS =
Olympiad geometry document for reference 143

̸ K ′ AS = ̸ K ′ AI ′ = ̸ K ′ LI ′ = ̸ M LK ′ =⇒ S, K ′ , L, O. Since K is the intersection of


OS, AI =⇒ K ≡ K ′ .

Q.E.D.

Problem 69: (buratinogigle) ⋆ Let △ABC be a triangle inscribed in circle ω. Common


tangent (other than BC) of mixtilinear incircles at the vertices B and C meet ω at M and
N . Prove that bisector of ̸ M AN goes through centroid of excentral triangle.

Solution:

Diagram M.Di.123

Let the incircle (I) to be tangent to BC at D.It is well known that I, O, G and 3GO = OI.
Let ψ be the radical axis of the B, C−mixtilinear incircles of △ABC. It is well known that
ψ passes through the midpoint K of ID, Q the midpoint of arc BC of (ABC) not containing
A. Let ψ intersects (ABC) at E different from Q. Let AG cut (ABC) again at Z. Let ψ
intersects (ABC) at E lie on arc M N not containing B, C,it is easy to show that E is the
midpoint of arc M N .
Claim: E, O, Z
Proof:
Olympiad geometry document for reference 144

Reflects G over O to get R so we will prove E, R, P instead.Let QD intersects (ABC) again


at S so it is well known that S, I, P with AP is the diameter of (ABC).
We will let readers prove this well known lemma on their own (Hint: Use sine)
Lemma: Let ω be a circle with points A. B, C, D lie on ω. X, Z lie on AB, AD and Y lie
on XZ. Suppose
AX CB YX
· =
AZ CD YZ
Then A, Y, C. and the conversed problem is also true.

Diagram M.Di.124

By the conversed of the lemma above we have


ES KD QI QI
= · =
EA KI QD QD

To prove E, R, P we need
P I ES RI 2
· = = =2
P O EA RO 1
Substitute in ,we need
P I QI PI QI P I QI
· = 2 ⇐⇒ · = 1 ⇐⇒ · =1
P O QD 2P O QD P A QD
By similar triangles,
PI QI QD
= =
PA QS QI
So we are done.
Olympiad geometry document for reference 145

Back to the problem, since E, O, Z and E is the midpoint of arc M N of (ABC) so clearly
Z is the midpoint of the other arc M N so that AG bisects ̸ M AN .

Q.E.D.

Problem 70: (Tran Viet Hung) Let △ABC be a triangle with the A−mixtilinear circle
ωa tangent to (ABC) at X. (I) is the incircle. The lines from X perpendicular to XA, XI
intersect ωa again at K, L. Prove that: The intersection of AK, IL lie on ωa .

Solution:

Diagram M.Di.125

Let XL, XK intersects (ABC) again at Q, A′ so it is easy to see that Q is the midpoint of arc
BC of (ABC),AA′ is the diameter of (ABC) .Let P be the midpoint of arcs BAC of (ABC)
so clearly AQA′ P is a rectangle. It is well known that A, I, Q, X, I, P . We have ̸ XA′ Q =
+
̸ XAQ = ̸ XAJ,̸ QXA′ = ̸ QP A′ = ̸ AA′ P = ̸ AXP = ̸ AXI =⇒ △XIA ∼ △XQA′
+
=⇒ △XIL ∼ △XAK (Spiral homothety and since ωa is tangent to (ABC) so KL//QA′
+
and △XLK ∼ △XQA′ ). Again by spiral homothety so ̸ (LI, KA) = ̸ (XL, XK) =⇒ LI
intersects AK on ωa .

Q.E.D.

Problem 71: Let △ABC be a triangle with the A−mixtilinear incircle,excircle tangent to
(ABC) at X, Y . (I) is the incircle of △ABC and AI cut BC at W . Prove that: (W XY )
and BC are tangent.
Olympiad geometry document for reference 146

Solution:

Diagram M.Di.126


Perform an inversion centered at A,with radius AB · AC then reflects over AI, X, Y be-
comes D, D′ where D is (I) touches BC and D′ is the A−excircle (J) touch BC. W becomes
M which is the midpoint of arc BC of (ABC). (W XY ) becomes (M DD′ ) and BC becomes
(ABC).It is easy to show that (M DD′ ) and (ABC) are tangent by symmetric so by the
inversion and reflection we get the problem.

Q.E.D.

Problem 72: Let △ABC be a triangle with A−excircle tangent to AC, AB at E, F . Sup-
pose EF intersects (ABC) at distinct points Y, X such that Y lie between X, F . The
A−bisector of △ABC cut BC at D.(F DY ), (EDX) intersects (ABC) again at U, V . Prove
that: The circumcenter of △DU V lie on AD.

Solution:

Perform an inversion over (A; AE), E, F becomes E, F ,B, C becomes B ′ , C ′ .D becomes D′


which is the midpoint of arc B ′ C ′ of (AB ′ C ′ ). BC becomes (AB ′ C ′ ).(J) becomes (J) which
is the A−mixtilinear incircle of △AN ′ C ′ .Y, X becomes the intersection of (AEF ) with B ′ C ′
which are Y ′ , X ′ . U, V becomes the second intersections of (D′ Y ′ F ), (D′ X ′ E) with BC which
are U ′ , V ′ . Our task is to prove the center of (D′ U ′ V ′ ) lie on AD′ .
Olympiad geometry document for reference 147

Diagram M.Di.127

Suppose AB ′ > AC ′ and we will solve based on our diagram’s condition. Let O be the
center of (D′ U ′ V ′ ),we have ̸ OD′ U ′ = 90◦ − ̸ D′ V ′ U ′ = 90◦ − ̸ X ′ V ′ D′ = 90◦ − ̸ X ′ ED′ =
90◦ − ̸ D′ EF + ̸ X ′ EF . Also ̸ JD′ U ′ = ̸ F D′ U ′ + ̸ F D′ J = ̸ F Y ′ U ′ + 90◦ − ̸ D′ F E =
̸ X ′ EF + 90◦ − ̸ D′ EF = ̸ OD′ U ′ , so O lie on JD′ .

Q.E.D.

Problem 73: (buratinogigle) Let △ABC be a triangle inscribed in circle (O). (J) is A-
mixtilinear excircle that is tangent to (O) at D . Circle diameter AJ meets line BC at E,
F . Circle (DEF ) meets (J) again at P . Prove that JP ⊥ BC and (DEF ) passes through
A1 which is the intersection of the A−symmedian line with (ABC).

Solution:

Suppose AB < AC.



Perform an inversion centered at A,radius AB · AC then reflects over AI. D becomes X
which is the touch point of incircle (I) with BC. A1 becomes midpoint M of segment BC.
Let the incircle (I) to be tangent to AC, AB at Y, Z and let Y Z intersects (ABC) at K, L
where Z lie between K, Y so (DEF ) becomes (KXL). Let Y Z cut BC at S, it is well known
that (SX, BC) = −1 so by Maclaurin we get SX · SM = SB · SC = SK · SL =⇒ (LKX)
passes through M.So by the inversion we get (DEF ) passes through A1 .
The line JP will become the circle (AN R) where N is the midpoint of Y Z, R is the second
Olympiad geometry document for reference 148

intersection of (XKL) and (I). Since BC becomes (ABC) so our task is to prove (AN R) is
orthogonal to (ABC).

Diagram M.Di.128

We will ignore the old point J which is the A−excenter and focus on the inversed problem.T
is the tangency point of the A−mixtilinear incircle with (ABC).Let BI, CI cut (ABC) at
V, U and W is the midpoint of U V . Let AT cut (I) at R′ closer to T .It is well known that
R′ , X are reflections of each other over T I and also it is easy to prove that K, L, R, X, M lie
on a circle with center W so R ≡ R′ (We can easily prove it can not be the other intersection
J mentioned below).
Let J be the other intersection of AT and (I). By Example M.E.5 so DJ ⊥ EF . So it is
a well known lemma for △XY Z that R, N, X” with XX” is the diameter of (I).
Let X ′ be the tangency point of the A−excircle with BC.Let O′ be the center of (AN R),AO′
intersects Y Z at T ′ so since ̸ AN T ′ = 90◦ so AT ′ is the diameter of (AN R). To prove the
orthogonal property,we will need to show AT is tangent to (ABC).
We have ̸ T ′ AB = ̸ T ′ AR − ̸ BAT = ̸ T ′ N R − ̸ CAX ′ = ̸ JX”N − ̸ CAX ′ = ̸ X”N Y −
̸ CAX ′ . Since it is well known that △X”ZY ∼ △ICB so ̸ X”N Y − ̸ CAX ′ = ̸ IM B −
Olympiad geometry document for reference 149

̸ CAX ′ = ̸ AX ′ B − ̸ CAX ′ = ̸ ACB (Since IM//A, X ′ , X”).


Hence AT is tangent to (ABC).

Q.E.D.

Problem 74: (buratinogigle) Let ABC be a triangle with mixtilinear incircles respectively
are (Oa ), (Ob ), (Oc ). (Oa ) cuts BC at A1 , A2 such that A1 is between B, A2 . (Ob ) cuts CA at
B1 , B2 such that B1 is between C, B2 . (Oc ) cuts AB at C1 , C2 such that C1 is between A, C2 .
Prove that A2 B1 , B2 C1 , C2 A1 intersect base a triangle that is perspective with triangle ABC.

Solution: (Luis Gonzales)

Diagram M.Di.129

Let (Oa ) touch AC, AB at Y, Z and Y Z cuts BC at A0 . B0 and C0 are defined similarly. By
Menelaus’ theorem we get

r
A0 B Y A BZ BZ BA1 · BA2
= · = = .
A0 C AZ CY CY CA1 · CA2

Now, multiplying the cyclic expressions together, since A0 , B0 , C0 are collinear on the or-
thopolar of the incenter I.
Olympiad geometry document for reference 150

We deduce by Carnot’s theorem that A1 , A2 , B1 , B2 , C1 , C2 lie on a same conic. Hence by


Pascal theorem, intersections BC ∩ B2 C1 , CA ∩ C2 A1 and AB ∩ A2 B1 are collinear and the
conclusion follows.

Q.E.D.

Problem 75: Let △ABC be a triangle with altitudes AD, BE, CF are concurrent at H.
Tangent lines at B, C to (ABC) intersects at T . T B, T C intersects EF at X, Y . M is the
midpoint of BC.

i) Prove that: M H, EF, T D are concurrent at a point called Z.

ii) Prove that: The projection of Z on BC called P lie on T V with V is the second
intersection of (AH), (ABC).

Solution:

Diagram M.Di.130

i) By simple angle chasing,we can prove that △F HD, △XM T are homothetic.
Hence M H, EF, T D are concurrent.
Olympiad geometry document for reference 151

ii) By Example M.E.1 we get that V is the tangency point of (ABC) and (XY T ) and also
(XY T ) is the T −mixtilinear incircle of △T XY . Now by Theorem M.T.12 we are done.

Q.E.D.

Problem 76: Let △ABC be a triangle with A−mixtilinear incircle tangent to AB, AC,
and (ABC) at E, F, X.(I) is the incircle of △ABC and (A; AE) intersects AO at S closer to
O with O is the center of (ABC). (AIS) intersects (ABC) at R. (A; AE) intersects (ASX)
again at N .Prove that: N, A, R.

Solution:

Diagram M.Di.131


Perform an inversion centered at A with radius AB · AC then reflects over AI. I becomes
J which is the A−excenter, the A−mixtilinear incircle becomes A−excircle (J).X becomes
D′ where (J) touch BC. (A; AE) becomes (A; AE ′ ) with E, F becomes E ′ , F ′ where they
are the touch points of (J) and AB, AC. Line AO becomes the A−altitude, so S becomes S ′
which is the intersection of the A−altitude with (A; AE ′ ).(ABC) becomes BC.R becomes R′
where JS ′ cut BC. (ASX) becomes S ′ D′ so N becomes N ′ where S ′ D′ cut again (A; AE ′ ).
Our task now is to prove N ′ , A, R′ . We will change the configuration from excircle to incircle
so it is easier to solve.We will prove the following problem (This is called an intraversion)
Olympiad geometry document for reference 152

Problem: Let △ABC be a triangle with incircle (I) tangent to BC, CA, AB at D, E, F.
AH ⊥ BC at H. (A; AE) intersects the opposite ray of ray AH at W ,DW intersects (A; AE)
at J. Prove that: A, J, R where W I cut BC at R.
Solution:

Diagram M.Di.132

Let W I cut (A; AE) again at V . We have ̸ V JW = ̸ V W x (W x is the tangent ray to


(A; AE)) = ̸ W RD =⇒ JV RD is cyclic =⇒ ̸ DV R = ̸ DJR.
2
Since IV · IW = R(I) = ID2 so ̸ DV I = ̸ W DI = ̸ DW H = ̸ AW J = ̸ AJW , but
̸ DV I = ̸ DJR so ̸ AJW = ̸ DJR =⇒ A, J, R.

Q.E.D.

Problem 77: (buratinogigle) Let △ABC be a triangle inscribed in circle (O) and A-
mixtilinear excircle (J). Two common external tangent of (O) and (J) touches (O) at M, N .
Prove that M N is tangent to A-mixtilinear incircle of ABC.

Solution:

Let the common tangent lines of (ABC) and the A−mixtilinear excircle to intersect at T .
Let U be the center of the A−mixtilinear excircle, T U intersects (ABC) at P, Q and P
closer to T . By Monge d’Alembert theorem for (ABC), A−mixtilinear incircle,excircle we
get T, A, X. So that quadrilateral XM AN is harmonic so since P, Q are midpoints of arcs
M AN, M N of (ABC) so P X, AQ, M N are concurrent (bisector ratio). Let the concurrent
Olympiad geometry document for reference 153

point be S, if we can prove S lie on the A−mixtilinear incircle so by homothety and P being
the midpoint of arc M AN of (ABC),we get the A−mixtilinear incircle and M N are tangent.

Diagram M.Di.133


Perform an inversion centered at A,radius AB · AC and reflects over AI. X becomes

D which is the touch point of the A−excircle and BC, Q becomes D where incircle (I)
tangent to BC.P becomes R which is the intersection of the line from A perpendiculat to
AD intersects BC. P X becomes (ARD′ ),AQ becomes AD and the A−mixtilinear incircle
becomes the A−excircle. Our task is to prove the intersection of AD, (ARD′ ) lie on the
A−excircle but it is trivia by homothety.

Q.E.D.

Problem 78: Let △ABC be a triangle such that AB + AC = 3BC. The A−mixtilinear
excircle ω is tangent to AB, AC at U, V . J is the A−excenter. (AJV ) intersects ω at S and
(AJU ) intersects ω at T . Prove that: B, S, T, C lie on a circle.

Solution:

Perform an inversion centered at A,radius AB · AC then reflects over AI we get the fol-
Olympiad geometry document for reference 154

lowing problem.

Diagram M.Di.134

Problem: (IMO SL 2015) Let △ABC be a triangle with AB + AC = 3BC and incircle (I)
tangent to AC, AB at E, F . EE ′ , F F ′ are diameters of (I). Prove that: B, E ′ , F ′ , C lie on
a circle.
Solution:

Let M be the midpoint of arc BC of (ABC). By Ptolemy theorem we have

AB · M C + AC · M B = BC · AM

Since AB + AC = 3BC we can easily show that M A = 3M I. Let N be the midpoint of AI


so M I = N I = F N = JF ′ . So F ′ lie on (BIC),similarly we are done.

Q.E.D.

Problem 79: Let △ABC be a triangle with A−mixtilinear circle ω tangent to (ABC) at
X. I is the incenter and IX intersects BC at S.Z lie on AS.Suppose segments ZB, ZC
intersects ω at P, Q. Prove that: BQ, CP intersects on AS.

Solution:
Olympiad geometry document for reference 155

As we mentioned in the proof of Problem 33, AS is the polar of W with ω where EF cut
BC at W and ω is tangent to AB, AC at E, F . It is well known that (RS, BC) = −1. Let
RQ cut ω again at P ′ so (R(RQ ∩ ZS), P Q) = −1 so that Z, P ′ , B so P ≡ P ′ . Again, since
(R(P Q ∩ ZS), P Q)) = −1 = (RS, BC) so we are done.

Diagram M.Di.135

Q.E.D.

Problem 80: (PCHP) Let △ABC be a triangle with the B, C−mixtilinear incircles ωb , ωc
have centers O2 , O3 and tangent to (ABC) at Y, Z. Prove that: The intersection of ZO2 , Y O3
lie on the internal bisector of ̸ ABC.

Solution:
Olympiad geometry document for reference 156

Diagram M.Di.136

Let the A−mixtilinear incircle to be tangent to (ABC) at X,M is the midpoint of arc BC
of (ABC) so it is well known that O2 O3 , Y Z, BC, XM are concurrent at a point called S.
Let I be the incenter of △ABC, we will prove △IO3 O2 and △AY Z are perspective,with
the idea of using Dersagues theorem we will need to prove S, R, P are collinear where IO3
or CI cut AY at R and IO2 or BI cut AZ at P . As mentioned in the proof of Problem
21 we known that C, X, P , R, B, X. It is well known that AX, BY, CZ are concurrent.Now
by Dersagues theorem for perspective triangles △XBC, △AY Z so R, S, P and we are done.

Q.E.D.

Problem 81: (Luis Gonzales) Let △ABC be a triangle with the A−mixtilinear circle ω
tangent to AB, AC, (ABC) at E, F, X.CW is tangent to ω at W and line CW is different
from line CA. AW intersects CE at R. M is the midpoint of segment AC and XF intersects
EW at L. Prove that: L, R, M .

Solution:

Diagram M.Di.137

Let AB intersects CW at S so it is easy to see that R is the Gergonne point of △SAC. We


will rewrite the problem like below.
Problem: (Luis Gonzales) Let △ABC be a triangle with incircle (I) tangent to BC, CA, AB
at D, E, F. A circle passes through B, C and tangent to (I) at S.DS intersects EF at P ,BE
intersects CF at Ge ,M is the midpoint of BC. Prove that: P, Ge , M .
Olympiad geometry document for reference 157

Solution:

Let J be the A−excenter of △ABC. We have a well known claim from IMO SL 2002 like
below.
Claim: S, D, J
Solution:

Diagram M.Di.138

Let Z be the midpoint of segment BC and (J) is the A−excircle which is tangent to BC at
D′ . Let IB cut DF at N so clearly N is the midpoint of DF . Let M be the midpoint of
JD so we have M D = M D′ ,but it is well known that BF = D′ C so it is easy to see that
M B = M C or M lie on the perpendicular bisector of BC.By simple angle chasing, we can
get △SF D ∼ △BDJ =⇒ △F SN ∼ △DBM =⇒ ̸ M BD = ̸ F SN . But since SB i
s the symmedian line of △SF D so ̸ F SN = ̸ BSD =⇒ ̸ M CB = ̸ M BC = ̸ F SN =
̸ BSD.Similarly ̸ CSD = ̸ M CB = ̸ M BC so SD bisects ̸ BSC,since M B = M C and
(I) tangent to (BSC) we can prove that S, D, M and B, M, C, S lie on a circle (Readers
can prove this by let SD cut (BSC) at M ′ then show M ≡ M ′ or let W be the center of
(BSC)). Since S, D, M so S, D, J and we are done.

Back to the problem.


Olympiad geometry document for reference 158

To prove P, Ge , M we need this ratio


sin(BGe M ) Ge C need P E Ge F PE Ge C Ge E
= = ⇐⇒ =
sin(CGe M ) Ge B P F Ge E PF Ge B Ge F

Diagram M.Di.139

We get S, P, D, J as proved above ,also from ((EF ∩ BC)D, BC) = −1 and SD bisects
̸ BSC so EF, BC, SD′ are concurrent at a point ,call it L where DD′ is the diameter of (I).
It is easy to see DF//JK, DE//JN
PK PN
=⇒ P F = P D; P E = PD
PJ PJ
PE P N (LP,KL)=J(LP,KL)=(LD,BC)=−1 LN M enelaus CN BJ
=⇒ = = =
PF PK LK CJ BK
Also
sin(KF B) anglechasing sin(AF E)
BK = BF = BF
sin(F KB) sin(JCB)
Similarly
sin(AEF ) CN BJ JB 2 CE
CN = CE =⇒ =
sin(JBC) CJ BK JC 2 BF
After finished this ratio ,now we will be back with the other ratio
Ge C Ge E Ge C Ge E M enelaus EC BA F A CE EC 2 AB
= = =( )
Ge B Ge F Ge F Ge B EA BF F B CA BF AC
Olympiad geometry document for reference 159

It is now left to prove


JB 2 AB CE
= =1
JC 2 AC BF
Which is true by the Steiner equation for △JBC with J−altitude and JI are isogonal in
̸ BJC (Notice the bisector ratio of ̸ BAC on BC). Hence proven.

Q.E.D.

Problem 82: (Lin yangyuan) Let △ABC be a triangle with A, B, C−mixtilinear incircles
centered at O1 , O2 , O3 and tangent to (ABC) at D, E, F.

i) (O3 ) is tangent to BC at T. Prove that: ̸ T DO3 = ̸ EDO2 .

ii) DO3 cut (O3 ) at P further to D,define Q similarly. Prove that: P Q//O2 O3

iii) BO3 cut CO2 at A′ . Prove that: A′ lie on the internal bisector of ̸ BT C.

Solution:

Diagram M.Di.140

i) It is well known that DO3 , DO2 are isogonal in ̸ BDC (See Theorem M.T.14) so our
+
task is to prove ̸ BDT = ̸ CDE or to prove △DT B ∼ △DCE. It is well known that
Olympiad geometry document for reference 160

EF, BC, DM are concurrent with M being the midpoint of arc BC.Let the B−mixtilinear
incircle to touch BC at K.Since ̸ BIT = ̸ BAI = ̸ CAI = ̸ CIK and it is well known that
SI is tangent to (BIC) so SI is tangent to (IT K). It is well known that ̸ SIM = ̸ SDI =
90◦ so SD · SM = SI 2 = ST · SK =⇒ T DM K is cyclic, so ̸ DT B = ̸ DM K = ̸ DCE
+
since ̸ DBT = ̸ DBC = ̸ DEC =⇒ △DT B ∼ △DCE so we are done.

ii) Let P ′ , Q′ be the other intersection of DP with (O2 ), DQ with O3 . Let I be an inversion

centered at D,radius DI or DB · DC.It is easy to see that P ′ becomes Q,Q′ becomes P so
DP ′ · DQ = DP · DQ′ so P Q//P ′ Q′ . Since O2 , O3 are midpoints of QQ′ , P P ′ so O2 O3 //P Q
and we are done.

iii) It is easy to see that O3 , I, C, O2 , I, B. Notice that we have an involution swaps DB, DC
and swaps DO3 , DO2 and swaps DI, DA′ but it is well known that DB, DC; DO3 , DO2 are
pairs of reflection lines over XI so A′ lie on XI. And we are done.

Q.E.D.

Problem 83: (Jean-Louis Ayme) Let △ABC be a triangle with the A−mixtilinear incircle
ω tangent to (ABC) at X. ω is tangent to AB, AC at E, F. I is the incenter of △ABC. Let
the perpendicular line from X to XA intersects ω again at L.LI intersects ω again at R.AX
intersects ω again at S. Prove that: SR, EF, BC are concurrent.

Solution:

Diagram M.Di.141
Olympiad geometry document for reference 161

Let the line from X perpendicular to XI intersects ω again at W .From Problem 70 so


A, R, W .Let M be the midpoint of arc BC of (ABC) so it is well known that T, X, W, M .
We have
TE XE W E SE RE
= · = ·
TF XF W F SF RF
(Since quadrilaterals SEXF, REW F are harmonic) From the ratio we get T, R, S.

Q.E.D.

Problem 84: (Lin yangyuan) Let △ABC be a triangle with the A−mixtilinear circle ω
tangent to (ABC) at X. XI intersects BC at S,R lie on BC such that AR//IX. L different
from X lie on ω such that (BLC) is internally tangent to ω. Prove that: M, L, S with M
being the midpoint of AR.

Solution:

Diagram M.Di.142

Let ω tangent to AB, AC at E, F. N is the midpoint of arc BC of (ABC). It is well known


that EF, BC, XN are concurrent and XI bisects ̸ BXC. By radical axis theorem,the tangent
lines from L to ω,X to ω intersects at V lie on BC. Since it is easy to see that V is the
midpoint of W S so V is the center of (SLW ) also so ̸ W LS = 90◦ . It is well known that
(W S, BC) = −1 so LS bisects ̸ BLC. As mentioned in the proof of Problem 33, AS is
the polar of W with ω so that W J ⊥ AS at H where J is the center of ω.We can easily
show H lie on (W LSX). It is well known that ̸ AHJ = 90◦ = ̸ AIW so JH · JW =
Olympiad geometry document for reference 162

JI · JA = Rω = JL2 = JX 2 so JL, JX are tangent to (W LSX) so (W H, LX) = −1. We


have −1 = (W H, LX) = S(W H, LX) = S(RA, LI), since S(RA, M I) = −1 so S, L, M .

Q.E.D.

Problem 85: (Dsoong) Let △ABC be a triangle with A−mixtilinear incircle with center
J tangent to (ABC) at X. M is the midpoint of segment BC and (I) is the incircle of
△ABC. (I) is tangent to BC at D. Prove that: The line from D parallel to IM ,the line
from J perpendicular to BC and XM are concurrent.

Solution:

Diagram M.Di.143

Let A′ be the reflection of A over I,O is the circumcenter of △ABC. Let DD′ be the
diameter of (I) so it is well known that AD′ //IM but since DA′ //D′ A so DA′ //IM . Let
OI cut BC at K, by Example M.E.19 so X, A′ , K. Let DA′ cut XM at R.
Apply Dersagues theorem for perspective triangles △IDA′ , △OM X and since ID, OM ⊥
BC so JR ⊥ BC.

Q.E.D.

Problem 86: (Wizard Math & buratinogigle) Let △ABC be a scalene triangle with the
A−mixtilinear circle to be tangent to (ABC), AB, AC at X, P, Q. Incircle (I) is tangent to
AC, AB at E, F. EF, P Q cut BC at S, T. Let d be the perpendicular bisector of BC,let d
cut arc BAC of (ABC) at M . Z is a point lie on d.AZ intersects (ABC) at L. Prove that:
LX, ZT, M S are concurrent.
Olympiad geometry document for reference 163

Solution:

Consider Z moving on d and fix the other points not link to Z.


Consider the map f1 : Z −→ ZT −→ W1 is projective.
Consider the map f2 : Z −→ AZ −→ L −→ LX −→ W2 is projective.
We will verify that W1 ≡ W2 through 3 cases.

Diagram M.Di.144

Case 1: Z ≡ L ≡ A1 where A1 is the midpoint of arc BC not containing A of (ABC). It is


well known that A1 , X, T so we are done.
Case 2: Z ≡ A2 where A2 is the midpoint of BC. In this case
LB A2 B AC AC
= · =
LC A2 C AB AB
By Menelaus we can easily prove that
WB BF DB
= =
WC CE DC
Also
XB AB DB
= ·
XC AC DC
(As shown in the comments of Theorem M.T.5 and Theorem M.T.6)
So that
XB LC W B
= · =⇒ W, X, L
XC LB W C
And in this case we are done.
Case 3: Z ≡ O with O is the center of (ABC).
Olympiad geometry document for reference 164

Diagram M.Di.145

Let M Q cut XL at W .Let (AEF ) intersects (ABC) again at Q,it is well known that in
this case Q, I, L and S, Q, M . It is well known that IT is tangent to (BIC) so by radical
axis theorem for (BIC), (AEF ), (ABC) we get T, Q, A. Let N be the midpoint of arc BC
of (ABC) not containing A so it is well known that M, O, N , T, X, N so by Pascal for
 
N, Q, L
A, X, M

We get W, T, O.
So W1 ≡ W2 .

Q.E.D.

Problem 87: (Jean-Louis Ayme) Let △ABC be a triangle with the A−mixtilinear incircle
ω tangent to (ABC), AB, AC at X, E, F. The line from X perpendicular to XA intersects
ω again at L. I is the incenter of △ABC,(IXL) intersects AX again at P , ω intersects AX
again at R. Prove that: RI//P K where AI cut BC at K.

Solution:

It is well known that I is the midpoint of EF .M is the midpoint of arc BAC of (ABC) so
it is well known that M, I, X.Let RL intersects EF at S. AA′ is the diameter of (ABC) so
X, L, A′ .Since ω, (ABC) are tangent so by homothety AA′ //R, J, L with J is the center of
ω. We have ̸ (EF, RJ) = ̸ (EF, AA′ ) = ̸ (AM, AA′ ) = ̸ (XM, XA′ ) = ̸ (XM, XL) =⇒ S
Olympiad geometry document for reference 165

lie on (IXL). Let AS intersects ω at G. By Reim for SI//AM so G lie on (ABC).

Diagram M.Di.146

Claim: G, K, M
Proof:

XI intersects ω again at K so it is well known that the tangent from K to ω is parallel to


BC.Let M ′ be the midpoint of BC, by homothety for the well known lemma (incircle (I)
tangent to BC, CA, AB at D′ , E ′ , F ′ and ID′ cut E ′ F ′ at A1 so AA1 is the A−median line)
we get KT, AM ′ , EF are concurrent at Q with KT is the diameter of ω. From Problem
70 we get that AL, T I intersects at W lie on ω. We have
GB sin(GAB) sin(SAE) SE AF SE
= = = · =
GC sin(GAC) sin(SAF ) SF AE SF
We have
SE RE LE XE W E
= · = · =ϕ
SF RF LF XF W F
(Since quadrilaterals REXF, W ELF are harmonic)
IE KF W E KF IF W E KF T F QF sin(QAF ) AF
ϕ= · · = · · = · = = ·
IF KE W F KE IE W F KE T E QE sin(QAE) AE
Olympiad geometry document for reference 166

sin(M ′ AC) M ′ C AB AB
= = · =
sin(M ′ AB) M ′ B AC AC
This shows
GB AB
=
GC AC
so ABGC is a harmonic quadrilateral,therefore G, K, M .
Back to the problem.Since AI · AJ = AE 2 = AR · AX =⇒ RIJX is cyclic so that
̸ AGK = ̸ AXM = ̸ RXI = ̸ RJI = ̸ SJI =⇒ SJKG is cyclic. So that AJ · AK =
AS · AG = AP · AX =⇒ P XKJ is cyclic. Since ̸ P KJ = ̸ P XJ = ̸ RIA =⇒ IR//P K.

Q.E.D.

Problem 88: (buratinogigle) ⋆ Let ABC be a triangle inscribed in circle (O) with A-
excircle (J). Circle passing through A, B touches (J) at M . Circle passing through A, C
touches (J) at N . BM cuts CN at P . Prove that: AP passes through tangent point of
A-mixtilinear incircle with (O).

Solution: (buratinogigle)

Perform an inversion I, centered at A, radius AB · AC then reflects over AI where I is
the incenter and we get the following problem.
Problem: Let ABC be a triangle inscribed in circle (O). Circle (K) touches CA, AB
and (O) internally. Draw tangent line CM, BN to (K) with M, N lie on (K). Circles
(ABN ), (ACM ) intersect again at P . Prove that: AP passes through the tangency point of
the A−excircle with BC.

We will prove those lemmas first.


Lemma 1: Let ABC be a triangle with incircle (I) touches BC at D. Circle passing through
A, B touches (I) at M . Circle passing through A, C touches (I) at N . BM cuts CN at P .
Prove that ̸ P AB = ̸ DAC.

Proof:

Let incircle (I) touches CA, AB at E, F and X, Y, Z is cevian triangle of P . Let BY cuts
DF at K. We have

YC [Y BC] [Y BC] [KBD] [KBF ] BC.BY KD BF.BK


= = . . = . .
YA [Y BA] [KBD] [KBF ] [Y BA] BD.BK KF BY.BA
Olympiad geometry document for reference 167

2 2
BC M D2 BC R(I) .DF
= = .
BA M F 2 BA EF 2 .IA2
.

Diagram M.Di.147

Similarly,
2 2
ZB BC R(I) .DE
= .
ZA CA EF 2 .IA2
.

By Ceva’s theorem then


XB Y A ZB AB.DE 2
= . =
XC Y C ZA AC.DF 2
.

So
DB XB p − b AB.DE 2 DF.IB AB.DE 2 AB 2
. = . = . =
DC XC p − c AC.DF 2 DE.IC AC.DF 2 AC 2
Thus AD, AX are isogonal.
(Note a = BC, b = CA, c = AB, 2p = a + b + c)

Lemma 2: Let ABC be a triangle and incircle (I, r) touches BC, CA, AB at D, E, F , resp.
Circle passes through B, C and touches (I) at X then

XE.XF r2
i) = .
XD2 IB.IC
Olympiad geometry document for reference 168

XE IB.DE 2
ii) =
XF IC.DF 2

Leading to the results:


XE 2 r2 .DE 2 XF 2 r2 .DF 2
= ; = .
XD2 IB 2 .DF 2 XD2 IC 2 .DE 2

Proof:

Diagram M.Di.148

As mentioned in the proof of Problem 81, we have EF, XK, BC are concurrent with DK
is the diameter of incircle (I). We have
XE KE GE HE [AED] DE DB IB
. = = = = . .
XF KF GF HF [AF D] DF IC DC
and note that
KE.IC = 2r2 = KF.IB

So
XE IB.DE 2
= .
XF IC.DF 2

And
F X EX GX GD2 XD2
. = = =
F K EK GK GK 2 4r2
and note that
KE.IC = 2r2 = KF.IB
Olympiad geometry document for reference 169

So
XE.XF r2
= .
XD2 IB.IC

Back to the problem.

Diagram M.Di.149

Let (ABN ) cuts CA again at Y and (ACM ) cuts AB again at Z. AP cuts (O) again at
XB ZB
X. We easily see that = . But by inversion and the lemmas in front, we can
XC YC
ZB DE 2 XB DE 2
prove = . So = . From this, we can prove that AP passes through the
YC DF 2 XC DF 2
tangency point of the A−excircle with BC.

Q.E.D.

Problem 89: Let △ABC be a triangle with distinct U, V lie on (ABC) such that AU =
AV. Let ω be the A−mixtilinear circle,ω is tangent to (ABC) at X, AX intersects ω again
at Y.Prove that: U V is parallel to the tangent from Y to ω.

Solution:

Let O be the circumcenter of △ABC then since OU = OV, AU = AV so AO ⊥ U V. Let ω


be tangent to AB, AC at E, F. Let (A; AE) intersects (ABC) at P, Q. Perform an inversion
over (A; AE), since (ABC) becomes P Q, ω becomes ω, we can easily prove P Q is tangent
to (ABC) at Y which is the image of X. Since OP = OQ, AP = AQ so P Q ⊥ OA, hence
U V //P Q.
Olympiad geometry document for reference 170

Diagram M.Di.150

Q.E.D.

Problem 90: (纯几何吧) Let △ABC be a triangle with incircle (I) tangent to AB, AC
at F, E and BC at D. ID intersects EF at L. N is the midpoint of arc BAC of (ABC).
AG is the diameter of (ABC).The A−mixtilinear circle ω is tangent to AB, AC at P, Q
and (AP Q) intersects (ABC) again at R. RG cut IN at T.Prove that: T, L are isogonal
conjugate in △ABC.

Solution:

Let ω touch (ABC) at X so it is well known that X, I, N .Perform an inversion centered



at A, radius AB · AC then reflects over AI. It is well known that R, J, G with J is the
center of ω. ω will become A−excircle (Ia ) tangent to AB, AC at V, U so R becomes the
intersection of U V with BC which is S (Since (ABC) becomes BC and (AP Q) becomes
U V ). G becomes H where AH ⊥ BC at H, so line RG becomes (AHS). X becomes D′
where A−excircle (Ia ) touch BC and I becomes Ia so that the line XI becomes the circle
(AD′ Ia ). Hence T becomes R which is the intersection of (AD′ Ia ) with (AHS).In order to
prove that T, L are isogonal conjugate in △ABC we will need to show A, L, R and (BLC)
passes through R.
Olympiad geometry document for reference 171

Diagram M.Di.151

We will rewrite the problem like below.


Problem: Let △ABC be a triangle with incircle (I) tangent to BC, CA, AB at D, E, F.
ID intersects EF at L. AH ⊥ BC at H. A−excircle (Ia ) is tangent to AB, AC at U, V and
BC at D′ . U V cut BC at S. (AHS) intersects (AD′ Ia ) at R. Prove that: A, L, R and R lie
on (BLC).
Solution:

Let M be the midpoint of segment BC, it is well known that A, L, M . By intraversion,or we


can simply say the excenter version of the well known lemma we mentioned, AM, U V, AD′
are concurrent at a point called K. It is easy to prove that S lie on (Ia N D′ ).By radical axis
theorem for (ABC), (Ia N D′ S), (AD′ Ia ) so AR, N S ≡ U V, Ia D′ at K. Hence A, M, K, R.
Let the external bisector of ̸ BAC to cut BC at W . It is easy to show that W lie on
(AD′ RIa ) and also it is easy to see that Z : //W A so by Reim =⇒ ZLD′ R is cyclic. We
have M L · M R = M D′ · M Z = M D · M Z = M B 2 (By Maclaurin and (ZD, BC) = −1)
= M B · M C so that LBRC is cyclic,
Olympiad geometry document for reference 172

Diagram M.Di.152

Q.E.D.

Problem 91: Let △ABC be a triangle with A, B, C−mixtilinear incircles be ωa , ωb , ωc .


Suppose ωa intersects BC at distinct points Ba , Ca . Tangents from Ba , Ca to ω intersects at
A′ . Define Ab , Ac , Cb , Ca and B ′ , C ′ similarly. Prove that: AA′ , BB ′ , CC ′ are concurrent at
point L′ which is the isogonal conjugate of L wrt. △ABC, where L is the symmedian point
of the excentral triangle of △ABC.

Solution:

Let △Ia Ib Ic be the excenteral triangle of △ABC.


Let M be the midpoint of Ib Ic then it is well known that M is the midpoint of arc BAC
of (ABC). Let ω tangent to (ABC) at X. It is well known that X, I, M . Let XI intersects
BC at V, EF cut BC at Q where ω is tangent to AB, AC at E, F. As mentioned in the
proof of Problem 33, AV is the polar of Q to ω. Let XI intersects ω again at W , it is well
known that the tangent from W to ω is parallel to BC ≡ Ba Ca so W is the midpoint of arc
Ba Ca of ω, so that W X bisects ̸ BA XCa , it is well known that ̸ QXV = 90◦ (Since QX
passes through the midpoint of arc BC not containing A of (ABC)) so (QV, Ba Ca ) = −1.
Olympiad geometry document for reference 173

Combining the face AV is the polar of Q to ω and (QV, Ba Ca ) = −1 so A, V, A′ . Let the


tangent lines from Ic , Ib to (Ia Ib Ic ) intersects at S, by Problem 75, we get S, A, V (Note
that I is the orthocenter of △Ia Ib Ic ). Let M Ia intersects BC at R,if we can prove A, L, R
and then AV, AR are isogonal conjugate, clearly the problem is done by AA′ , BB ′ , CC ′ is
concurrent.

Diagram M.Di.153

Now we will complete the left part.Change the configuration to orthocenter, we will prove
the problem below.
Problem: Let △ABC be a triangle with altitudes AD, BE, CF concur at H. Tangent lines
from B, C to (ABC) intersects at T. M is the midpoint of BC. AM intersects EF at P . As
mentioned above, HM, T D, EF are concurrent at S. L is the symmedian point of △ABC.
Prove that: D, L, P and DS, DP are isogonal in ̸ EDF .
Solution:
Olympiad geometry document for reference 174

Diagram M.Di.154

Let AT cut BC at K, N is the midpoint of EF . It well known that A, N, K, T . Since


△AEF ∼ △ABC so ̸ AM K = ̸ AM B = ̸ AN E = ̸ AN P =⇒ N P M K is cyclic
=⇒ ̸ M KP = ̸ M N P = ̸ M N E = 90◦ , also it is well known that L lie on the A−Schwatt
line RM with R is the midpoint of AD ,combine with P K//AD so D, L, P . Let EF intersects
BC at W, AH intersects EF at V . So that −1 = (AH, V D) = M (AH, V D) = (P S, V W )
=⇒ DS, DP are isogonal in ̸ EDF (It is well known that DA bisects ̸ EDF .)

Q.E.D.

Problem 92: ⋆ ⋆ In △ABC with circumcenter O .Let X be radical center of three mix-
tilinear incircles and Y is the radical center of three mixtilinear excircles .Prove that: O is
the midpoint of XY .

Solution: (Telv Cohl)

Let I be the incenter of △ABC and △DEF be the intouch triangle of △ABC .
Let Ia , Ib , Ic be the A-excenter, B-excenter, C-excenter of △ABC, respectively .
Let ωA , ωB , ωC be the A-mixtilinear incircle, B-mixtilinear incircle, C-mixtilinear incircle of
△ABC, respectively .
Let ΩA , ΩB , ΩC be the A-mixtilinear excircle, B-mixtilinear excircle, C-mixtilinear excircle
of △ABC, respectively .
Let △A1 B1 C1 be the anti-complementary triangle of △Ia Ib Ic and △A2 B2 C2 be the mic arc
Olympiad geometry document for reference 175

triangle of △ABC .
Let A′2 , B2′ , C2′ be the antipode of A2 , B2 , C2 with (O), respectively and A3 , B3 , C3 be the
midpoint of A1 D, B1 E, C1 F , respectively.
Let Ab ≡ ωA ∩ AB, Ac ≡ ωA ∩ AC, A′b ≡ ΩA ∩ AB, A′c ≡ ΩA ∩ AC
(We define Ba , Bc , Ca , Cb , Ba′ , Bc′ , Ca′ , Cb′ similarly) A4 ≡ ωA ∩ ⊙(O), B4 ≡ ωB ∩ ⊙(O), C4 ≡
ωC ∩ ⊙(O), A5 ≡ ΩA ∩ ⊙(O), B5 ≡ ΩB ∩ ⊙(O), C5 ≡ ΩC ∩ ⊙(O).

Diagram M.Di.155

Since the midline of isosceles trapezoid F DBc Ba , DECa Cb is the radical axis of (I), ωB and
(I), ωC , respectively , so the radical center of (I), ωB , ωC is the midpoint of ID =⇒ the
midpoint D′ of ID lie on the radical axis of ωB , ωC , hence combine A2 ≡ B4 Bc ∩ C4 Cb and
Olympiad geometry document for reference 176

A2 Bc · A2 B4 = A2 Cb · A2 C4 =⇒ A2 D′ is the radical axis of ωB , ωC , so the radical axis of


ωB , ωC passes through the midpoint of IT where T is the homothetic center of △DEF and
△Ia Ib Ic .

Similarly, the radical axis of ωC , ωA passes through the midpoint of IT =⇒ X is the midpoint
of IT (X999 in ETC.)(⋆)

Diagram M.Di.156

Since the midline of F DBc′ Ba′ , DECa′ Cb′ is the radical axis of ⊙(I), ΩB ; (I), ΩC , resp , so A3
is the radical center of (I), ΩB , ΩC =⇒ A3 lie on the radical axis of ΩB and ΩC , hence
combine A′2 ≡ B5 Bc′ ∩ C5 Cb′ and A′2 Bc′ · A′2 B5 = A′2 Cb′ · A′2 C5 =⇒ A′2 A3 is the radical axis
of ΩB , ΩC , so the radical axis of ΩB , ΩC passes through the homothetic center of △A′2 B2′ C2′
and △A3 B3 C3 .

Similarly, B2′ B3 is the radical axis of ΩC , ΩA =⇒ Y is the homothetic center of △A′2 B2′ C2′
and △A3 B3 C3 .

Since I lie on the perpendicular bisector of B1 C1 , C1 A1 , A1 B1 , respectively , so I is the


Olympiad geometry document for reference 177

circumcenter of △A1 B1 C1 =⇒ I is the circumcenter of △A3 B3 C3 . (because I is the cir-


cumcenter of △DEF and A3 , B3 , C3 is the midpoint of A1 D, B1 E, C1 F , resp) Notice that
4R − r YI 4R − r
radius of (A3 B3 C3 ) is (because the radius of (A1 B1 C1 ) is 4R) =⇒ = ,
2 YO 2R
TI r XI r
so combine (⋆) =⇒ ′
= (I ′ is the reflection of I in O) =⇒ = =⇒ X, Y are
TI 2R XO 2R
symmetry wrt. O .

Q.E.D.

Problem 93: (Stanley Rabinowitz) Let △ABC be a triangle with ̸ ABC = 2̸ ACB. M is
the midpoint of arc AC not containing B of (ABC). The A−mixtilinear incircle is tangent
to (ABC) at T. Prove that: T B + T A = T M.

Solution:

Diagram M.Di.157

Let the A−mixtilinear incircle to touch AB, AC at X, Y .It is well known that T, Y, M by
BT CT AT
homothety and T X being the bisector of ̸ BT A. It is also well known that = =
BX CY AY
and = k. By Ptoleme we have

AC · T M = M A · T C + M C · T A = AB · (T A + T C)

=⇒ T M · (Y A + Y C) = AB · k · (AY + CY ) =⇒ T M = AB · k

(Since ̸ ABC = 2̸ ACB)


We have T B + T A = k(BX + AY ) = k(BX + AX) = k · AB = T C.
Olympiad geometry document for reference 178

Q.E.D.

Problem 94: Let △ABC be a scalene triangle with AB < AC and A−mixtilinear incircle
tangent to (ABC) at T. Let O1 , O2 be the circumcenters of △AT F , △AT E. Prove that:
Circumcenter O of △ABC is the midpoint of O1 O2 .

Solution:

Diagram M.Di.158

Note that we will perform the angle chasing based on our diagram .We have O1 , O, O2 since
they all lie on the perpendicular bisector of AT.
Let the A−mixtilinear incircle to touch AB, AC at E, F and let I be the incenter and also
being the midpoint of EF.
Since O1 is the center of (AT F ) and O1 O2 ⊥ AT so ̸ T O1 O2 = 180◦ − ̸ AF T = ̸ T F C =
+
̸ T EF . Similarly ̸ T O2 O1 = ̸ T F E =⇒ △T EF ∼ △T EF . We have ̸ O1 T O = ̸ O1 AO =
̸ BAO − ̸ BAT − ̸ T AO1 = ̸ BAO − ̸ BAT − 90◦ + ̸ AF T = ̸ BAO − ̸ BAT − 90◦ +
̸ AF E + ̸ T F E = ̸ BAO− ̸ BAT −90◦ + ̸ AF E + ̸ T EB = ̸ BAO−90◦ + ̸ AF E + ̸ ET A =
̸ BAO − 90◦ + ̸ AF E + ̸ F T I = ̸ BAO − 90◦ + ̸ AF E + ̸ ICA
(Since T A is the symmedian line and T IF C is cyclic.)
= −̸ ACB + ̸ AF E + ̸ ICA = ̸ IBA = ̸ IT E
(Since it is well known that T BEI is cyclic).
+
Hence ̸ ET I = ̸ O1 T O ,combine with the fact that I is the midpoint of EF and △T EF ∼
△T EF we get O is the midpoint of O! O2 .
Olympiad geometry document for reference 179

Q.E.D.

Problem 95: (Telv Cohl) ⋆ ⋆ Let I, O be the incenter, circumcenter of △ABC, respectively.
Let (wa ), (wb ), (wc ) be the A, B, C−mixtilinear incircles of △ABC and they are tangent to
(ABC) at X, Y, Z. Let P be a point on OI and X1 = XP ∩ (wa ), Y1 = Y P ∩ (wb ), Z1 =
ZP ∩ (wc ). Prove that: AX1 , BY1 , CZ1 are concurrent

Solution: (Telv Cohl)

Diagram M.Di.159

Let XP intersects (ABC) again at X2 .Define Y2 , Z2 similarly.


Let AX1 intersects incircle (I) at X3 .Define Y3 , Z3 similarly. (X3 closer to A and similar to
B3 , C3 )
Let (I) tangent to BC, CA, AB at D, E, F.
Let △D1 E1 F1 be the midarc triangle of △ABC.

It’s well known that AX, BY, CZ are concurrent at T which lie on OI.
 
Y, E1 , C
By Pascal theorem for we get the intersection of Y F1 , ZE1 lie on OI.
Z, F1 , B
Olympiad geometry document for reference 180

 
Y, E1 , Z2
By Pascal theorem for we get the intersection of E1 Y2 , F1 Z2 lie on OI.
Z, F1 , Y2
Similarly, we can prove the intersection of D1 X2 , E1 Y2 lie on OI =⇒ D1 X2 , E1 Y2 , F1 Z2 are
concurrent on OI. (⋆)

Since A, X is the exsimilicenter of (I), (wa ); (wa ), (O), respectively, so we get D1 X2 , DX3
are the corresponding line of (O), (I). Similarly we have EY3 , E1 Y2 and F Z3 , F1 Z2 are the
corresponding lines of (I), (O). so from (⋆) we get DX3 , EY3 , F Z3 are concurrent (on OI) ,
hence from Steinbart theorem we get AX3 ≡ AX1 , BY3 ≡ BY1 , CZ3 ≡ CZ1 are concurrent .

Q.E.D.

Problem 96: (andria) Let △ABC be a triangle with A, B, C−mixtilinear incircles respec-
tively are ωa , ωb , ωc tangent to (ABC) at X, Y, Z. I is the incenter of △ABC and IX cut
BC at D. Define E, F similarly. Let Ge be the Gergonne point and G be the centroid of
△ABC. Prove that: GGe , AD, BE, CF are concurrent.

Solution:

Diagram M.Di.160
Olympiad geometry document for reference 181

Let M be the midpoint of arc BAC of (ABC). It is well known that M, I, D, X. Let △Ia Ib Ic
be the excentral triangle of △ABC and △U V T be the tangential triangle of △Ia Ib Ic . By
Problem 75 and notice that I is the orthocenter of △Ia Ib Ic so U, A, D,similarly T, B, E
and V, C, F .

Diagram M.Di.161

It is easy to prove that BC//T V, CA//V U, AB//U T so U A, T B, V C are concurrent at W


which is the homothetic center of △U V T , △ABC. Let incircle (I) tangent to BC at D′ , it
D′ B Ia T
is easy to show that ′ = =⇒ W, D′ , Ia . Similarly define E ′ , F ′ and similarly we get
DC Ia V
Ia D′ , Ib E ′ , Ic F ′ are concurrent at W. So W is also the homothetic center of △D′ E ′ F ′ , △Ia Ib Ic
=⇒ W Ge passes through the symmedian point K of △Ia Ib Ic . But K is also the complement
point of Ge in △ABC
(Notice that BC//T V, CA//V U, AB//U T and also D′ E ′ //Ia Ib , E ′ F ′ //Ib Ic , F ′ D′ //Ic Ia )
So that W, G, Ge .

Q.E.D.

Problem 97: Given △ABC with the A−mixtilinear incircle tangent to (ABC) at X. (I)
Olympiad geometry document for reference 182

is the incircle of △ABC.The radical axis of (ABC), (I) intersects the polar of T with (I)
at W. (I) is tangent to BC, CA, AB at D, E, F. DD′ is the diameter of (I) and S lie on (I)
such that DS ⊥ EF. Prove that: W, S, D′ .

Solution:

Diagram M.Di.162

Let M be the midpoint of arc BAC so it is well known that T, I, M .DD′ is the diameter of
(I).
Let the tangent lines from T to (I) touch (I) at U, V and cut (ABC) at K, L. By Poncelet
porsim KL is tangent to (I), also it is well known that KL//BC and tangent to (I) at D′ .
It is easy to see that T U IV is cyclic. Let (T U IV ) cut (ABC) again at R. By radical axis
theorem for (I), (ABC), (T U IV ) so RT passes through W. By Example M.E.5 so A, S, T .
Consider the well known lemma for △T KL with (I) touch KL at D′ and M being the
midpoint of arc KL so R, D′ , M . Since SD′ //EF//AM so by Reim SD′ T R is cyclic. By
radical axis theorem for (I), (ABC), (RSD′ T ) so SD′ , U V, RT are concurrent.

Q.E.D.

Problem 98: (Dogu) Let △ABC be a triangle with incircle (I) tangent to BC, CA, AB at
D, E, F . M is the midpoint of arc BC of (ABC) not containing A. M F intersects (ABC)
again at R,M D intersects (ABC) again at K. KB intersects EF at Z. RZ intersects (ABC)
again at W . W D intersects (ABC) again at Q. AP is the diameter of (ABC). Prove that:
Quadrilateral QBP C is harmonic.
Olympiad geometry document for reference 183

Solution:

Diagram M.Di.163

 
R, K, A
Let the A−mixtilinear incircle to touch (ABC) at T .By Pascal for so we get
B, W, M
the intersection J of KM, AW lie on ZF or EF. Let U be the midpoint of BC then let U P
cut (ABC) again at L, as proved in the proof of Problem 28 so if we let LT cut BC at G
so A, J, G, W . We have
QB DB W C DB GC AB DB LC T C AB LC
= · = · · = · · · =
QC DC W B DC GB AC DC LB T B AC LB

(As mentioned in the comments of Theorem M.T.5 & Theorem M.T.6)

UC P B PB QB PB
= · = =⇒ =
UB P C PC QC PC
So that QBP C is harmonic.

Q.E.D.

Problem 99: (Dogu) Let △ABC be a triangle with A−excircle (J) tangent to AB, AC
at F, E and BC at D. (AEF ) intersects BC at distinct U, V . The A−symmedian line of
△ABC cut (ABC) again at K, (ADJ) intersects (ABC) again at W. Prove that: U KW V
lie on a circle.

Solution:
Olympiad geometry document for reference 184

Diagram M.Di.164


Perform an inversion centered at A with radius AB · AC then reflects over bisector AJ. J
becomes I which is the incenter, (J) becomes the A−mixtilinear incircle ω. D becomes T
where ω touches (ABC). E, F becomes E ′ , F ′ where ω touches AC, AB. The circle (AU V )
becomes the line E ′ F ′ ,the points U, V becomes Y, X. K becomes midpoint M of segment
BC and W becomes L which is the intersection of IT and BC. Now our task is to prove
X, L, M, Y lie on a circle.
Let XY cut BC at S, it is well known that (SL, BC) = −1 so my Maclaurin we have
SL · SM = SB · SC = SX · SY =⇒ XLM Y is cyclic.

Q.E.D.

Message: We will end this grand section with a super hard problem.

Problem 100: ⋆ ⋆ ⋆ (Nguyen Minh Quang) Let ABC be a triangle is incribed (O), incenter
I. △Ia Ib Ic is cevian triangle of I wrt. △ABC. Let ωa , ωb , ωc are the A, B, C-mixtilinear
incricle of △ABC,respectively. The tangents from Ib , Ic (̸= AC, AB) to ωa intersect at Pa .
Similary, we define for Pb , Pc .

i) Prove that APa , BPb , CPc are concurrent.

ii) A∗ , B ∗ , C ∗ are touch points of ωa , ωb , ωc with (O),respectively.


Prove that A∗ Pa , B ∗ Pb , C ∗ Pc , are concurrent on OI.

Solution: (Telv Cohl)


Olympiad geometry document for reference 185

Lemma: Given △ABC with a point P on BC. Let Q be the isotomic conjugate of P wrt.
B, C and let the isogonal conjugate of AP, AQ wrt. ̸ A, respectively cuts (ABC) at P ∗ , Q∗ ,
respectively. Then the tangent of (ABC) passing through A passes through the intersection
V of BC and P ∗ Q∗ .

Proof:

Diagram M.Di.165

Let ℓ be the line passing through A and parallel to BC. Since ℓ is fixed under the involution
that swaps (AB, AC) and (AP, AQ), so the isogonal conjugate of ℓ wrt. ̸ A is fixed under the
involution that swaps (AB, AC) and (AP ∗ , AQ∗ ) =⇒ BC, P ∗ Q∗ and the tangent of (ABC)
passing through A are concurrent so that AV is tangent to (ABC) at A and we are done.

i) Let the perpendicular from I to AI cuts BC, Ib Ic at D, D∗ , respectively. It is well known


that DD∗ is the polar of A wrt. ωa , so APa is the polar of D∗ wrt. ωa (well-known property
of tangential quadrilateral) =⇒ A(BC, D∗ Pa ) = −1.

On the other hand, from A(DI, BC) = A(D∗ I, BC) we know AD∗ and AD are isogonal
conjugate wrt. ̸ A, so APa passes through the isogonal conjugate (wrt. △ABC) of the or-
thocorrespondent of I wrt. △ABC =⇒ APa passes through the Mittenpunkt Mt of △ABC.
Similarly, we can prove Mt lies on BPb and CPc , so APa , BPb , CPc are concurrent at Mt .

ii) Let Ja , Jb , Jc be the A−excenter, B−excenter, C−excenter of △ABC, respectively. Let


Ma , Bb , Mc be the midpoints of arc BC, arc CA, arc AB, so clearly they are also the midpoints
of Jb Jc , Jc Ja , Ja Jb . Let O be the circumcenter of △ABC and let Ka be the exsimilicenter of
(O), (Ja ) (Ka ∈ AA∗ (Monge D’Alembert theorem)).
Olympiad geometry document for reference 186

Diagram M.Di.166

Since △Ja Jb Jc and △Ma Mb Mc are homothetic, and notice the circumcenter Be of △Ja Jb Jc
is the reflection of I in O we get Ja Ma , Jb Mb , Jc Mc are concurrent at R lie on OI. Since the
second intersection Ta of Ja Ma and (O) is the tangency point of (O) and the A-mixtilinear
excircle Ωa of △ABC (well known), so from D’Alembert theorem we get Ka Ta passes through
the insimilicenter of (Ja ), Ωa =⇒ Ta (V Ka , OR) = Ta (AKa , OJa ) = −1 where V ≡ ATa ∩
OI is the insimilicenter of (I), (O), hence S ≡ Ka Ta ∩ OI is the harmonic conjugate of V
wrt. O and R.

Let L ≡ A∗ Pa ∩ Ib Ic . Since Ib Ic is the polar of Ka wrt. (O) (well-known), so the pole G


of AA∗ wrt. (O) lies on Ib Ic . From the dual of Desargue involution theorem =⇒ A∗ G is
fixed under the involution that swaps (A∗ Ib , A∗ Ic ), (A∗ A, A∗ Pa ), so AG is fixed under the
involution that swaps (AB ≡ AIc , AC ≡ AIb ) and (AA∗ , AL), hence if Q ≡ AG ∩ BC then
the intersection of QA∗ and AL lies on (O) =⇒ A, L, Ta are collinear (from the Lemma
above).
Olympiad geometry document for reference 187

Let U ≡ AA∗ ∩ OI be the exsimilicenter of (I), ⊙(O). Let X1 and X2 be the intersection
of (O) with OI. Since L lies on the polar Ib Ic of Ka wrt. (O), so the intersection Z of Ka Ta
and A∗ Pa lies on (O), hence from Desargue involution theorem (for AA∗ ZTa ) we get W ≡
A∗ Pa ∩ OI is the image of V under the involution that swaps (S, U ), (X1 , X2 ).

Similarly, we can prove W lies on B ∗ Pb and C ∗ Pc , so we conclude that A∗ Pa , B ∗ Pb , C ∗ Pc


are concurrent on OI.

Q.E.D.
Olympiad geometry document for reference 188

Tests

Message: After huge effort of practice, now it is time to test yourself with some non-solution
problems. Try solve each problem with the time about 30 to 120 minutes because in the
test room, you don’t have much time, but if the problem is real hard so you can spend more
time if you want.

Test Problem Set 1

Problem 1: (VMO 2023) Let △ABC be a scalene triangle with orthocenter H and circum-
center O. Incircle (I) of the △ABC is tangent to the sides BC, CA, AB at M, N, P respec-
tively. Denote ΩA to be the circle passing through point A, external tangent to (I) at A′ and
cut again AB, AC at Ab , Ac respectively. The circles ΩB , ΩC and points B ′ , Ba , Bc , C ′ , Ca , Cb
are defined similarly. Prove that: Bc Cb + Ca Ac + Ab Ba ≥ N P + P M + M N .

Problem 2: Let △ABC be a triangle with the B, C−mixtilinear incircles tangent to BC at


U, V . I is the incenter of the triangle. Prove that: The radical axis of the B, C−mixtilinear
circle passes through the symmedian point of △IU V .

Problem 3: Let △ABC be a triangle with incircle (I),the tangent line of (I) which is
parallel to BC intersects AB, AC at E, F . A circle ω is internal tangent to the incircle (I ′ )
of △AEF at T . Prove that: There exists the common tangent to (I ′ ), B, C−mixtilinear
incircles of △ABC and it passes through T .

Problem 4: ⋆ Let △ABC be a triangle with A−mixtilinear circle tangent to (ABC) at X.


From X perpendicular to XA intersects the mixtilinear circle at R. AR cut (ABC) at Z
and BC at U . S is the esximilicenter of (ABC) and incircle (I). Prove that:

i) (Tran Viet Hung) SZ, U X, AI are concurrent.

ii) (Dsoong) SU, AO, XZ are concurrent with O is the center of (ABC).
Olympiad geometry document for reference 189

Test Problem Set 2

Problem 1: (Le Viet An) In triangle △ABC,the incircle (I) touches segment BC at D,
the A−excircle touches segment BC at E, the A−mixtilinear incircle touches circle (ABC)
at T. DT intersects AE at S. Prove that the incenter of triangle △ABC is the incenter of
triangle △AT S.

Problem 2: Let △ABC be a triangle with A, B, C−mixtilinear incircles tangent to (ABC)


at X, Y, Z. AD, BE, CF are bisectors of △ABC and D, E, F lie on BC, CA, AB. Prove that:
(AXD), (BY E), (CZF ) are coaxial.

Problem 3: Let △ABC be a triangle with incenter I. M is the midpoint of arc BC


of (ABC) not containing A. The A−mixtilinear incircle is tangent to (ABC) at T . T M
intersects the perpendicular bisector of AI at R. The tangent line from A to (ABC) intersects
BC at S. Prove that: RS//AI.

Problem 4: Let △ABC be a triangle with AI cut BC at J. Suppose J is the center of the
A−mixtilinear incircle BC. AD, BE, CF are altitudes of △ABC.

i) (Sharygin 2015) Prove that: EF is tangent to incircle (I) and DF, DE are tangent to
the C, B−excircles of △ABC.

ii) Let Ω1 , Ω2 be the excircles of △BDF , △CDE. Prove that: RΩ1 = RΩ2 .
Olympiad geometry document for reference 190

Test Problem Set 3

Problem 1: (Serbia JBMO TST 2022) Let I be the incenter of △ABC. A1 , B1 are midpoints
of sides BC, AC. M, N are midpoints of arcs AC, BC of (ABC) which does contain the other
vertex of the triangle. Suppose M, I, N ,prove that: ̸ AIB1 = 90◦ = ̸ BIA1 .

Problem 2: (Korea Winter Program 2022) Let ABC be an acute triangle with incenter
I and circumcircle Ω. The line passing I and perpendicular to AI meets AB, AC at D, E,
respectively. A-excircle of △ABC meets BC at T . AT meets Ω at P . The line passing
P and parallel to BC meets Ω at Q. The intersection of QI and AT is K. Prove that
Q, D, K, E are concyclic.

Problem 3: Let △ABC be a triangle with J is the radical center of the A, B, C−mixtilinear
incircles.If O, (I) are the circumcenter,incircle of △ABC. Prove that:

OJ : JI = 2R(ABC) : −R(I) .

Problem 4: Let ABC be a triangle and let I be its incenter. Denote by A′ , B ′ , C ′ the
intersections of AI, BI, CI with the sides BC, CA, and AB, respectively. Prove that: The
isogonal conjugate of I with respect to triangle A′ B ′ C ′ lies on the line OI, where O is the
circumcenter of triangle ABC.
Olympiad geometry document for reference 191

Test Problem Set 4

Problem 1: (Ravi Painuly) You are given 4 million dollars. A smart stone gives you 3 lines
intersecting at A, B, C and tangency points D, E, F of incircle with BC, CA, AB (Note: The
incircle is not given).The stone gave you the task of constructing a circle passing through
A, D tangent to the incircle using only the following tools some of them having some cost.

i) Constructing the circumcircle of 3 marked points costs you 1 million dollars.

ii) Constructing the circle having 2 marked points as diameter costs Enots 1 million
dollars.

iii) Marking the intersection of any 2 objects is free.

If you can complete the task, the stone will give you the ultimate,sumpreme power of geom-
etry. Now help yourself to complete the task.

Problem 2: (Tran Viet Hung) Let △ABC be a triangle with the A−mixtilinear incir-
cle,excircle ω, Ω tangent to (ABC) at X, Y. One of the internal common tangent of ω, Ω
touch ω, Ω at W, W ′ . Prove that: XW X ′ W ′ is cyclic. (Ignore the other tangent line)

Problem 3: (ferma2000) Let ABC be a triangle. The incircle (I) of △ABC touches
BC, CA, AB at D, E, F respectively. ϱ denotes a circle passing throw B, C that is tangent
to (I). ϱ ∩ AC = S, ϱ ∩ AB = T .J denotes the A-excenter of △AST . SJ ∩ EF =
K, T J ∩ EF = L. O is the circumcenter of △JKL. M is midpoint of JO. Prove that: The
triangle formed by reflections of M with respect to AI, BI, CI is perspective with △DEF .

Problem 4: (Dogu) Let △ABC be a triangle with incircle center I.B ′ , C ′ lie on the plane

such that △IBC ∼ △IB ′ C ′ and B ′ , A, C ′ . B ′ C ′ intersects (ABC) again at W . IB ′ , IC ′
intersects BC again at X, Y . The A−mixtilinear incircle is tangent to (ABC) at T.

i) Prove that: (W XY ) passes through T.

ii) Prove that: The tangent lines from X, Y to incircle (I) intersects at S lie on AW .

iii) Let BB ′ intersects CC ′ at W . Prove that: SW, BC, AI are concurrent.


Olympiad geometry document for reference 192

Test Problem Set 5

Problem 1: (andria) Let ABC be a triangle. let ω be its incircle. ω touches BC, CA, AB
at D, E, F respectively. let R, T be the projections of A, D onto EF respectively. let M be
the midpoint of AR. Let M T ∩ ω = N . Prove that RT N D is cyclic quadrilateral.

Problem 2: (Peru NMO 2012) Let ABC be a triangle with AB = BC.Let D be a point in
AC and let γ be a circumcircle of ABC , let wa be a circle such that is tangent a to segment
DA, DB and γ with center is X similary defined wc with center Y . Prove that: BD is
bisector of ̸ XBY .

Problem 3: (Taiwan TST Mock 2023) Given triangle ABC with A-excenter IA , the foot
of the perpendicular from IA to BC is D. Let the midpoint of segment IA D be M , T lies
on arc BC(not containing A) satisfying ̸ BAT = ̸ DAC, IA T intersects the circumcircle of
ABC at S ̸= T . If SM and BC intersect at X, the perpendicular bisector of AD intersects
AC, AB at Y, Z respectively, prove that AX, BY, CZ are concurrent.

Problem 4: (livetolove212) ⋆ ⋆ Let △ABC be a triangle with a point P lie on the plane
satisfying AB + BP = AC + CP. CP cut AC at E and CP cut AB at F . Let Y be the
second intersection of CA and (BP C),let Z be the second intersecton of AB and (BP C).

i) Prove that: The exsimilicenter of the incircles of △AP Y , △AP Z lie on BC.

ii) Prove that: There exists two circles tangent to four circles,the incircles of
△BP F , △CP E, △BEY , △BF Z and there exists a circle has the same center with
(BP C).
Olympiad geometry document for reference 193

Lemmas & Definitions list

Message: Here are the lists of well known lemmas mentioned in the solutions and we won’t
present the proof, instead we use them directly.

1. (Homothety) Three points A, B, C lie on a circle Ω. A circle passes through A, tangent to


Ω at A and tangent to BC at T . Then A, T, M with M is the midpoint of arc BC of Ω not
containing A.

Diagram M.Di.167

2. (Homothety) Let (O1 ), (O2 ) be two circles with A1 , A2 lie on (O1 ), (O2 ) respectively such
that O1 A1 //O2 A2 and A1 , A2 lie on the same half plane of O1 O2 . Then A1 A2 and the exter-
nal tangent lines of (O1 ), (O2 ) are concurrent,similarly we state the statement for internal
tangent.
Olympiad geometry document for reference 194

Diagram M.Di.168

3. (Steiner equation) Let △ABC be a triangle with points E, F lie on BC so AE, AF are
isogonal in ̸ BAC if and only if
EB F B AB 2
· =
EC F C AC 2

Diagram M.Di.169

4. (Cyclic quadrilateral ratio lemma) Let A, B, C, D be four points lie on circle Ω. Suppose
AC intersects BD at E then
AB CB EB
· =
AD CD ED

Diagram M.Di.170
Olympiad geometry document for reference 195

5. (Definition of circumconic) Let △ABC be a triangle.If C passes through A, B, C such that


the locus of the points which are isogonal conjugate to the points lie on C wrt. △ABC is a
line d so C is a conic, or the circumconic of △ABC.

Diagram M.Di.171

6. Poncelet porism,Dersagues theorem,... (Readers can find out themselves)

7. (Method of Moving Points) Let C1 , C2 be objects that cross ratio is defined (lines,conics,pencils
of lines,...) .If f1 : C1 −→ C2 with f2 : C1 −→ C2 are two projective maps so f1 ≡ f2 if and
only if f1 = f2 holds for three distinct points on C∞ .

(Definition by Vladyslav Zveryk)

8. (Definition of Involution) Let P be the set of all points lie on line or conic. The function
f : P −→ P is call an involution if it satisfies two conditions:

i) f preserves cross ratio.

ii) f (f (A)) = A for A ∈ P.

(Involution on a pencil of lines is defined merely similarly)

9. (Dersagues Involutions’ Theorem) Quadrilateral ABCD inscribed in a conic C. A line


ℓ cut AB, CD, AD, BC, AC, BD at X1 , X2 , Y1 , Y2 , Z1 , Z2 and cut C at W1 , W2 then pairs
(W1 , W2 ), (X1 , X2 ), (Y1 , Y2 ), (Z1 , Z2 ) are reciprocal pairs of some involution on ℓ.
Olympiad geometry document for reference 196

10. (Dual of Dersagues Involutions’ Theorem) Let P, A, B, C, D be points on a plane with


AB ∩ CD = E, AD ∩ BC = F . Let a conic C tangent to lines AB, CD, AD, BC. Let
P X, P Y be the tangent lines from P to C. Then (P X, P Y ), (P A, P C), (P B, P D), (P E, P F )
are reciprocal pairs of some involution on pencil of lines pass through P .

(Definitions by MarkBcc168)

11. (Pitot’s theorem) Quadrilateral ABCD inscribed a circle (I) if and only if AD + BC =
AB + CD. Similarly,readers can change the form of the theorem (the inscribed circle change
place).

Diagram M.Di.172

12. (Summarize on Intraversion) When a property is true for the incircle,when we change
the configuration to excircle,it remains true.

Diagram M.Di.173
Olympiad geometry document for reference 197

13. (Schwatt line) Let △ABC be a triangle with altitudes AH. M, N are midpoints of
BC, AH. The the A−schwatt line M N passes through L which is the symmedian point of
△ABC.

Diagram M.Di.174

14. (Steinbart theorem) Let △ABC be a triangle with incircle (I) tangent to BC, CA, AB
at D, E, F. P lie on the plane. △XY Z is the circumcevian triangle of P wrt. △DEF . Then:
AX, BY, CZ are concurrent.

Diagram M.Di.175

(We proved the generalization of this theorem in the proof of Problem 37)

15. (Carnot theorem) In triangle ABC with point CA , CB lie on AB. AB , AC lie on BC and
Olympiad geometry document for reference 198

BC , BA lie on AC. So they lie on a common conic if the below equality holds
|ACA | |ACB | |BAB | |BAC | |CBC | |CBA |
· · · · · =1
|BCA | |BCB | |CAB | |CAC | |ABC | |ABA |

Diagram M.Di.176

16. (Ceva nest theorem) Let △ABC be a triangle with point P . △DEF is the cevian triangle
of P wrt. △ABC. X, Y, Z lie on EF, F D, DE. Then DX, EY, F Z are concurrent if and
only if AX, BY, CZ are concurrent.

Diagram M.Di.177

17. (Sawayama lemma) Let △ABC be a triangle with incircle (I). D lie on segment BC.
ω is a circle tangent to DA, DC at X, Y and internally tangent to (ABC). Here ω is the
Thébault circle respects to AD,vertice C of △ABC. Then X, I, Y .
Olympiad geometry document for reference 199

Diagram M.Di.178

18. (Pappus theorem) Let d, d′ be two lines on the plane. A, B, C lie on d and A′ , B ′ , C ′ lie
on d′ . The the intersections of pairs AB ′ , BA′ ; AC ′ , CA′ ; BC ′ , CB ′ are collinear.

Diagram M.Di.179
Olympiad geometry document for reference 200

Sources & Signs & Final Messages

Sources:
1. Duong tron Mixtilinear - Nguyen Van Linh
https://nguyenvanlinh.files.wordpress.com/2013/11/mixtilinear.pdf

2. AoPs forum
https://artofproblemsolving.com/community

3. Andreescu Korsky ,Cosmin Pohoata - Lemmas in olympiad


https://cdn.bc-pf.org/resources/math/geometry/Andreescu Korsky Pohoata-Lemmas in olympiad
geometry.pdf

4. Romantics of Geometry
https://www.facebook.com/groups/parmenides52/

5. Hinh hoc phang


https://www.facebook.com/groups/hinhhocphang.geometry/

6. Jean-Louis Ayme Geometry blog


http://jl.ayme.pagesperso-orange.fr/

7. Nguyen Van Linh blog


https://nguyenvanlinh.wordpress.com/

8. On Mixtilinear circles - Jafet Baca


https://www.awesomemath.org/wp-pdf-files/math-reflections/mr-2020-02/mr 2 2020 mixtilinear.
pdf

9. Geometry Discussion & Problems Discord server


https://discord.gg/pYd4p7b22y

10. IMO Shortlist 2019


https://www.imo-official.org/problems/IMO2019SL.pdf

11. Viet Nam IMO Booklet 2017


https://drive.google.com/file/d/1l9PGjsCVXW5LFhYx4xJfRg1T82VC8hs3/view
Olympiad geometry document for reference 201

Coloring & Staring:


Meanings of our coloring:
Inspired by the five elements.
Lime (Definition) - Stands out for the easiness of this part.
1. Gold/Earth (Theories) - The most important part. The basic of everything.
2. Metal (Examples) - Lasting,sure metal steps for beginners to start and becomes better,
understand the usage of the basics.
3. Water (Extra) - Water is an essential part of life, water is huge, cosmological, just like
the extra part.
4. Green (Problems & Solutions) - Green is nature and it stands out like a forest, in here
a forest of practice problems with solution.
5. Red (Advanced/Hard problem) - Fire is aggressive, and the problems are to.
With all five elements combined, you will master the Mixtilinear secret magic.
Meanings of our staring:
(Using the scale of IMO SL)
0 star: Below the hard rate. (G1 - G6)
1 star: ⋆ Hard. (G5 - G6)
2 stars: ⋆ ⋆ Very hard. (G6 - G8)
3 stars: ⋆ ⋆ ⋆ Super hard. (G8+)
Unusual signs used in the book:

1. (AB): the circle with diameter AB.(ABC): The circumcircle of △ABC. Similar for
other inscribed polygons.
+ −
2. ∼, ∼: Similar the same direction or similar the different direction.
3. Rω : Radius of ω.
X
4. =: X is the explaination for the equation.
5. (AB, CD): Harmonic bundle, or we can write as (A, B; C, D).
6. AB: Algebraic sign of length.
7. X, Y, Z: X, Y, Z are collinear.
8. ≡: Equivalent to.
9. [ABC]: The area of △ABC.
10. A−mixtilinear,A−X;circles/median/...: Corresponding to angle A.
11. (O,
 OA) or(O; OA) or (O; R), (O, R): The circle with center O and radius OA or R.
A, B, C
12. : Pascal theorem applied for six points A, B, C, A′ , B ′ , C ′ .
A′ , B ′ , C ′
Olympiad geometry document for reference 202

Final message:

Dear readers,
Thank you for reading our books and trusting our works. We hope after you read this book,
somehow you will get a lot better at geometry and get a lot of enjoyment practicing with it.
Again, I want to say thank you to all the contributors of this book ,without them the book
could not be completed

(Picture showing avatars of authors,contributors team of the book - Discord)

(Picture showing avatars of authors,contributors team of the book - AoPs)


Olympiad geometry document for reference 203

In this message, I also want to say something about our effort and inspiration too. This
book is written from our love and passion for geometry, it is a free gift from us for every
geometers, or else we can say this is a work dedicated to the public. The books contain many
nice,exciting solutions from us and those are many of our efforts working days and nights
too. We also introduce some very creative,insane solutions from other geometers for many
hard problems which we hope you also understand the goodness of it.

For us, this is the first time ,we, ever work and cooperate together. This can be regarded as a
”debut” from us. Although this is the first cooperation, but in my opinion, this cooperation
is so great that it creates such a master piece like this book.

In conclusion, we hope everyone will always support us and our geometric works. We will
consider on working on more great handouts like this.

Regards,

土偶

You might also like